MCQs For The New MRCPsych Paper
MCQs For The New MRCPsych Paper
MCQs For The New MRCPsych Paper
MRCPsych Paper A
with answers explained
Edited by
David Browne, Selena Morgan Pillay,
Guy Molyneaux, Brenda Wright,
Bangaru Raju, Ijaz Hussein,
Mohamed Ali Ahmed and Michael Reilly
Index191
iii
David Browne qualified in Medicine from University College Galway. He completed the
MRCPsych in 2002. He was a Stanley Research Fellow with the Royal College of Surgeons
in Ireland and carried out research in the area of epidemiology of first-episode and prevalent
psychoses. He has an interest in medical education and training. He was a member of the
Royal College of Psychiatrists, Psychiatric Training Committee. He completed a Masters in
Medical Education with Queens University Belfast in 2011 and a Masters in Leadership and
Management Development with the Royal College of Surgeons in Ireland in 2014. He is a
certified coach with the International Coaching Federation. In 2002 he co-authored MCQs
for the New MRCPsych, Part II; in 2009 he co-authored MRCPsych Paper 1, One Best Item
MCQs. He is a consultant psychiatrist in Beaumont Hospital, Dublin.
Selena Morgan Pillay qualified in Medicine from University College Dublin in 2001. She
began basic specialist training in psychiatry with the Dublin University Psychiatric Rotational
Training Programme in 2002 and obtained her Membership of the Royal College of
Psychiatry in 2005. She completed higher training in General Adult Psychiatry with a special
interest in Forensic Psychiatry. During this time, she spent two years working as a Lecturer
in Psychiatry, in Trinity College, Dublin, and in the Royal College of Surgeons in Ireland. She
obtained a Masters in Medical Education in 2011 from Queens’s University Belfast. She is a
Senior Clinical Lecturer with the Royal College of Surgeons in Ireland and has been involved
in the development of their international psychiatry teaching programmes in Perdana and
Bahrain. She is currently working as a Consultant Psychiatrist in Beaumont Hospital.
Dr Guy J. Molyneux, MB, BCH, BAO, MRCPsych, Postgraduate Diploma in CBT, Diploma
in Management; Clinical Director in Psychiatry at HSE Dublin North City and County Mental
Health Services and MMUH, and Consultant in Adult Psychiatry at St Vincent’s Hospital,
Fairview, Dublin 3, Ireland
Guy Molyneux received his medical degree (MB BCh BAO) at Dublin University in 1996. He
has a special interest in Cognitive Behavioural Therapy and his research on carer burden
and mental Illness has been published in national and international scientific journals.
Brenda Wright qualified in Medicine from UCD in 1997. She completed her psychiatric
training with the St John of God Rotation in Dublin, having obtained her Membership of the
Royal College of Psychiatry in 2002. She is currently a Consultant in Forensic Psychiatry
vii
with the National Forensic Mental Health Service. She previously held the post of Lecturer
in Forensic Psychiatry with the National University of Ireland, Trinity College, Dublin. She is a
member of the Faculty of Forensic and Legal Medicine of the Royal College of Physicians.
She has done research particularly in the areas of psychiatric morbidity in prison and
cognitive patterns in sex offenders.
Mohamed Ali Ahmed graduated in 1996 and worked in Jordan, Sudan and Saudi Arabia
prior to completing his basic psychiatric training on the Western Health Board Psychiatric
Training Scheme. He then completed a higher diploma in clinical teaching and participated
in teaching postgraduate psychiatric trainees in the west of Ireland. His special interests are
transcultural psychiatry, neuroimaging and metabolic disorders in schizophrenia.
Ijaz Hussain graduated in 1999 and worked in Ireland. He completed his basic psychiatric
training on the Western Health Board Psychiatric Training Scheme. He developed an interest
in teaching as an senior house officer and worked as a Clinical Lecturer with the Department
of Psychiatry, National University of Ireland, Galway. He has successfully organised courses
for the MRCPsych exam under the old and new curriculum, and is experienced in the recent
changes in psychiatric training.
Raju Bangaru qualified in Medicine from the University of Madras, India, in 1981. He
completed basic and higher psychiatric training at the Institute of Psychiatry in Madras.
He worked as Assistant Professor of Psychiatry in the University of Madras for nine years.
Subsequently, he completed basic and higher psychiatric training in Ireland. His MD
(Madras) thesis was on the dexamethasone suppression test. He has completed an MBA in
Health Service Management on the UCD/RCSI programme. He has co-authored MCQs for
the New MRCPsych, Part I; MCQs for the New MRCPsych Part II and Extended Matching
Items for the MRCPsych, Part 1. His interests include medical education, forensic psychiatry,
mood disorders and administration. Since 2006 he has been the Clinical Tutor in Psychiatry
of Dublin Northwest Area, and since 2007 he has been the Programme Co-ordinator of
RCSI Postgraduate Psychiatric Training Programme. He is the current chair of Continuing
Professional Development sub-committee of the Irish College of Psychiatrists.
Michael Reilly qualified in Medicine from UCD in 1995. He completed basic specialist training
in Psychiatry in 1999 on the Western Health Board Psychiatric Training Scheme. He spent
two years researching biological and clinical correlates of suicidal behaviour as part of
the INSURE Collaborative Project on suicidal behaviour. He completed higher psychiatric
viii
training in the Cavan/Monaghan Mental Health Services and in the Department of Psychiatry,
University College Hospital, Galway. He has co-authored a number of MCQ books for
the MRCPsych examinations: MCQs for the New MRCPsych, Part I; MCQs for the New
MRCPsych, Part II; Extended Matching Items for the MRCPsych, Part 1 and MRCPsych
Paper 1, One Best Item MCQs. His interests include medical education, psychiatric ethics
and the psycho-social treatment of severe and enduring mental illness.
ix
xi
xii
1. A 25-year-old woman with a history of schizoaffective disorder has been referred by her GP to your
outpatient clinic. She is six weeks pregnant. Her GP requests a review of her medication with regard
to teratogenicity. Which of the following medications needs to be most urgently reviewed?
A. Carbamazepine
B. Clonazepam
C. Fluoxetine
D. Olanzapine
E. Sodium valproate
A. Akathisia
B. Anxiety
C. Catatonic excitement
D. Mania
E. Restless leg syndrome
3. A 33-year-old woman with a history of schizoaffective disorder is referred for assessment by the
obstetrics team. She has just given birth to a healthy baby and wishes to breastfeed. They are
seeking immediate advice regarding her medication and the risks of breastfeeding. Which of the
following medications needs to be most urgently reviewed?
A. Carbamazepine
B. Diazepam
C. Fluoxetine
D. Olanzapine
E. Sodium valproate
4. Which of the following factors would help to distinguish the presentation of malingering from
factitious disorder?
5. A 20-year-old woman presents to Casualty, having inflicted superficial cuts to her wrists. While
waiting to be seen she becomes belligerent and verbally aggressive with staff. She describes
recent paranoid ideation and says that she has begun binge eating again in recent months. The
current episode of self-harm was precipitated by a row with her boyfriend of three weeks, who she
describes as the ‘love of my life’. There is a history of unstable employment and relationships. Which
is the most likely DSM-IV diagnosis?
6. A 22-year-old man is sent for assessment by his GP for depression that is not responding to
antidepressant therapy. During the interview, the man reports that from childhood he has wanted
to be a girl. He frequently dresses and goes out in public in female clothing. He reports that he is
sexually attracted to men, which he believes is evidence of the wrong gender assignment. He has
no paraphilic interests. What is the most likely diagnosis?
7. A 65-year-old single man presents with a history of alcohol dependence syndrome. He recently
relapsed, drinking approximately 20 units of alcohol per day for the past two months. He has not
had a drink in 24 hours. He has a history of seizures on one previous occasion when withdrawing
from alcohol. Which of the following is the most appropriate approach to his management?
A. Anxiety symptoms
B. Decreased consciousness
C. Extreme fatigue
D. Runny nose and eyes
E. Vomiting
9. A 46-year-old married man with a history of dysthymia presents in the outpatient department
complaining of impotence. Which of the following medications is the most likely cause of this
problem?
A. Clomipramine
B. Mirtazapine
C. Phenelzine
D. Reboxetine
E. Venlafaxine
10. A 25-year-old woman with a diagnosis of depression is reviewed in the outpatient department. She
anxiously describes the experience of hearing voices at night in bed while she goes to sleep. Which
of the following perceptual abnormalities is this woman most likely experiencing?
A. Extracampine hallucinations
B. Hypnagogic hallucinations
C. Hypnopompic hallucinations
D. Pseudohallucinations
E. Reflex hallucinations
11. A 45-year-old woman presents for assessment, describing a long history of anxiety when out in
public. She recalls the onset of her difficulties when, as a 20-year old, she experienced her first panic
attack. She avoids social events and in recent times is leaving her home much less frequently. She
says that she is fearful of experiencing a panic attack away from home, although she has not had
a panic attack in over three months. Which is the most likely diagnosis in this case according to
DSM-IV criteria?
12. Which of the following syndromes is least associated with a psychotic illness?
A. Couvade syndrome
B. De Clerambault’s syndrome
C. Fregoli’s syndrome
D. Othello syndrome
E. Syndrome of subjective doubles
11. Answer: C. DSM-IV classifies the following disorders: panic disorder without
agoraphobia, panic disorder with agoraphobia, agoraphobia without
a history of panic disorder. Agoraphobia is not a codable disorder, so
there is no diagnosis of agoraphobia with panic disorder. Therefore
this woman has panic disorder with agoraphobia. Her avoidance
of public situations is provoked by the fear of a panic attack rather
than a fear of social performance which would characterise a social
phobia. In generalised anxiety disorder there is excessive anxiety and
worry occurring more days than not for at least six months. The focus
of the anxiety is not confined to features of an Axis I disorder, e.g. the
anxiety or worry is not about having a panic attack. Separation anxiety
disorder has an onset before the age of 18 years. (4, pp 77, 209–16)
12. Answer: A. In Couvade syndrome a person develops extreme anxiety and various
physical symptoms of pregnancy when their partner is pregnant. The
partner may have morning sickness, abdominal pains, constipation,
food craving and so on. It is thought to be a manifestation of anxiety.
It may be an expression of frustrated creativity, jealousy of the
attention paid to the pregnant partner or over-identification with the
pregnant partner. It is usually managed with reassurance. In Fregoli’s
syndrome the patient believes that ordinary people in his environment
are persecutors in disguise. In De Clerambault’s syndrome the patient
believes that another person loves him intensely. The object of the
delusion is often of higher social status. Othello syndrome involves
delusion of infidelity on the part of a sexual partner. In the syndrome
of subjective doubles the patient believes that doubles of himself
exist. (6, pp 160–3)
13. A 65-year-old right-handed man is seen in hospital following a cerebrovascular accident. At interview
he is noted to have significant difficulties with comprehension. He is also noted to speak fluently
but with numerous errors in the use of words, syntax and grammar. Which of the following best
describes the location of the cortical lesion in this man’s case?
14. A 35-year-old man with schizoaffective disorder presents with urinary retention. Which of the
following medications is most likely to be responsible?
A. Carbamazepine
B. Clomipramine
C. Clonazepam
D. Haloperidol decanoate
E. Lithium
15. An advertising company is planning a campaign for an alcoholic beverage. The television
advertisement shows a tropical beach. Two actors then walk along the sand drinking the alcoholic
beverage. What type of conditioning is the advertising company using in this advertising campaign?
A. Backward conditioning
B. Higher order conditioning
C. Simultaneous conditioning
D. Stimulus generalisation
E. Trace conditioning
16. A researcher wishes to evaluate the attitudes of the general public to people with mental health
problems. He designs a questionnaire using a Likert scale. What is the most likely source of bias
associated with the use of this questionnaire?
A. Bias to middle
B. Defensiveness
C. Halo effect
D. Hawthorn effect
E. Response set
17. A 65-year-old man is referred for assessment by the cardiology team. He has a history of unstable
angina and has become depressed. Which of the following antidepressants would be the most
suitable for treating this man’s depressive episode?
A. Clomipramine
B. Mirtazapine
C. Phenelzine
D. Reboxetine
E. Venlafaxine
13. Answer: B. This man has Wernicke’s dysphasia (sensory or receptive dysphasia),
resulting from a lesion to the angular gyrus in the posterior part of
the superior gyrus of the dominant temporal lobe. Broca’s dysphasia
(expressive or motor dysphasia) results from a lesion to the precentral
gyrus in the dominant frontal cortex. In 90% of right-handed people
the left hemisphere plays the predominant role in speech. Wernicke’s
dysphasia can be distinguished from Broca’s dysphasia by the
fluency of speech. (5, pp 159–63)
16. Answer: A. Responders on Likert scales are more likely to show a bias to the
middle or the avoidance of extreme responses. Responders to a
questionnaire on mental illness might also demonstrate the halo
effect, allowing preconceptions to influence their responses. However
this is not the result of the Likert scale design of the questionnaire.
(7, p 92)
17. Answer: B. Tricyclics and venlafaxine are the highest risk in treating patients
with cardiac disease. MAOIs and reboxetine pose a moderate risk.
Mirtazapine along with the SSRIs is believed to pose the lowest risk
in patients with cardiac problems. (2, p 207)
18. A 44-year-old man is attending for treatment of bipolar affective disorder. A routine blood
investigation reveals a raised ALP and gamma GT. Which of the following drugs is most likely to have
caused this abnormality?
A. Carbamazepine
B. Haloperidol
C. Lithium
D. Olanzapine
E. Sodium valproate
19. A 50-year-old woman is attending for treatment of bipolar affective disorder. She has been on lithium
for ten years. Which of the following blood profiles is most likely in this woman?
A. Ca2+ Phosphate
B. Ca2+ Phosphate
C. T4 T3 TSH
D. T4 T3 TSH
E. T4 T3 TSH
20. A 30-year-old man with diabetes mellitus is referred by his general practitioner for treatment of a
depressive episode. Which of the following antidepressants would be the most suitable?
A. Clomipramine
B. Fluoxetine
C. Mirtazapine
D. Phenelzine
E. Sertraline
21. A 22-year-old woman is referred by her general practitioner who reports that she has anorexia
nervosa. Following your assessment, you are of the opinion that this woman has bulimia nervosa.
Which of the following features in her history would most support your diagnosis?
A. Binging episodes
B. Body mass index 22.5
C. Morbid fear of fatness
D. Persistent preoccupation with eating
E. Self-induced vomiting
22. A 35-year-old married man, who is long-term unemployed, is referred for psychiatric assessment.
He is accompanied by his wife. He was attacked three months ago by two men who drove him
around in the boot of a car for two hours before holding a gun to his head and threatening to shoot
him. He was then beaten and left on the side of the road. You diagnose a post-traumatic stress
disorder. He also has a history of depressive disorder. Which of the following features in this man’s
case would most likely improve his prognosis?
10
21. Answer: B. Bulimia nervosa and anorexia nervosa share a number of features,
including a morbid fear of fatness, a preoccupation with eating,
vomiting or purging behaviour and binge eating. The diagnosis of
anorexia requires a BMI of <17.5, and amenorrhea. It should be
remembered, however, that 15% of patients with anorexia progress
to bulimia. (6, pp 93–100)
11
23. You have been seeing a 45-year-old man who presented with symptoms of depressive disorder
and post-traumatic symptoms following an accident at work. He has shown little response to
antidepressant medication at high doses, and the diagnosis remains unclear. You refer him for
psychological assessment. Which of the following instruments would you most likely expect the
psychologist to use?
24. A 22-year-old patient presents with a movement disorder. He demonstrates rapid sudden blinking.
This movement is more noticeable when the patient is anxious. His partner reports that it does not
occur when he is asleep, and that it is less prominent when he is concentrating on an activity. Which
movement is this patient most likely displaying?
A. Dyskinesia
B. Hemiballismic movements
C. Myoclonic movements
D. Tics
E. Torticollis
25. A 35-year-old married woman with a history of depressive disorder presents in the outpatient
department complaining of loss of libido. Which antidepressant medication would be the most
suitable to prescribe to address this side effect?
A. Fluoxetine
B. Lofepramine
C. Phenelzine
D. Mirtazapine
E. Venlafaxine
26. A 24-year-old woman with a diagnosis of schizophrenia presents to the outpatient department.
She reports that she is being sexually assaulted every night by terrorists. Which of the following
perceptual abnormalities is she most likely to be experiencing?
A. Autoscopic hallucinations
B. Haptic hallucinations
C. Hygric hallucinations
D. Hypnagogic hallucinations
E. Kinaesthetic hallucinations
12
25. Answer: D. Mirtazapine has a lower incidence of sexual side effects than most
other antidepressants at 24%. SSRIs may indirectly decrease libido
through a direct inhibition of sexual arousal and orgasm. Tricyclic
antidepressants may decrease libido indirectly through sedation.
Venlafaxine can inhibit arousal and orgasm. (2, pp 195–7)
13
27. A 26-year-old man is seen following a road traffic accident in which he sustained serious head and
limb injuries. He is noted to have a palsy in his left arm. Which of the following features would most
likely suggest that the upper-limb paralysis results from an upper rather than a lower motor neuron
lesion?
A. Babinski sign
B. Fasciculation
C. Flaccid paralysis
D. Hyporeflexia
E. Hypotonia
28. A 56-year-old man with a history of alcohol dependence syndrome is referred by the Accident
and Emergency Registrar who reports that he has Wernicke’s encephalopathy. Following your
assessment, you are of the opinion that he has progressed to Korsakoff’s syndrome. Which of
the following features would be most supportive of your diagnosis?
A. Confabulation
B. Confusion/Clouding of consciousness
C. Nystagmus
D. Peripheral neuropathy
E. Staggering gait
29. A 36-year-old woman attends for assessment at the outpatient clinic. She reports that she has
been previously diagnosed with ‘seasonal affective disorder’. Which of the following features of her
depressive disorder is least supportive of this diagnostic term?
A. Hyperphagia
B. Insomnia
C. Loss of interest
D. Low energy
E. Symptoms worsening in the months of November to February
30. A 25-year-old man who has been attending the service for treatment of anxiety disorder is involved
in a road traffic accident. Which of the following factors increases this man’s risk for significant
cognitive sequelae?
14
29. Answer: B. Seasonal affective disorder was a term used to describe patients
who recognised a seasonal pattern to their mood and described
consistent atypical symptoms of hypersomnia, hyperphagia,
tiredness and low mood in winter. Community studies have not
supported the existence of a specific syndrome, although there is
some seasonality in mood disorder. (8, p 407)
15
31. A 36-year-old man who is being treated for depressive disorder complains that he has been
experiencing sleep difficulties. He reports that his wife complains he appears frightened and
shouts in his sleep at night. He has no recollection of the incidents on waking. Which type of sleep
disturbance is this man most likely describing?
A. Narcolepsy
B. Night terrors
C. Nightmares
D. Sleep drunkenness
E. Somnambulism
32. Which of the following symptoms is least likely characteristic of temporal lobe epilepsy?
A. Automatism
B. Déjà vu
C. Echolalia
D. Olfactory hallucinations
E. Pseudohallucinations
33. A male patient with Tourette’s syndrome is referred to the clinic for assessment. The referring doctor
reports that in addition to vocal tics, he has a number of other psychiatric difficulties associated with
Tourette’s. Which of the following symptoms is this patient least likely to display?
A. Anxiety
B. Attention deficits
C. Depressive episode
D. Obsessional symptoms
E. Psychotic symptoms
34. A 55-year-old man presents with sudden onset unilateral facial paralysis. The upper and lower facial
muscles are equally affected. The eyebrow droops and the wrinkles of the forehead are smoothed
out. The muscles are equally affected for voluntary, emotional and associated movements. There
is loss of taste on the anterior two-thirds of the tongue. The patient is well otherwise. Which of the
following is the most likely diagnosis in this case?
A. Bell’s palsy
B. Horner’s syndrome
C. Multiple sclerosis
D. Poliomyelitis
E. Ramsay–Hunt syndrome
16
31. Answer: B. Night terrors occur in deep sleep early in the night. They more
typically occur in childhood, but can occur in adulthood. The person
displays intense anxiety, may shout, and has a rapid pulse and
respiration. Usually there is complete amnesia for the experience on
waking. This latter point distinguishes night terrors from nightmares,
a type of dream that is remembered vividly if the person awakes
immediately after the experience. Narcolepsy is characterised
by short episodes of sleep that occur irresistibly during the day.
Somnambulism is another term for sleepwalking. Sleep drunkenness
is the complaint of feeling drowsy, incompetent and uncoordinated
for a prolonged period of time on waking. (5, pp 40–3)
33. Answer: E. Patients with Tourette’s syndrome have been found to have an
increased prevalence of depressive, anxiety and obsessional
symptoms than normal controls. There is additionally an increased
prevalence of deliberate self-harm. There does not appear to be any
association with increased psychotic symptoms. (5, p 341, 6, p 287)
34. Answer: A. Bell’s palsy is diagnosed by the sudden onset of a unilateral facial
paralysis with or without loss of taste in the anterior two-thirds of the
tongue in a person who is otherwise well. Ramsay–Hunt syndrome
occurs with herpes zoster of the geniculate ganglia. There is a facial
palsy identical to Bell’s palsy but with herpetic lesions in the external
auditory meatus. Deafness may occur. Multiple sclerosis should be
considered as a possible cause when unilateral facial paralysis occurs
in a young adult, especially if it is painless, not very severe and clears
up in two or three weeks. Poliomyelitis should be considered as a
cause when unilateral facial paralysis occurs during an epidemic of
that disease, especially in a child or adolescent, and if it occurs a few
days after a febrile illness. (3, p 888, 9, pp 67–9)
17
35. A 36-year-old woman who was diagnosed with puerperal psychosis attends the outpatient
department accompanied by her husband. They would like to have more children but want to know
what the chances are of the illness recurring. Which of the following figures best represents this
woman’s chance of becoming psychotic again following childbirth?
A. 1%
B. 5%
C. 10%
D. 20%
E. 30%
36. A 22-year-old woman has been referred to the neurology service for assessment. She was initially
referred to a neurologist for treatment of seizures; however, the neurologist is of the opinion that
she is not experiencing true seizures. Which of the following features most likely suggests that this
woman is presenting with non-epileptic seizures?
37. A patient reports that he has been experiencing visual hallucinations. He describes seeing the face
of his sister in the clouds. Which of the following perceptual abnormalities is this man most likely
experiencing?
A. Affect illusion
B. Completion illusion
C. Elementary hallucination
D. Functional hallucination
E. Pareidolic illusion
38. A 22-year-old woman is referred for assessment by the obstetric team. She is reported to be feeling
depressed following the birth of her first baby three days ago. She has no history of depressive
disorder. The baby is healthy, although the woman required a caesarean delivery. Which of the
following is the most likely diagnosis?
A. ‘Baby blues’
B. Adjustment disorder
C. Adjustment reaction
D. Major depressive episode with post-partum onset
E. Major depressive disorder
18
35. Answer: E. Women who experience a puerperal psychosis have a one in three
chance of a further psychotic episode following a subsequent
pregnancy. This woman’s risk would be further increased by a
personal or family history of a major mental disorder. Risk is higher in
women with a history of bipolar rather than unipolar affective disorder.
(8, p 560)
36. Answer: C. Non-epileptic seizures occur very commonly in patients who also
experience epileptic seizures. The movements in non-epileptic
seizures often involve generalised rigidity with arching of the back
and random thrashing of the limbs, contrasting with the stereotypical
tonic-clonic movements in grand mal seizures. Reflexes are unaltered
in non-epileptic attacks. Incontinence is common in epileptic seizures
but rare in non-epileptic episodes. (8, p 31)
37. Answer: E. There are three type of illusions. A completion illusion occurs when
an incomplete perception that is meaningless in itself is filled in a
process of extrapolation from a previous experience to produce
significance. An affect illusion is one that can be understood in
the context of the person’s mood state, for example when a child
frightened by the dark mistakes a blowing curtain for a person in the
room. Pareidolic illusions are created out of sensory percepts by an
admixture with imagination. Functional hallucinations occur when a
certain percept is necessary for the production of a hallucination but
the hallucination is not a transformation of that percept. For example,
the patient hears voices when a tap is turned on. An elementary
hallucination is an auditory hallucination of unstructured sounds, often
occurring in an organic state. (5, pp 81–5)
38. Answer: A. Baby blues typically occur 3–6 days following the birth of the
baby in 50–60% of mothers. Post-partum depression occurs in
approximately 10% of women after parturition. It usually occurs
within 6–8 weeks of birth, most often beginning between days 3 and
14 after parturition. Adjustment disorder is usually diagnosed within
six months of the occurrence of a significant stressor, and usually
resolves within six months of the resolution of the stressful situation.
An adjustment reaction is a short-lived anxiety response to a stressful
situation. Given the time frame of this presentation, the birth and the
prevalences, the most likely diagnosis is baby blues. (4, pp 285–6,
6, pp 380–1)
19
39. A 22-year-old man with schizophrenia, an inpatient on the psychiatric ward, develops sweating
and confusion. On examination he has severe rigidity and tachycardia, and his blood pressure is
fluctuating. His temperature is 106 °F. He has been treated with aripiprazole. Which of the following
is the most likely diagnosis in this case?
A. Catatonia
B. Delirium
C. Myocardial infarct
D. Neuroleptic malignant syndrome
E. Tetanus
40. A 35-year-old woman is referred to your clinic. She reports that she worked for a prominent firm but
was overlooked for promotion on a number of occasions and left. She believes that others in her
department conspired to prevent her promotion as they were jealous of her potential, but feels that
she is overqualified for the positions available. She says that she finds it difficult to befriend women
as they are often envious of her. She reports that she is currently in a wonderful relationship, which
she says began two weeks previously with her new partner, an airline pilot. What is the most likely
diagnosis in this case?
41. A 42-year-old man presents for assessment having been transferred from another service. He tells
you that he has ‘manic depression’. Which of the following phenomena would most lead you to
doubt this diagnosis?
A. Delusional mood
B. Delusional perception
C. Delusions of persecution
D. Flight of ideas
E. Voices talking to the patient
42. A 48-year-old man attends for assessment. He has been transferred from another service and you
have been told that he has a diagnosis of schizophrenia. Which of the following symptoms would
provide the weakest evidence for this diagnosis?
A. Delusional perception
B. Delusions of passivity
C. Delusions of persecution
D. Third-person auditory hallucinations
E. Thought broadcasting
20
39. Answer: D. The diagnosis of neuroleptic malignant syndrome requires rigidity and
elevated temperature in a patient on antipsychotic medication. The
onset may be gradual or rapid, and most typically occurs in a young
male recently commenced on neuroleptics. There are case reports
in the literature of NMS occurring with aripiprazole. Catatonia may
present with rigidity but is not accompanied by pyrexia or signs of
autonomic instability. Tetanus presents with muscular spasm usually
beginning in the masseter muscle and generalising. It is accompanied
by autonomic dysfunction, but the patient is mentally alert. Delirium
presents with confusion of varying degrees and may be accompanied
by pyrexia though not by rigidity. Myocardial infarction may present
with hypotension, diaphoresis and tachycardia, but will be marked by
severe chest pain. There will be no rigidity. (10, pp 103, 106)
21
43. Which of the following drugs is least likely to cause serotonin syndrome?
A. Venlafaxine
B. Ecstasy (MDMA)
C. Fluoxetine
D. Olanzapine
E. Phenelzine
44. A woman is pregnant with her first child. Her father had schizophrenia. She is afraid that her child will
develop schizophrenia later in life. Which figure best represents her child’s lifetime risk of developing
schizophrenia?
A. 1%
B. 5%
C. 15%
D. 20%
E. 50%
45. Which of the following disorders is least likely to be an underlying cause of depressive disorder?
A. Acromegaly
B. Addison’s disease
C. Cushing’s disease
D. Hyperparathyroidism
E. Hypothyroidism
46. In the classification of personality disorders, which of the following disorders is contained in DSM-IV
but not in ICD-10?
47. Which of the following personality disorders is not part of the Cluster B of personality disorders in
DSM-IV?
22
43. Answer: D. Olanzapine is not associated with serotonin syndrome. The other
drugs have all been implicated in serotonin syndrome. Ecstasy
has been associated with serotonin syndrome in combination with
venlafaxine. (2, pp 372–3)
23
48. Which of the following scenarios best illustrates the defence mechanism of displacement?
A. An overweight woman has been advised to exercise by her GP. She decides not to take
exercise, as her parents, who were also both overweight, died with cardiac problems, and she
thinks exercising could put undue pressure on her heart.
B. A man who has been diagnosed with terminal cancer is belligerent and demanding with medical
staff.
C. A man who is dissatisfied with his work accuses his wife of being disappointed in him and his
achievements.
D. A woman notices a large, hard lump in her breast. She stops her regular breast examination and
does not attend her GP.
E. A woman whose mother recently died begins attending church regularly, having not attended
church in a number of years. Her mother was a regular church attender.
49. A 72-year-old woman is referred for assessment by her GP. He is concerned that she is experiencing
abnormal grief following the death of her husband five months previously following a long illness.
Which of the following symptoms would most likely suggest that this woman is experiencing an
abnormal grief reaction?
A. Guilt
B. Hallucinations
C. Panic attacks
D. Psychomotor retardation
E. Sleep disturbance
50. Which of the following psychiatric disorders occurs with equal frequency in men and women?
A. Agoraphobia
B. Generalised anxiety disorder
C. Panic disorder
D. Simple phobia
E. Social phobia
51. A 50-year-old man with schizophrenia is noted on examination to move his arm in response to slight
pressure on it, despite being instructed to resist the pressure. Which of the following movement
disorders best describes what he is exhibiting?
A. Ambitendence
B. Mannerism
C. Mitgehen
D. Stereotypy
E. Waxy flexibility
24
49. Answer: D. Many bereaved people experience guilt that they failed to do
enough for the deceased. About one in ten will experience brief
hallucinations. Sleep disturbance is very common, and anxiety may
occur as panic attacks. An abnormal grief reaction occurs when the
symptoms are more intense than usual and meet the criteria for a
depressive disorder, if they are prolonged beyond six months, or if
they are delayed in onset. This woman demonstrates many of the
symptoms of normal grief; however, the symptom of psychomotor
retardation is seldom present in uncomplicated grief and suggests
that she is experiencing abnormally intense grief. (13, pp 152–3)
50. Answer: E. Social phobia is the only anxiety disorder to occur equally frequently
in men and women. The other anxiety disorders occur approximately
twice as frequently in women as in men. (13, pp 166–78)
51. Answer: C. Ambitendence occurs when the patient begins to make a movement
but, before completing it starts the opposite movement, for example
bending up and down over a chair without sitting on it. A mannerism
is a normal, purposeful movement that appears to have social
significance but is unusual in appearance. A stereotypy is a repeated
purposeless movement. Waxy flexibility occurs when the patient
allows himself to be placed in an awkward posture which he then
maintains without distress for much longer than most people could
achieve without significant discomfort. (13, pp 249–50)
25
52. An 18-year-old woman is referred for assessment by her GP who reports that she has lost over
20 kg in the past year. She admits that she has been restricting her diet as she believes that she is
very overweight. She self-induces vomiting 2–3 times per day and has begun using laxatives. She
attends the gym daily and twice on Saturday and Sunday. She has not menstruated in the last six
months. Which of the following most accurately represents the biochemical profile of this woman?
53. Which of the following psychiatric disorders has the highest lifetime risk of suicide?
54. A 28-year-old man with schizophrenia reports that his wife has been replaced by a double. Which of
the following syndromes is this man most likely exhibiting?
A. Capgras syndrome
B. Couvade syndrome
C. De Clerambault’s syndrome
D. Fregoli’s syndrome
E. Othello syndrome
55. A right-handed 72-year-old man develops right-left disorientation, finger agnosia, dyscalculia and
dysgraphia. Which cerebral lobe is most likely to have been injured?
56. Which of the following figures best represents the proportion of people with a mental health problem
who receive their treatment in a primary care setting?
A. 5%
B. 10%
C. 20%
D. 30%
E. 50%
26
52. Answer: A. This patient has anorexia nervosa, purging type. Patients with
anorexia nervosa may become hypokalemic secondary to vomiting.
Amenorrhea is associated with negative calcium balance with loss
of skeletal calcium in the range of 4% per year. Many eating disorder
patients have significant bone mineral deficiency, usually osteopenia.
Salivary amylase may be increased when purging is present.
(14, pp 2014–5)
53. Answer: B. For bipolar affective disorder, the lifetime risk of death by suicide
is 15–20%. The lifetime risk for death by suicide for the remaining
options is approximately 10%. (14, pp 1184, 2447)
55. Answer: A. Injury of the posterior parietal lobe can result in Gerstmann’s
syndrome, characterised by right-left disorientation, finger agnosia
(inability to recognise the fingers), dyscalculia (a defect in the ability
to use mathematical symbols) and dysgraphia (disorder of writing
not related to paralysis of the hands). In a right-handed person this
indicates damage to the left parietal lobe. (6, p 169, 9, p 140)
27
57. A 45-year-old woman with schizophrenia requires depot antipsychotic medication. She has been
unsuccessfully treated with an atypical depot, and a typical depot is being considered. She has a
history of third-person auditory hallucinations, persecutory delusions and depressive symptoms
when unwell. Which of the following depots would be the most suitable?
A. Flupenthixol decanoate
B. Fluphenazine decanoate
C. Haloperidol decanoate
D. Pipotiazine palmitate
E. Zuclopenthixol decanoate
58. Which of the following figures best represents the proportion of patients who develop
agranulocytosis on clozapine treatment?
A. 0. 0001%
B. 0.1%
C. 1%
D. 5%
E. 10%
59. Which of the following antipsychotics at average daily dose has the highest monthly cost?
60. A 72-year-old woman is referred for assessment by her GP. He reports that her family have
complained that she is losing her memory. He suspects that she is demonstrating features of
pseudodementia rather than dementia. Which of the following features would most likely suggest
that his diagnosis is correct?
A. Apraxia
B. No diurnal mood variation
C. Normal sleep/wake cycle
D. Prominent memory disturbance
E. Rapid onset
28
58. Answer: C. The number of patients treated with clozapine who develop
agranulocytosis over one year is 0.7%. Approximately 3% of patients
treated with clozapine develop neutropenia. The risk of death
from clozapine-induced agranulocytosis is thought to be less than
1/10 000. (1, pp 77–8)
59. Answer: B. The proprietary costs (not including dispensing fees, or the costs
attached to a private prescription) of the drugs mentioned are as
follows: (1, p 18)
60. Answer: E. Patients with pseudodementia are more likely to present with
rapid onset, distressed affect, fluctuating cognitive deficits, with
no dyspraxia or dysphasia and a past or family history of affective
disorder. Patients with dementia are more likely to present with a
normal sleep/wake cycle, no diurnal variation in symptoms, gradual
onset, prominent memory disturbance and focal features such as
apraxia, agnosia and dysphasia; however, there can be significant
overlap between dementia and pseudodementia, and patients who
develop pseudodementia while depressed have a higher incidence of
organic dementia at follow-up. (7, p 300)
29
61. A 32-year-old woman presents with multiple physical complaints for the past four years. These
include headaches, abdominal pain, dysmenorrhoea, nausea, food intolerance and loss of libido.
Most recently she has been complaining of difficulty swallowing, and has taken extended sick leave
from work. These symptoms have been extensively investigated but no physical cause has been
found. What is the most likely diagnosis in this woman’s case?
A. Conversion disorder
B. Hypochondriasis
C. Major depressive episode
D. Somatisation disorder
E. Somatoform pain disorder
62. A 17-year-old male patient with a two-year history of anorexia nervosa is referred for inpatient
treatment. Which of the following factors in his history will most likely worsen his prognosis?
A. Age of onset
B. Being male
C. Co-morbid depression
D. Duration of illness
E. Premorbid sexual activity
63. Which of the following individuals is most strongly associated with operant conditioning theory?
A. Bandura
B. Pavlov
C. Skinner
D. Thorndike
E. Watson
30
62. Answer: C. Bad prognostic signs include psychiatric co-morbidity, very young or
older age at onset and longer duration of illness. Good prognostic
signs in anorexia nervosa are a short duration of illness with onset in
the early to mid-teens. Male sex by itself does not confer a greater
risk of a poor outcome. Men with some degree of sexual fantasy or
activity before the development of anorexia nervosa have a better
outcome. (14, p 2017)
63. Answer: E. Skinner’s ideas of radical behaviourism made the effects of the
environment a central feature of learning. Operant conditioning is a
form of learning in which behavioural frequency is altered through
the application of positive and negative consequences. Thorndike
preceded Skinner in identifying the relationship between appropriate
behaviour and experiences of success and failure. Pavlov and
Watson are associated with classical conditioning, the association
of a neutral stimulus with an unconditioned stimulus such that the
neutral stimulus comes to bring about a response similar to that
originally elicited by the unconditioned stimulus. Pavlov performed
experiments examining the idea that learning occurs when two
events occur closely together. Watson demonstrated that classical
conditioning can give rise to phobia-like behaviour in a famous
experiment involving an 11-month-old infant, in which he paired a
loud noise with the sight of a white rat, leading the child to fear the
rat and also similar objects, an example of stimulus generalisation.
Bandura advocated social cognitive learning theory, which argues
that the influence of environmental events on the acquisition and the
regulation of behaviour is primarily a function of cognitive processes.
(15, pp 541–7)
31
64. A 32-year-old man presents following a road traffic accident six months previously in which he
was severely injured. He experiences recurrent nightmares of the event and distressing intrusive
memories. He does not drive and avoids the area in which the accident occurred. What form of
learning most likely contributes to the avoidance symptoms exhibited by this patient?
A. Classical conditioning
B. Instrumental learning
C. Modelling
D. Operant conditioning
E. Social-cognitive learning
65. A 20-year-old woman experiences episodes in which she falls down with a sudden loss of muscle
tone after which she sleeps. These episodes tend to be provoked by strong emotion. What
symptom best describes what this woman is experiencing?
A. Catalepsy
B. Cataplexy
C. Catatonia
D. Cathexis
E. Sleep paralysis
66. An inpatient with schizoaffective disorder says the following ‘the man in the van, can, can. He’s a
plan to stand, understand. The band fanned me.’ What form of language disorder is he most likely
exhibiting?
A. Clang association
B. Echopraxia
C. Metonymy
D. Palilalia
E. Verbigeration
67. Which of the following brain structures is least likely included in the limbic system?
A. Amygdala
B. Hippocampal formation
C. Hypothalamus
D. Parahippocampal cingulate gyrus
E. Posterior nucleus of the thalamus
68. Which of the following patients carries the most risk factors for tardive dyskinesia?
A. A male patient with schizophrenia who has never taken antipsychotic medications.
B. A young female patient with bipolar disorder.
C. An elderly female patient with bipolar disorder.
D. An elderly female patient with schizophrenia.
E. An elderly male patient with schizoaffective disorder.
32
67. Answer: E. The limbic system includes the anterior nucleus of the thalamus.
Different authorities list different components, but generally the
limbic system includes the hippocampus, mammillary bodies,
hypothalamus, anterior nucleus of the thalamus, septal nuclei,
fornix, cingulate gyrus, parahippocampal gyrus, amygdala, nucleus
accumbens and the mammillothalamic tract. (7, p 24)
68. Answer: C. Although tardive dyskinesia can occur in patients who have never taken
antipsychotics, it is much more common in those who have taken
antipsychotic drugs for many years. Tardive dyskinesia is more common
among women, the elderly and those with diffuse brain pathology.
A diagnosis of an affective disorder is also a risk factor. (6, p 316)
33
69. For which of the following antipsychotic side effects are anticholinergic drugs least suitable?
A. Akathisia
B. Dystonia
C. Rigidity
D. Tardive dyskinesia
E. Tremor
70. A 35-year-old married woman is referred for assessment by her GP who reports that she is having
sexual difficulties. She is physically well. The patient reports that she and her husband both enjoy an
intimate relationship but have been unable to achieve penetration. Which of the following is the most
likely diagnosis?
71. A 55-year-old patient being treated for depressive disorder has shown very little response to
medication. Which of the following factors would be the strongest predictor of this man’s positive
response to electroconvulsive therapy?
A. Delusions
B. Hypochondriacal symptoms
C. Personality disorder
D. Previous response to electroconvulsive therapy
E. Psychomotor retardation
72. Which of the following is least likely to be an indication for electroconvulsive therapy?
A. Mania
B. Neuroleptic malignant syndrome
C. Obsessive compulsive disorder
D. Parkinson’s disease
E. Schizophrenia
73. Which of the following features is least likely a characteristic of a hypnotic state?
A. Acceptance of distortions
B. Attention is indiscriminately directed
C. Diminished reality testing
D. Increased suggestibility
E. Post-hypnotic amnesia
34
69. Answer: D. The appropriate treatment for tardive dyskinesia is slow withdrawal
or reduction in antipsychotic medication, and consideration of an
alternative. Anticholinergic medications can provoke or exacerbate
tardive dyskinesia. Anticholinergics are an appropriate treatment
option for antipsychotic-induced tremor, dystonia and rigidity.
Anticholinergics may have some efficacy in treating akathisia, which is
part of an extra pyramidal side-effect profile. (2, pp 97–104)
70. Answer: E. Sexual disorders can be categorised into those affecting sexual
desire, affecting sexual arousal, affecting orgasm and causing
pain. Vaginismus is a recurrent or persistent involuntary spasm of
the musculature of the outer third of the vagina that interferes with
sexual intercourse. It is the only one of the female sexual disorders
that prevents completion of sexual intercourse. (4, pp 245–50,
13, pp 488–90)
71. Answer: D. Hypochondriasis and personality disorder are two negative predictors
of response to ECT. Psychomotor retardation and delusions are
positive predictors; however a history of previous response to ECT is
the most robust predictor of all. (7, pp 602–3)
72. Answer: C. An acute response to ECT has been demonstrated in OCD, but
patients soon relapsed and ECT is not recommended for this
disorder. The use of ECT in mania is reserved for patients who are
resistant or intolerant to the usual medication treatments or who have
severe symptoms, for example manic delirium. Neuorleptic malignant
syndrome shows outcomes with ECT which are equivalent to those
obtained pharmacologically. Schizophrenia is the second most
common indication for ECT, although there is a lack of consensus
about its use in this disorder. Motor symptoms in Parkinson’s
disease have been shown to be lessened by ECT, with effects lasting
4–6 weeks. (14, pp 2972–4)
35
74. A 35-year-old man with schizophrenia reports that he has realised that he is being targeted by a
terrorist organisation that is accessing his thoughts through the Internet. When asked what makes
him think this he replies that he does not ‘think’ it, but that it is true. He says that the realisation
came to him ‘out of the blue’. Which of the following options best describes this man’s experience?
A. Autochthonous delusion
B. Delusional memory
C. Delusional mood
D. Delusional percept
E. Nihilistic delusions
75. A patient with schizophrenia shows marked disturbance of speech. While the words he uses
are recognisable they are so disorganised in sentences that they are meaningless. Which of the
following speech disorders best describes this speech disturbance?
A. Cluttering
B. Echolalia
C. Logoclonia
D. Palilalia
E. Paragrammatism
76. Which of the following medications acts primarily by modifying serotonin degradation?
A. Clomipramine
B. Fluoxetine
C. Mirtazapine
D. Phenelzine
E. Venlafaxine
77. A 55-year-old woman with a history of depressive disorder tells you that she is planning to go out for
a meal to celebrate her birthday. She is currently taking phenelzine. Which of the following foods is
safest for this woman to order?
A. Brie cheese
B. Caviar
C. Guacamole
D. Pheasant
E. Fresh sausages
36
37
78. A 22-year-old woman working as an administrative manager is referred by her GP who reports that
she is suicidal. She is accompanied by her parents with whom she has a good relationship. The
patient tells you that she has been feeling depressed for about three months and has been feeling
worse in the last two weeks. She has never harmed herself in the past. Which of this patient’s
characteristics would most likely increase your concern regarding her suicidal risk?
A. Employment status
B. Gender
C. Socioeconomic group
D. Mental illness
E. Social support
A. Add bupropion
B. Add lithium
C. Change to an alternative antidepressant
D. Consider electroconvulsant therapy
E. Increase the dose of escitalopram
80. A 45-year-old woman attending your service has been diagnosed with a depressive disorder.
However, she has shown little response to her antidepressant medication. She has been
on escitalopram 5 mg for six weeks with no response. She complains of loss of libido since
commencing on the medication. She is also taking cimetidine. What is the likely cause of treatment
resistance in this woman’s case?
A. Drug interactions
B. Inadequate dosage of antidepressant
C. Insufficient time for antidepressant to be effective
D. Non-compliance
E. Unaddressed psychiatric comorbidity
81. A 55-year-old man has a long history of schizophrenia. He has a disorganisation syndrome,
characterised by inappropriate affect, poverty of content of speech, tangentiality, derailment and
distractability. This symptom cluster was best described by which of the following researchers?
A. Andreasen
B. Bleuler
C. Crowe
D. Liddle
E. Schneider
38
78. Answer: D. The factors for increased risk of suicide include male gender, lack
of social support, unemployment, lower socioeconomic category,
especially for those in a middle age group, severe mental illness,
especially depression and a history of self-harm. This woman’s
mental illness is her most significant risk factor. (16, p 400)
80. Answer: B. This woman has been taking only 5 mg escitalopram, albeit for
an adequate period of six weeks. The recommended dose for the
treatment of depression is 10 mg daily, increasing to a maximum
of 20 mg. Loss of libido is a potential side effect of escitalopram
indicating that this patient is compliant with her medication.
Escitalopram has a low likelihood of drug interaction, so the
co-prescription of cimetidine is unlikely to be causing a difficulty.
(2, pp 54, 279)
39
82. A 35-year-old woman is referred for assessment by her GP who reports that she has been
complaining of widespread pain for the last five months. She has complained of pain in various areas
on both the right and left sides of her body, above and below the waist. She additionally complains
of fatigue, low mood and sleep disturbance. On examination she exhibits tenderness in many areas,
including the occiput, trapezius, gluteus and knees. Physical investigations are normal. What is the
most likely diagnosis in this woman’s case?
83. A 45-year-old Asian man is referred by his GP for assessment. He recently arrived in the country
and is concerned that his penis is shrinking. On further interview the patient expresses the fear that
his penis will eventually disappear into his body and that he will then die. He has marked anxiety
symptoms. His family report that he has tied a string to his penis in an attempt to stop the perceived
shrinking. Which of the following culture-bound syndromes is this patient most likely experiencing?
A. Amok
B. Brain fag
C. Dhat
D. Koro
E. Latah
40
83. Answer: D. Koro involves an episode of sudden and intense anxiety that the
penis will recede into the body and possibly cause death. It is
associated with a feeling of overwhelming panic. The patient may
attempt to prevent retraction. The syndrome is reported in South
and East Asia. Dhat is a term used in India to refer to severe anxiety
and hypochondriacal concerns associated with the discharge of
semen, whitish discoloration of the urine, and feelings of weakness
and exhaustion. Latah is hypersensitivity to sudden fright, often
with echopraxia, echolalia, command obedience and dissociative
behaviour. The syndrome has been found in many parts of the
world. Brain fag is a term initially used in West Africa to refer to a
condition experienced by student in response to the challenges
of schooling. Symptoms include difficulties in concentrating,
remembering and thinking. Amok is a dissociative episode
characterised by a period of brooding followed by an outburst of
violent aggressive or homicidal behaviour directed at people and
objects. (14, pp 2286–9)
41
84. A patient with epilepsy reports frequent seizures. He initially experiences an intense fear, followed by
a period of absence, amnesia and motor activity. He is confused following the seizure. Which term
best describes this seizure?
A. Petit-mal
B. Complex-partial
C. Partial seizures evolving to secondarily generalised seizures
D. Simple-partial
E. Tonic-clonic
85. A 55-year-old patient who has been drinking alcohol heavily for over ten years attends the service
requesting information about alcohol support services. What stage of the change cycle is this patient
most likely at?
A. Action
B. Contemplation
C. Decision
D. Maintenance
E. Precontemplation
42
Maintenance: The change has been integrated into the person’s life.
43
86. An EEG shows high-voltage slow waves all over the scalp. δ Waves account for <50% of the
rhythm. Sleep spindles and K complexes diminish. What stage of the sleep cycle is the EEG most
likely detecting?
A. Stage I
B. Stage II
C. Stage III
D. Stage IV
E. Wakefulness
87. A 25-year-old man with schizophrenia tells you that thoughts are being taken out of his head by
terrorists. He says that these terrorists are responsible for transmitting his thoughts to others, who
use his words to write songs. What form of thought disorder is this man experiencing?
A. Audible thoughts
B. Thought blocking
C. Thought broadcasting
D. Thought insertion
E. Thought withdrawal
88. A 55-year-old patient with a history of alcohol dependence is admitted to the hospital for alcohol
detoxification. He is noted on assessment to have clouding of consciousness. On physical
examination, he is found to have paralysis of the external rectus muscle, paralysis of conjugate gaze,
ataxia and peripheral polyneuropathy. Ophthalmoscopic examination reveals retinal haemorrhages.
What vitamin deficiency is most likely responsible for this clinical picture?
A. A
B. B1
C. B12
D. E
E. K
89. A 44-year-old man is referred for assessment by his GP who reports that he has been complaining
of memory loss following a road traffic accident. Which of the following findings would most likely
suggest that this man is malingering?
44
Stage II: Low voltages and δ and slower frequencies dominate the
recording. Sleep spindles and K complexes occur.
Stage III: High-voltage slow waves occur all over the scalp. δ Waves
account for <50% of the rhythm. Sleep spindles and K complexes
diminish.
Stage IV: δ Waves dominate the EEG accounting for >50% of the
rhythm. Sleep spindles and K complexes are absent.
89. Answer: C. The diagnosis of malingering is made when the features of the
presentation and history are atypical for an amnestic disorder. The
patient may feign amnesia in preparation for legal proceedings. On
neuropsychological testing a pattern of performance that is worse
than chance or guessing suggests manipulation of the results for
deliberate failure. Densely amnestic patients score at least at a
chance level of 50% accuracy simply by guessing. Malingering
patients may purposely avoid the correct response and score
below chance. On tests of progressive difficulty, the malingering
patient might perform poorly even on the easier tasks and may
not demonstrate the graduated decrements in performance of an
amnestic patient. The remaining options are suggestive of a true
amnesia. (8, p 1105)
45
90. A man who believes in the rights of the disabled finds that there is no parking space beside his
workplace. It is raining heavily and he is late for an important meeting. There are two parking spaces
for the disabled available. He parks in a disabled parking space. Which of the following factors will
most likely decrease his cognitive dissonance?
A. A sign requesting that drivers refrain from using the disabled parking spaces.
B. Deciding that disabled parking spaces discriminate against drivers without a disability.
C. The approach of a traffic warden.
D. The knowledge that there are parking spaces five minutes’ walk away.
E. The knowledge that two of his co-workers are disabled.
91. A 40-year-old patient with a history of alcohol dependence syndrome is currently abstinent from
alcohol. He reports that he feels ready to return to work. Which of the following features of this man’s
plans for employment would least concern you?
A. Autonomous working
B. Mobility
C. Office environment
D. Sale of alcohol
E. Spending much time away from home
92. A patient with bipolar affective disorder on lithium therapy complains of sudden onset of tremor. His
speech is slurred at interview, and his gait is unsteady. His serum lithium is 2.0 mmol/L. Which of the
following is most likely to have contributed to the development of lithium toxicity?
A. Alcohol
B. Caffeine
C. Frusemide
D. Lamotrigine
E. Warfarin
93. A 54-year-old woman is due to have electroconvulsive therapy for the treatment of a severe
depressive episode. She is reluctant as she has heard that it can cause severe memory problems.
What type of memory difficulty is this patient most likely to experience?
46
91. Answer: C. The factors that increase the risk of relapse in the work environment
are job mobility, an absence of the restraining structure of home/
regular workplace, the absence of supervision at work, and the ready
availability of alcohol. (6, p 121)
92. Answer: C. Thiazide diuretics reduce the renal clearance of lithium and levels can
rise within a few days. Excessive caffeine can cause a decrease in
lithium levels. A rapid decrease in caffeine intake can result in lithium
toxicity. Alcohol may result in a slight increase (about 12%) in peak
lithium levels. There is no documented interaction between lithium
and lamotrigine or warfarin. (2, pp 302–6)
93. Answer: D. Most patients will experience some degree of anterograde amnesia,
particularly if the patient is confused after the treatment. Some
patients will have difficult retaining new learning for a few days or
even weeks after a course of ECT. This impairment, while common, is
short-lived. A smaller number of patients will experience a retrograde
amnesia for events leading up to and during a course of ECT.
Some patients will report holes or gaps in their memory extending
back several years. The issue of long-term permanent anterograde
amnesia is unresolved but is complained of only by a minority.
Immediate memory refers to the sensory store in which information is
held for less than a second in the form in which it was perceived. This
level is not usually affected in organic memory disorders. (5, p 49,
8, pp 894–6)
47
94. A patient is admitted to a psychiatric hospital as a voluntary patient. During his admission he tells
his doctor that he no longer wishes to take medication. His doctor tells him that if he does not take
medication his status will be changed to involuntary and he will be given medication by force. What
ethical principle has this doctor most transgressed?
A. Autonomy
B. Beneficence
C. Justice
D. Non-maleficence
E. Confidentiality
95. A 35-year-old male patient with schizophrenia and a history of violence when unwell reports that
he has stopped taking his medication as he was experiencing intolerable side effects. He has
been thinking about killing his psychiatrist, who he blames for the breakdown of his marriage, and
expresses a number of delusional beliefs about his ex-wife’s fidelity. A decision is taken to admit the
man as an involuntary patient. Which of the following best represents the ethical tension that this
scenario presents?
96. A 36-year-old man with an episode of severe depression, unresponsive to antidepressant therapy, is
undergoing a course of ECT. On his first treatment, despite two applications of an electrical stimulus,
the fit is of sub-maximal duration. Which of the following medications is most likely to be causing the
difficulty?
A. Chlorpromazine
B. Clomipramine
C. Diazepam
D. Lithium
E. Phenelzine
97. Which of the following disorders typically has the oldest age at onset?
A. Anorexia nervosa
B. Asperger’s syndrome
C. Gilles de la Tourette’s syndrome
D. Obsessive compulsive disorder
E. Social phobia
48
94. Answer: A. The four main ethical principles of medical treatment are:
95. Answer: A. The four main ethical principles of medical treatment as outlined
above are: Autonomy, Beneficence, Non-maleficence and Justice. In
this scenario, the patient’s right to make choices about his treatment
is in conflict with the doctor’s obligation to do good for the patient.
Given the stated intent to harm a specified person, the psychiatrist is
most likely to treat the patient against his will overruling his autonomy.
This scenario might also be represented by a tension between
autonomy and justice, considering the doctor’s duty to protect third
parties. (18, pp 41–4)
96. Answer: C. Benzodiazepines are likely to raise the seizure threshold, reduce
seizure duration and increase the number of treatments needed.
Lithium used with ECT is reported to result in severe memory
difficulties, neurological difficulties and a reduced antidepressant
effect. Owing to the additional potential for ECT to result in lithium
toxicity, this combination should be used only with very clear
indications. MAOIs are normally contraindicated with anaesthetics
as they can interact with opiates, but there is thought to be no
significant problem with ECT itself. Tricyclics and ECT combined are
thought to produce few difficulties, although the combinations with
anaesthetic agents may raise the risk of cardiac arrhythmias and
hypotension. Antipsychotics lower the seizure threshold and would
be expected to lead to seizures at lower ECT doses. (2, pp 158–60)
97. Answer: D. The mean age of onset for obsessive compulsive disorder is 20
years. Social phobia typically begins to develop after puberty. The
mean age of onset of Gilles de la Tourette’s syndrome is seven years.
Features of Asperger’s syndrome are commonly noted from the age
of three or earlier. The majority of females with anorexia nervosa have
onset within five year of menarche. (6, pp 68, 75, 93, 277, 287)
49
98. A baby and his carer enter an unfamiliar room. A stranger enters and the carer leaves. The baby’s
play decreases and he appears upset. When the carer returns he greets her, stays close to her
and begins to re-explore. According to Ainsworth what type of attachment is this baby most likely
displaying?
A. Autonomous
B. Disorganised
C. Insecure-ambivalent
D. Insecure-avoidant
E. Secure attachment
99. A 45-year-old patient with schizophrenia is unemployed and lives with his mother. Attempts have
been made to engage him in an occupational therapy programme, but he shows poor motivation.
He was last hospitalised about two years ago when he had persecutory delusions, delusions of
reference and second- and third-party auditory hallucinations. More recently he appears to harbour
the same delusions but is not preoccupied by them. He reports that he occasionally hears voices
but they do not bother him. According to DSM-IV, what type of schizophrenia best describes this
man’s symptoms?
A. Catatonic
B. Disorganised
C. Paranoid
D. Residual
E. Undifferentiated
100. A 40-year-old man is referred for assessment. While taking his history you note that he expresses
a lot of anger towards his parents and is preoccupied with events from his childhood. According to
Main, which form of adult attachment is this man most likely to display?
A. Autonomous
B. Dismissing
C. Disorganised
D. Preoccupied
E. Unresolved
50
98. Answer: E. Sixty to seventy per cent of infants show secure attachment.
Insecure-avoidant infants do not appear upset as the mother leaves.
On her return they stay close to her but avoid her if picked up (20%).
Infants with an insecure-ambivalent attachment appear upset when
left, but are ambivalent on her return, reaching out but pushing her
away at the same time (10%). Infants in the disorganised attachment
category lack consistency in their behaviour (10–15%). Autonomous
is a form of adult attachment that has been linked to childhood
secure attachment. (7, p 66)
51
References
1. Taylor D, Paton C, Kerwin R. The South London and Maudsley NHS Foundation Trust and Oxleas NHS
Foundation Trust: prescribing guidelines. 9th ed. London: Informa Healthcare; 2007.
2. Bazire S. Psychotropic Drug Directory 2005: the professionals’ pocket handbook and aide memoire.
Salisbury: Fivepin; 2005.
3. Kumar P, Clark M. Clinical Medicine: a textbook for medical students and doctors. London; Philadelphia,
PA: Baillière Tindall; 1994.
4. American Psychiatric Association. Desk Reference to the Diagnostic Criteria from DSM-IV-TR. Washington,
DC: American Psychiatric Association; 2000.
5. Sims A. Symptoms in the Mind: an introduction to descriptive psychopathology. 2nd ed. London:
Saunders; 1995.
6. Buckley P, Bird J, Harrison G. Examination Notes in Psychiatry: a postgraduate text. Oxford: Butterworth-
Heinemann; 1995.
7. Wright P, Stern J, Phelan M. Core Psychiatry. London: WB Saunders; 2000.
8. Freeman CPL, Zealley AK. Companion to Psychiatric Studies. 6th ed. Johnstone EC, editor. Edinburgh:
Churchill Livingstone; 1998.
9. Bannister R. Brain’s Clinical Neurology. London; New York: Oxford University Press; 1985.
10. Taylor D, Paton C, Kerwin R, editors. The Maudsley Prescribing Guidelines. 9th ed. London: Informa
Healthcare; 2007.
11. World Health Organization. The ICD-10 Classification of Mental and Behavioural Disorders: clinical
descriptions and diagnostic guidelines. Geneva: World Health Organization; 1992.
12. American Psychiatric Association. Diagnostic and Statistical Manual of Mental Disorders: DSM-IV-TR.
Washington, DC: American Psychiatric Association; 2000.
13. Gelder MG, Gath D, Mayou R. Oxford Textbook of Psychiatry. 3rd ed. Oxford; New York: Oxford University
Press; 1996.
14. Sadock BJ, Sadock VA, Kaplan HI. Kaplan & Sadock’s Comprehensive Textbook of Psychiatry. 8th ed.
Philadelphia, PA: Lippincott Williams & Wilkins; 2005.
15. Sadock B, Sadock V. Kaplan & Sadock’s Comprehensive Textbook of Psychiatry. 7th ed. Philadelphia, PA:
Lippincott Williams & Wilkins; 2000.
16. Puri BK, Hall AD. Revision Notes in Psychiatry. 2nd ed. London: Arnold; 2004.
17. Kaplan HI, Sadock BJ. Comprehensive Textbook of Psychiatry. 6 ed. Baltimore, MD: Lippincott Williams &
Wilkins; 1995.
18. Mills S. Clinical Practice and the Law: Dublin: Tottel; 2002.
52
101. Which one of the following psychotropic drugs has the shortest elimination half-life (t½)?
A. Haloperidol
B. Lithium
C. Trazodone
D. Tranylcypromine
E. Chlormethiazole
102. Which one of the following antidepressants has non-linear pharmacokinetic properties?
A. Citalopram
B. Fluoxetine
C. Escitalopram
D. Sertraline
E. Amfebutamone (bupropion)
103. Which of the following benzodiazepines has the shortest elimination half-life (t½)?
A. Lorazepam
B. Alprazolam
C. Diazepam
D. Oxazepam
E. Triazolam
104. Which of the following depot antipsychotic drugs is not prepared as an ester (decanoate)?
A. Flupenthixol
B. Fluspirilene
C. Fluphenazine
D. Clopenthixol
E. Haloperidol
105. Which one of the following antidepressant medications requires the longest washout period before
switching a patient to a monoamine oxidase inhibitor antidepressant?
A. Citalopram
B. Paroxetine
C. Sertraline
D. Fluoxetine
E. Escitalopram
54
101. Answer: E. Chlormethiazole has an elimination half-life (t½) of less than one
hour. The t½ of tranylcypromine is less than two hours. Trazodone
and lithium’s half-lives are 4 and 13 hours respectively. Haloperidol’s
elimination half-life is between 12 and 38 hours. (1, pp 141–7)
103. Answer: E. Triazolam has the shortest elimination half-life (3–5 hours) of the
benzodiazepines listed. The half-lives for diazepam, lorazepam,
alprazolam and oxazepam are 50–150, 10–18, 12–15 and
4–10 hours respectively. (4, pp 364–82)
105. Answer: D. Fluoxetine requires a washout period of 5-6 weeks when switching
a patient from fluoxetine to a monoamine oxidase inhibitor
antidepressant, while sertraline and paroxetine each require two
weeks, and citalopram and escitalopram each require one week.
(6, pp 235–6)
55
106. Which one of the following antidepressant drugs has the least number of active metabolites?
A. Citalopram
B. Fluoxetine
C. Sertraline
D. Amitriptyline
E. Mirtazapine
107. Which one of the following psychotropic medications is least affected by haemodialysis?
A. Venlafaxine
B. Lithium
C. Sodium valproate
D. Alprazolam
E. Sertraline
108. Which of the following types of antidepressant medications is most strongly associated with
anti-adrenergic side effects?
109. Ninety per cent of cases of medication-induced parkinsonism become apparent within which one of
the following time periods?
A. Three days
B. One week
C. Three months
D. Six months
E. One year
110. Which of the following drugs is the least likely to cause drug-induced dystonic reactions in adults?
A. Fluphenazine
B. Metoclopramide
C. Olanzapine
D. Amoxapine
E. Chlorphenamine
111. Which one of the following drug classes has been least associated with an increased risk of falling
in the elderly?
A. Sedatives
B. Antidepressants
C. Benzodiazepines
D. Hypnotics
E. Antihypertensives
56
57
112. Weight gain is most likely to occur in which one of the following antidepressants?
A. Paroxetine
B. Fluoxetine
C. Venlafaxine
D. Sertraline
E. Amfebutamone (bupropion)
113. Which one of the following cranial nerves is least likely to exit from the superior orbital fissure?
A. Oculomotor nerve
B. Abducens nerve
C. Trochlear nerve
D. Ophthalmic (first) branch of the trigeminal nerve
E. Facial nerve
114. Which one of the following cranial nerves is least likely to receive sensory input from the spinal
trigeminal nucleus?
A. V
B. VII
C. VIII
D. IX
E. X
115. Which one of the following tracts is least likely to be involved in the extrapyramidal pathway?
A. Tectospinal
B. Corticospinal
C. Reticulospinal
D. Vestibulospinal
E. Rubrospinal
116. Which of the following clinical features best indicates a dominant (usually left)-sided parietal lobe
lesion?
A. Ocular apraxia
B. Tactile agnosia
C. Anosognosia
D. Optic ataxia
E. Agraphia
117. Which one of the following genetic conditions is least likely to result from autosomal abnormalities?
A. Edwards’ syndrome
B. Down’s syndrome
C. Turner syndrome
D. Patau’s syndrome
E. Cri-du-chat syndrome
58
112. Answer: A. Paroxetine has been consistently reported to be the SSRI most
associated with increased weight gain. Venlafaxine has been
generally reported to have little or no effect on weight gain and
bupropion treatment has been associated with weight loss.
(6, pp 235–6, 12, pp 15–25)
113. Answer: E. The facial nerve exits from the internal auditory meatus. The
oculomotor, abducens, trochlear and trigeminal (ophthalmic branch)
nerves all exit the skull from the superior orbital fissure. (13, p 22)
115. Answer: B. The corticospinal tract provides excitatory input to anterior horn
cells of the spinal cord. In contrast the extrapyramidal pathway tract
system (tectospinal, reticulospinal, vestibulospinal and rubrospinal)
are tracts that pass motor impulses from the cerebral cortex to the
spinal cord and tend to be inhibitory. (1, p 43)
116. Answer: E. Lesions in the dominant (usually left) parietal lobe can result in
Gerstmann’s syndrome, which includes right-left confusion, difficulty
with writing (agraphia) and difficulty with mathematics (acalculia).
It also can lead to tactile agnosia and conduction aphasia while
non-dominant (usually right)-sided parietal lobe lesions can result
in contralateral neglect, tactile agnosia, anosodiaphoria and
anosognosia. Bilateral lesions can lead to Balint’s syndrome, a visual
attention and motor syndrome that includes ocular apraxia, optic
ataxia and simultanagnosia. (14, p 110, 15)
59
118. Which one of the following disorders is least likely inherited in an autosomal recessive manner?
A. Friedrich’s ataxia
B. Wilson’s disease
C. Galactosaemia
D. Neurofibromatosis
E. Niemann–Pick disease
120. Which one of the following statements relating to premenstrual syndrome (PMS) is least correct?
121. Which of the following statements relating to disorders of pregnancy is least correct?
A. Couvade syndrome has been described in the male partner of the pregnant female.
B. In the general population, 10% of women become depressed in the first few months after
delivery of their baby.
C. Clomipramine can be especially helpful in the treatment of post-natal depression with
associated panic disorder.
D. The progesterone deficiency hypothesis is associated with both ‘baby blues’ and post-natal
depression.
E. Puerperal psychosis commonly starts within 48 hours of delivery.
A. Twenty to forty per cent of seasonal affective disorder sufferers report a positive family history of
affective disorder.
B. Ten per cent of those who initially suffer a depressive episode subsequently develop mania.
C. Falret described ‘folie circulaire’ in relation to bipolar affective disorder.
D. ‘Bell’s mania’ describes cases of mania where there is rapid onset of confusional symptoms
without obvious evidence of underlying physical illness.
E. Five to ten per cent of bipolar affective disorder is accounted for by unipolar mania.
60
120. Answer: B. All of the statements provided are correct with the exception of B.
Five to ten per cent of women experiencing PMS report significant
or severe symptoms that impact adversely on their lives. (5, p 942,
14, pp 302–4)
121. Answer: E. The statements provided are correct with the exception of E.
Puerperal psychosis usually starts within four to five days post
partum with development of symptoms such as confusion and
affective changes. (5, pp 904, 920–7, 14, pp 297, 302–4, 307–10)
122. Answer: A. All statements provided are correct with the exception of A. More
than 50% of those suffering from seasonal affective disorder report a
positive family history of affective disorder. (5, pp 33, 42, 51, 56, 58)
61
123. In relation to psychological theory, which one of the following statements is least true?
124. Which one of the following statements relating to psychoanalytic thought is least correct?
125. According to findings of the CATIE trial comparing antipsychotic treatments in the treatment of
schizophrenia, which of the following statements is least true?
126. Regarding treatment of unipolar depressive disorder in adults, which one of the following statements
is least true?
A. Approximately two-thirds of patients with depressive disorder are treated successfully with
antidepressant medication alone.
B. Combined treatment with antidepressant medication and cognitive behavioural therapy (CBT)
confers an additional treatment effect of approximately 30% over treatment with each treatment
alone.
C. Relapse rates following treatment discontinuation are higher in those taking medication
compared with those who have received CBT.
D. CBT is just as effective in the treatment of severe depressive disorder as antidepressant
medication.
E. Interpersonal therapy and CBT have been found to be equally effective in the treatment of major
depressive disorder.
62
123. Answer: B. The statements provided are correct with the exception of B. Freud
posited that there were two principles of mental functioning – primary
and secondary processes – and that of these, primary process
thinking occurs in infantile life, dreaming and fantasy. (13, pp 250–5,
14, p 103)
124. Answer: E. The statements provided are correct with the exception of E. Donald
Winnicott described the concept of the ‘false self’ in terms in mother-
baby separation. (13, pp 257–61)
125. Answer: D. The statements provided are correct with the exception of D. There
were no significant differences between the selected antipsychotic
medications evaluated in the CATIE trial in terms of relative risk to
developing tardive dyskinesia. (16)
126. Answer: D. The statements provided are correct with the exception of D. CBT
has been found to be slightly less effective than antidepressant
therapy in the treatment of unipolar major depressive disorder and
is recommended as equally efficacious in the treatment of mild-to-
moderate unipolar depressive disorder. (17)
63
127. A 35-year-old man was admitted with severe depressive symptoms and psychosis. Medications on
admission were venlafaxine slow release 225 mg daily. Olanzapine 10 mg and mirtazapine 15 mg
were added. He became anxious, tremulous and confused. Examination revealed a tachycardia,
excessive sweating, hypertension, pyrexia hyperreflexia, rigidity and clonus in the lower limbs. There
was raised CK-MM. Which is the best fit diagnosis?
A. Malignant catatonia
B. Malignant hyperpyrexia
C. Neuroleptic malignant syndrome
D. Sepsis
E. Serotonin syndrome
128. A 50-year-old woman is referred by her GP with her second episode of depressive illness. She is
otherwise healthy, but she has a history of hyponatraemia on sertraline in the past. In view of her
history, which antidepressant would you consider the safest choice?
A. Duloxetine
B. Escitalopram
C. Fluoxetine
D. Mirtazapine
E. Moclobemide
129. Which of the following characteristics would be least likely to assist in diagnosing DSM IV criteria for
paranoid personality disorder?
130. In considering the current available evidence for risk factors for the condition known as postpartum
psychosis, which of the following is least likely to be associated with its development?
131. A 26-year-old woman with a history of schizophrenia presents with a relapse of psychosis, due to
non-compliance with medication. She has a history of hepatitis C infection and liver function tests
are abnormal with moderate impairment. All other routine blood tests are normal. She is on no
medication. Which of the following antipsychotic medications would be the most suitable to treat her
symptoms?
A. Amisulpride
B. Chlorpromazine
C. Flupenthixol decanoate
D. Quetiapine
E. Risperidone
64
130. Answer: E. Stressful life events have consistently been shown not to be
associated with the development of puerperal psychosis in women
at high risk of same, whereas answers A–D inclusive have all been
found to be associated with it. (20)
65
132. A 63-year-old right-handed man was referred to the local CMHT for assessment. Collateral history
reported that the man, who previously had been a keen artist, now seemed to paint only on the right
side of the canvas. He was no longer able to set the table or put on his clothes properly. His memory
seemed intact. Neuroimaging was requested and revealed a malignant brain tumour. Based on the
information above, which of the following areas of the brain are the most likely sites for the tumour?
133. This drug acts as a potent antagonist at the alpha1 and H1 receptors and to a moderate degree
at the 5HT2A, D2 receptors and the muscarinic cholinergic receptors. It has a low affinity for the
nigrostriatal dopaminergic neurons. It does not elevate serum prolactin. Which of the following drugs
most closely matches the above profile?
A. Amisulpride
B. Aripiprazole
C. Clozapine
D. Quetiapine
E. Risperidone
134. On considering the distribution of dementias among those under the age of 65 years compared
with those over that age, which dementia in particular occurs more commonly as an early onset
dementia than a late onset one?
A. Alzheimer’s dementia
B. Dementia with Lewy bodies
C. Fronto-temporal dementia
D. Mixed dementia
E. Vascular dementia
135. A 70-year-old woman is referred to the CMHT for evaluation of a one-year history of progressive
cognitive decline. The woman’s husband says that her cognition appears to fluctuate; at times her
concentration and attention appear quite good but at other times appear very poor. According to
McKeith, which of the following features if also present would most strongly support the diagnosis of
dementia with Lewy bodies (DLB)?
66
132. Answer: D. In the example above the patient neglects the contralateral half
of space. He has dressing dyspraxia and there is evidence of
visuospatial disorganisation, all typical of non-dominant parietal lobe
lesions. Lesions of the dominant parietal lobe produce deficits in
keeping with Gerstmann’s syndrome. Dominant temporal lobe lesions
may produce a sensory aphasia, while lesions of the orbitofrontal
area may produce symptoms of irritability, impulsivity and antisocial
behaviour. Lesions of the deep midline structures usually present with
an amnestic syndrome and hypersomnia. (21, pp 492–4)
133. Answer: D. Clozapine, though structurally similar to quetiapine, has only a weak
affinity for D2 but binds more strongly to D4 and has a strong affinity
for serotonergic receptors such as 5HT2A and 5HT2c. Amisulpride
is a pure D2 and D3 antagonist. Aripiprazole is a partial agonist at
D2 and 5HT1A receptors. Risperidone has potent 5HT2 antagonistic
action, with a high affinity for D2 receptors. As the dosage increases,
it can cause extrapyramidal side effects and high prolactin levels.
(21, pp 248–50)
135. Answer: D. According to the Consensus criteria for the clinical diagnosis of
probable and possible dementia with Lewy bodies, answers B, C
and E are all features suggestive of the diagnosis, but answer D
is one of the three core features, two of which are required for a
probable diagnosis of DLB. Answer A is incorrect; if present it is more
suggestive of a vascular dementia. (21, p 515)
67
136. A 30-year-old woman was referred to a psychotherapist for treatment of depression. The initial
assessment revealed that a key focus for this lady was grief, as her father had died suddenly six
months earlier. The therapist ascertained that she would benefit from a structured time-limited
therapy, using a collaborative approach. Emphasis would be on the here and now. However, the
client felt that she would be unable to engage in regular, formal ‘homework’. Which of the following
psychotherapies would be the most suitable for this client, considering the above information?
137. In considering the disorder ‘pathological fire-setting’ in ICD-10, which of the following statements is
least true?
138. In considering obsessive compulsive disorder, which of the following statements is least true?
A. A bimodal age of onset with peaks occurs at ages 12–14 years and 20–22 years.
B. Brain imaging shows morphological changes in basal ganglia structure.
C. First-degree relatives have a higher than normal incidence of psychiatric disorders.
D. Predominance of phobic ruminative ideas and absence of compulsions is a
poor prognostic factor.
E. Relapse often follows discontinuation of pharmacological treatment.
139. On considering the results of Kendall’s study (1987) on the epidemiology of puerperal psychosis,
which of the following statements is least correct?
68
136. Answer: C. Interpersonal therapy satisfies all the above criteria for the required
form of therapy. IPT does not involve homework per se as in CBT, but
patients are encouraged to try out new skills or develop their social
network in between sessions. Note that apart from this woman not
wishing to engage in homework, the second closest correct answer
would have been B (CBT). Both therapies A and C involve references
to past experiences, and collaborative psycho therapy E is less
structured and collaborative. (23)
139. Answer: C. The rate of puerperal psychosis identified by Kendal’s study was
1 per 1000 deliveries. Janssen’s study in Sweden (1964) reported a
rate of 4.8 per 1000 admissions to a university hospital (Answer C).
It is thought that a greater availability of hospital beds and a lower
threshold for admission may account for the different rates. All the
other answers are correct. (21, p 637)
69
140. The replication study of the National Comorbidity Survey in the U.S. by Kessler (2005) and the study
by Wittchen and Jacobi (2005), which looked at the frequency and burden of mental disorders in
Europe, produced a number of interesting results. Which one of the following results is least true?
A. Lifetime prevalence for social phobia in the U.S. study was almost twice that reported in the
European study.
B. Median age of onset for specific phobias was 7 years according to the U.S. study.
C. Median age of onset for substance abuse was 20 years according to the U.S. study.
D. Panic disorder was equally prevalent among the sexes across all the age bands in the
European study.
E. The lifetime prevalence of post-traumatic stress disorder was estimated to be 6.8% according
to the U.S. study.
141. On considering the neuroanatomy of the spinal cord, which of the following statements is least
correct?
A. Discriminative touch and proprioception are carried in the ipsilateral posterior white column.
B. Light touch and pressure sensations are carried in the ipsilateral anterior spinothalamic tract.
C. Pain and temperature sensations are carried in the contralateral lateral spinothalamic tracts.
D. The average spinal cord is approximately 45 cm long.
E. Thirty-one pairs of spinal nerves are attached to the spinal cord.
142. Regarding amino acids that have been found to act as central neurotransmitters, which of the
following statements is least correct?
143. A young man with epilepsy is admitted into an epilepsy monitoring unit for continuous EEG and
video monitoring because his seizure frequency has increased. Which of the following is the least
commonly used EEG-activating procedure used to expose EEG abnormalities?
70
140. Answer: D. Panic disorder is 3.4 times more common in women than in men
below the age of 34, but the prevalence becomes approximately
equal after the age of 50 years, according to the study by Wittchen
and Jacobi (2005). However, this study showed that agoraphobia
(without panic), social phobia and specific phobia were more
common in women than men across all age bands. All the other
statements are correct. Of note, a lifetime prevalence for PTSD could
not be calculated in the European study. (21, p 319)
141. Answer: B. The anterior spinothalamic tract conveys light touch and pressure
sense, while the lateral spinothalamic tracts are concerned with
transmission of pain and temperature. After entering the posterior
roots and before entering the anterior and lateral spinothalamic
tracts, the sensory fibres ascend a short distance before crossing
over to ascend in the contralateral tracts. However, the fibres carrying
discriminative touch, proprioception and vibration sensation pass
mainly uncrossed to the cuneate and gracile nuclei in the medulla
oblongata. The other statements are all correct. (1, pp 4–5)
142. Answer: B. The excitatory amino acids that cause depolarisation of neurons
include glutamic acid, aspartic acid, cysteic acid and homocysteic
acid. The inhibitory amino acids include GABA and glycine, and they
cause hyperpolarisation in neurones. NMDA (N-methyl-d-aspartic
acid) receptors are a form of glutamic acid receptors. Memantine is
an NMDA receptor antagonist used in the management of moderate
to severe Alzheimer’s dementia. GABA has been found to be reduced
in the brains of patients with Huntington’s disease and Alzheimer’s
disease. (1, pp 117–8)
71
144. Which of the following statements regarding EEG rhythms is least true?
A. When an adult human is awake but the eyes are closed, the normal resting EEG rhythm is
composed of alpha activity.
B. Delta activity has a frequency of less than 4 hertz per second.
C. During loss of consciousness from a general anaesthetic, theta activity is prominent.
D. High-frequency beta activity is present when an individual is very alert.
E. EEG recordings from newborn infants may be recognised by the relative lack of electrical activity.
145. Regarding genetically inherited disorders, which of the following pairings are least correct?
146. Regarding the macroscopic findings of the following conditions, which of the following pairings is
least correct?
A. Alzheimer’s disease with global atrophy of the brain and widened sulci and ventricular
enlargement.
B. Huntington’s chorea and marked atrophy of the corpus striatum, basal ganglia and the caudate
nucleus.
C. Pick disease and selective asymmetrical atrophy of the frontal and temporal lobes.
D. Wernicke’s encephalopathy and lesions in the mammillary bodies.
E. Wilson’s disease and depigmentation of the substantia nigra, particularly of the zona compacta.
147. A 60-year-old man presents with severe stabbing pains in his legs lasting a few seconds. He
walks with a wide-based and stomping gait. On neurological examination, there is absence of
proprioception and vibration sensation in the lower limbs. Assuming the diagnosis is tabes dorsalis,
which of the following clinical or pathological findings are least likely to be found in this patient?
148. Regarding the epidemiology of somatisation disorder, which of the following statements is least
correct?
A. A history of parents with frequent complaints of poor physical health has been identified.
B. A history of physical and sexual abuse has been found in patients with this condition.
C. Bass and May (2002) claimed that 4% of the general population and 9% of those admitted into
tertiary care have a history of multiple chronic functional somatic symptoms.
D. It has not been found to be associated with a history of unexplained symptoms in childhood.
E. It is twice as common in women.
72
147. Answer: E. These cerebral cortex findings are found more specifically in
Creutzfeldt–Jakob disease. Argyll Robertson pupils is a condition
where the pupils are small and irregular, there is no light reflex, but
the pupils do constrict with accommodation. It is due to a syphilitic
lesion and an interruption of the fibres from the pretectal nucleus
to the Edinger–Westphal nucleus. Tabes dorsalis can also occur in
neurosyphilis; there may be a loss of deep pain sensation that can
lead to neuropathic joints. Frontal lobe personality changes can occur
in General Paralysis of the Insane and the granulomatous lesions
are typical of the syphilitic gummas that can occur in the cerebral
hemispheres or cerebellum. (1, pp 212–4)
73
149. A 30-year-old man presents with his second psychotic episode, the first occurring a year earlier.
Collateral history reveals that he has a history of epilepsy. Which of the following facts would most
support a diagnosis of schizophrenia?
150. Regarding the psychiatric aspects of epilepsy, which of the following statements is least correct?
A. Approximately 20% of patients with ‘intractable epilepsy’ have non-epileptic seizures, though
these may coexist with true epilepsy.
B. Depression and anxiety are the most common interictal psychiatric disorders.
C. Elation is the most common ictal emotion.
D. SSRIs may cause an increase in the level of phenytoin and carbamazepine.
E. The suicide risk is increased fivefold in those with epilepsy.
151. Regarding long-acting depot preparations, which of the following statements is least correct?
152. Which of the following statements is most true with regard to motivational interviewing?
74
150. Answer: C. The most common ictal emotion is actually fear. If not remembered
by the patient, witnesses usually recall that the patient often looked
terrified during the seizure. Depression is also a common ictal
emotion whereas elation is rare. Non-epileptic attack disorders often
coexist in patients with real epilepsy and in fact patients with epilepsy
are one of the groups most likely to have them. It has been estimated
that the lifetime prevalence of depression is about 62% in those with
epilepsy, compared with 17% in the general population. One study
also suggested that the suicide risk is increased by a factor of five.
SSRIs are commonly used in the treatment of depression in epilepsy,
and though the interactions mentioned above are theoretically
possible, they are rarely of clinical significance. (21, pp 528–30)
152. Answer: D. Motivational interviewing assumes that many clients are still
ambivalent about their motivation to change when they consult
for treatment. The technique therefore uses the key principles of
expressing empathy, developing discrepancy, rolling with resistance,
and supporting self-efficacy to enhance intrinsic motivation to
change by exploring and resolving ambivalence. It tries to avoid a
defensive reaction of patients to advice or instruction. The interview
model is described as ‘confrontational in purpose but not in style’.
The technique was pioneered by Miller and is most useful in the
pre-contemplation and contemplation stages of change. Abraham
Maslow developed a hierarchy of needs to explain human motivation
(A). Defence mechanisms are concepts used in psychodynamic
psychotherapy (B). (25, pp 126–7)
75
153. Which of the following statements is most true relating to women performing better than men on
IQ tests?
154. In learning theory, the term contingency best refers to which of the following?
156. In neurons, lithium has been shown to be least likely to modulate which of the following systems?
157. When euthymic volunteers are given tricyclic antidepressant (TCA) medications, which of the
following statements is most true?
76
153. Answer: A. Total IQ scores for men and women show no significant statistical
differences. Males perform typically better on visuospatial tasks,
mathematical tasks, and motor tasks, whereas women traditionally
excel in verbal abilities, fine motor tasks, and perceptual thresholds
(touch, taste, hearing, olfaction). However, verbal test scores for men
have converged over the last decades towards those of women,
and similarly women now match the mathematical abilities of males.
The only persisting gender difference remaining today is the better
performance of men in visuospatial tasks. (25, p 47)
158. Answer: D. Epileptic seizures, confusional states and hallucinations may occur
in benzodiazepine withdrawal, but these are noted in less than
5% of cases. Symptoms of abnormal sensory perception, such as
hyperacusis, paraesthesia, photophobia, hypersensitivity to touch
and pain, occur frequently. Other somatic symptoms often include
muscle stiffness or twitching. Peculiar to benzodiazepine withdrawal
is a sensation of abnormal body sway. (26, pp 164–5)
77
159. Sodium valproate is best associated with which one of the following side effects?
A. Weight gain
B. Anaemia
C. Hirsutism
D. Nephrotoxicity
E. Ataxia
160. Which of the following statements is most true in relation to drug development?
A. Phase IV clinical trials are carried out when a drug is licensed but not yet marketed.
B. Phase I clinical trials are carried out in university hospital units on a small group of very ill
patients.
C. Phase I clinical trials are carried out on healthy non-human primates.
D. Phase II clinical trials are usually placebo controlled and double blind.
E. In phase III, trials of usually up to 10 different doses of the drug are examined against placebos.
161. Which of the following statements is most true regarding suicide rates?
A. Suicide rates among young adults are higher than those in the elderly.
B. Suicide rates in the northern hemisphere are highest in autumn and winter.
C. Suicide rates in the Western world are highest in social classes V and I.
D. Suicide rates increased significantly in Europe during the world wars.
E. Suicide rates among farmers are significantly lower than those in the general population.
162. Which of the following statements is most true with respect to post-mortem brains of patients with
schizophrenia?
78
159. Answer: A. Weight gain is a common side effect of sodium valproate. Other
common side effects include tremor, ankle swelling, hair thinning
and cognitive dysfunction. Less common side effects include
hepatotoxicity and thrombocytopenia. Metabolic abnormalities such
as hyperammonaemia and glycinaemia have been described. A
severe but rare side effect is encephalopathy. (26, pp 269–70)
160. Answer: D. Drug development takes place in defined phases, which include the
pre-clinical research and development phase (where experiments
are carried out on cell cultures and animals) and the clinical phases
I–IV (where experiments are carried out in humans). Phase I clinical
trials are carried out on drug company premises or within a contract
research organisation and include a small number of healthy
volunteers to determine safety and correct dose. Phase II studies
examine the drug’s efficacy, and are usually carried out as placebo-
controlled, double-blind trials in the patient population. In Phase III,
typically one or two doses of the drug are examined in thousands of
patients over prolonged periods of time to further establish efficacy
and to screen for uncommon side effects. Pharma companies
can then apply for licence with the relevant government authority.
Phase IV trials are carried out after the drug is marketed, to extend
knowledge about side effects and to possibly extend the target
indications. (26, pp 78–83)
161. Answer: C. Age-wise, suicide rates of both men and women are highest in the
elderly. In terms of social class, social classes V (unskilled workers)
and I (professionals) have the highest rates. Certain professions such
as veterinary surgeons, pharmacists and farmers have suicide rates
twice as high as expected. Significantly more suicides occur in spring
and early summer. During each of the world wars, suicide rates
dropped throughout Europe. Being employed and being married are
thought to have protective effects. (27, pp 408–10)
79
163. Which of the following statements is most true in family studies of schizophrenia?
164. Which of the following statements is most true regarding dementia with Lewy bodies (DLB)?
A. The term ‘inception rate’ refers to the number of people who graduate from acutely ill to
chronically ill during a defined period.
B. Cohort studies are carried out in retrospective manner.
C. Period prevalence describes the number of people who are or have been ill over a defined
interval.
D. The term ‘undeclared cases’ refers to methodological flaws in the publication of a study.
E. An odds ratio of 1 generally indicates statistical significance.
166. Which of the following statements is most true related to temporal lobe damage?
A. Temporal lobe damage results in verbal memory impairment when the right hippocampus is
involved.
B. Temporal lobe damage results in spatial memory impairment when the left hippocampus is
involved.
C. Temporal lobe damage generally does not result in personality changes.
D. Temporal lobe damage often results in affective symptoms when the right medial temporal lobe
is involved.
E. Temporal lobe damage often results in psychotic symptoms when the right medial temporal
lobe is involved.
80
165. Answer: C. ‘Inception rate’ refers to the number of people who are healthy at
the beginning of a defined period but became ill during that period.
Cohort studies are carried out prospectively on a group of people
who are followed for a defined period of time. Period prevalence
refers to the number of people who are or have been ill over a defined
period of time, as opposed to point prevalence, which describes the
number of ill people at a given point in time. The term ‘undeclared
cases’ refers to cases of an illness or condition in the community
which are not (yet) known to medical or other agencies. In statistics,
an odds ratio of 1 indicates the absence of any differences between
two groups. (27, pp 92–3)
81
168. Which of the following statements is most true regarding the brains of severely depressed patients?
169. Which of the following statements about mindfulness-based psychotherapy is most true?
82
168. Answer: B. There are subtle neuropathological changes in patients with severe
mood disorders. Lateral ventricles are enlarged (predominantly in
elderly subjects with late onset depression), hippocampal volume is
decreased (more consistently in unipolar than in bipolar depression).
Basal ganglia volume is decreased (in unipolar subjects), volume of
the grey matter of subgenual prefrontal cortex is decreased (unipolar
and bipolar subjects) and there is increased amygdala volume (bipolar
subjects). Cellular pathology includes decreased glial cell numbers
(anterior cingulated cortex), decreased neuronal size and density
(anterior cingulate and prefrontal cortex) and decreased synaptic
markers (pre-frontal cortex). Hyperintense MRI signals of the white
matter can be found in subjects with major depression and in normal
ageing. The neurotrophic hypothesis of depression is based on the
finding that adult neurogenesis, particularly in the hippocampus, is
down-regulated in the disorder. (27, pp 242–3)
83
171. Which of the following statements related to obsessive compulsive disorder (OCD) is most true?
172. Which of the following statements related to cognitive analytical therapy (CAT) is most true?
A. CAT describes a procedural sequence model that refers to the contents of each therapeutic
session.
B. CAT holds that faulty procedures are responsible for the low response rate to psychotherapeutic
treatments.
C. CAT assumes that unrevised maladaptive procedural sequences can help overcome neurotic
difficulties.
D. CAT postulates that lack of opportunities or neglect in childhood is a main cause for restricted
procedural repertoires of the neurotic individual.
E. CAT holds that the concept of reciprocal roles unilaterally affects the neurotic individual but not
their environment.
84
171. Answer: D. Obsessive compulsive disorder has associations with conditions that
have known effects on brain function. Up to 70% of childhood cases
of Sydenham’s chorea have obsessive compulsive symptoms. These
symptoms are also associated with Group A streptococcal infections
in childhood. While Gilles de la Tourette included obsessional
symptoms in his original description of the disorder that now bears
his name, he did not define OCD for the first time. Brain imaging
studies have revealed no structural abnormalities in OCD, but SPECT
and PET studies show increased activity in the orbitofrontal cortex,
anterior cingualte, caudate nucleus and parts of the thalamus. 5-HT
reuptake inhibitors are an effective treatment of the disorder, but this
does not prove that 5-HT function is abnormal. (27, p 198)
85
174. In experiments on living patients, which of the following statements about clozapine is most true?
A. Clozapine has the highest occupancy of striatal D1 receptors of all clinically effective
antipsychotics.
B. Clozapine at higher doses occupies 70–80% of striatal D2 receptors.
C. Clozapine at low doses only partially occupies cerebral 5-HT2A receptors.
D. Clozapine dissociates less rapidly from D2 receptors than first-generation antipsychotics.
E. Clozapine is a potent agonist to 5-HT1A receptors.
175. Which of the following statements regarding anorexia nervosa is most true?
A. Anorexia nervosa has repeatedly been linked to a polymorphism in the promoter region of the
5HT2A receptor.
B. Anorexia nervosa affects 5–10% of female siblings of patients with an established anorexic
illness.
C. Anorexia nervosa affects significantly more individuals today than in the 1970s.
D. Anorexia nervosa affects only 1 male for every 100 females.
E. Anorexia nervosa has the same concordance rate in monozygotic and dizygotic twin pairs.
86
173. Answer: A. The anticonvulsant lamotrigine has been shown to be effective in the
prophylactic treatment of mood disorders, particularly rapid cycling
bipolar disorder or treatment-resistant depression. In bipolar disorder,
lamotrigine is superior to placebo in the prevention of a depressive
relapse, but inferior to lithium in the prevention of manic episodes.
This has led to its use predominantly in bipolar disorder type II. The
most common side effect is skin rash, which is more likely to occur
in the initial stages of treatment. A gradual increase of a low starting
dose to therapeutic doses (100–200 mg/day) over 1–2 months
is recommended to decrease the risk of side effects. Unlike other
anticonvulsants, lamotrigine has no direct effect on CNS kindling, a
process by which low-level electrical stimulation to certain areas of
the brain can lower the seizure threshold. (6, p 197, 26, p 270)
175. Answer: B. Reported incidence rates of anorexia nervosa (AN) in the UK and the
U.S. range between 0.37 and 4.06 per 100 000 population per year,
and have stayed fairly stable since the 1970s. Of diagnosed patients,
5–10% are males. Family studies suggest a genetic component of
the disorder: five to ten per cent of female siblings of patients with
established AN will also develop the condition, and monozygotic
twins show much higher concordance rates (55%) than dizygotic
twins (5%). Association studies have implicated a polymorphism
in the promoter region of the 5-HT2A receptor, but this was not
replicated in many other investigations. (27, pp 362–3)
87
176. Which of the following statements about the amnesic syndrome is most true?
177. Which of the following is likely to be least effective in the treatment of antipsychotic-induced
akathisia?
A. Clonazepam
B. Cyproheptadine
C. Mirtazapine
D. Procyclidine
E. Propranolol
178. Which of the following is thought to best describe the receptor binding of aripiprazole?
A. Dopamine antagonist
B. Dopamine partial agonist
C. Dopamine serotonin antagonist
D. Glutamate antagonist
E. Serotonin partial agonist
179. Which of the following is the most suitable treatment for psychosis in a person who has Parkinson’s
disease?
A. Aripiprazole
B. Olanzapine
C. Quetiapine
D. Risperidone
E. Sertindole
180. For which of the following receptors does haloperidol have the greatest affinity?
A. Alpha1 adrenergic
B. Dopamine D3
C. Histamine H1
D. Muscarinic cholinergic
E. Serotonin 5HT3
88
179. Answer: C. Quetiapine has a low propensity to cause extrapyramidal side effects.
Other second-generation antipsychotics are more likely to have
adverse effects on motor function. First-generation antipsychotics
should be avoided in Parkinson’s disease. (6, p 426, 30, p 415)
89
181. Which of the following is the most common discontinuation symptom on stopping selective
serotonin reuptake inhibitor medication?
182. Which of the following drugs with sedative properties is not a GABA-A-positive allosteric modulator
(PAM)?
A. Flurazepam
B. Trazodone
C. Zaleplon
D. Zolpidem
E. Zopiclone
183. Which of the following antipsychotic drugs is least likely to cause elevated serum prolactin?
A. Amisulpride
B. Clozapine
C. Olanzapine
D. Risperidone
E. Zuclopenthixol
A. Agomelatine
B. Caffeine
C. D-amphetamine
D. Methylphenidate
E. Modafinil
185. Which of the following most closely represents the mortality rate of those with neuroleptic malignant
syndrome?
A. 0.5%
B. 1%
C. 5%
D. 10%
E. 20%
186. Cigarette smoking most likely induces which of the following cytochrome P450 systems?
A. 1A2
B. 2D6
C. 2C9
D. 2C19
E. 3A4
90
183. Answer: B. Clozapine does not exert a significant effect on serum prolactin levels.
First-generation antipsychotics, risperidone and amisulpride, have a
high propensity to cause elevated serum prolactin. Olanzapine has a
minimal effect on serum prolactin. (6, p 10)
186. Answer: A. Cigarette smoking can induce CYP450 1A2, which metabolises
clozapine and olanzapine. Cigarette smokers may require higher
doses of these drugs. Conversely, smoking cessation can cause
serum levels of drugs to rise. (30, pp 403–4)
91
187. Which of the following is most likely to have equally prevalent in women and men?
A. Agoraphobia
B. Animal phobias
C. Obsessive compulsive disorder (OCD)
D. Panic disorder
E. Social phobia
188. Which of the following best describes the peak incidence of age of onset of OCD?
A. 14–23 years
B. 24–34 years
C. 35–44 years
D. 45–54 years
E. 55–64 years
189. Which of the following is most likely a susceptibility gene for ADHD?
A. ADH2
B. Apolipoprotein E
C. BDNF
D. DRD4
E. Dysbindin-1
190. Which of the following most closely reflects the risk of bipolar affective disorder in a first degree
relative of an affected proband?
A. 0.5–1%
B. 5–10%
C. 15%
D. 20%
E. 50%
A. Carbamazepine
B. Flurazepam
C. Lithium
D. Olanzapine
E. Sodium valproate
92
187. Answer: E. Social phobia is equally prevalent in men and women. All other
phobias and anxiety disorders are more common in women.
(31, p 195)
188. Answer: B. The peak incidence for OCD is 24-34 years of age. (31, p 197)
189. Answer: D. DRD4, DRD5 and DAT1 are thought to be susceptibility genes for
the development of ADHD. ADH2 is a susceptibility gene for alcohol
dependence, BDNF for bipolar affective disorder, apolipoprotein E for
Alzheimer’s disease and Dysbindin-1 for schizophrenia. (31, p 46)
190. Answer: B. Five to ten per cent. This contrasts with 5–10% in population
controls. (31, p 47)
191. Answer: D. Fifty per cent of people with epilepsy demonstrate epileptiform
abnormalities on a first routine EEG. Repeated readings will increase
the proportion of positive findings. This increases to 80% if a sleep
recording is obtained. Approximately a third of those with epilepsy
never show an EEG abnormality in an alert state. A generalised spike
and wave response to photic stimulation correlates strongly with
clinical generalised epilepsy. It is thought that 10–20% of children with
epilepsy have impaired intellectual development, with those who have
poorly controlled epilepsy being at highest risk. In adults, progressive
memory deterioration has been described in up to 50% of those with
poorly controlled TLE. There is a higher prevalence of epilepsy among
those committed to prison than in the general population. There is no
increased prevalence in violent crime, and violence as a consequence
of seizures or post-ictal automatisms is rare. (32, p 350)
93
193. Which of the following models of family therapy focus on hierarchies within families, challenging
boundaries and ‘unbalancing’ family equilibrium?
A. Brief solution-focussed
B. Social constructionist
C. Strategic
D. Structural
E. Systemic
194. A 22-year-old man who has been using cannabis intermittently for five years has been referred by
his GP with depressive symptoms. He says that cannabis induces a sense of well-being and sees
no point in discontinuing it. Which of the following statements represents the most likely negative
outcome of his pattern of drug use?
195. Which of the following is most true with regard to the treatment of opiate dependence with
methadone?
196. A 25-year-old female asylum seeker who survived stoning for adultery has pushed thoughts of her
trauma aside and does not address what happened to her. Which of the following best describes
the defence mechanism she is using?
A. Denial
B. Displacement
C. Projection
D. Repression
E. Sublimation
197. Which of the following in relation to subarachnoid haemorrhage (SAH) is least likely?
94
193. Answer: D. Salvador Minuchin is associated with the structural school of family
therapy. He studied slum families, psychosomatic families and
families with a member with an eating disorder. He developed the
one-way screen. This is an active approach to family therapy in which
dysfunctional alliances are challenged and therapeutic crises are
deliberately incited. According to Minuchin, a family is functional or
dysfunctional depending on its ability to adapt to different crises. Brief,
solution-focused therapy emphasises the competencies of families
and individuals. In strategic family therapy the therapist identifies
solvable problems, sets goals, designs interventions to achieve these
goals and examines the responses. Social constructionist approaches
are based on awareness that the observed reality is a creation and
the therapists’ perceptions are influenced by their own cultures and
beliefs. Systemic family therapy was founded in Milan. (31, pp 557–8)
194. Answer: C. In usual intoxicating doses, cannabis can have both sedative and
stimulating effects. It can lead to a sense of well-being, relaxation,
a loss of temporal awareness, slowing of cognitive processes and
short-term memory impairment. Amotivational syndrome can be
a feature of long-term, heavy daily use. Seizures can be a feature
of stimulant overdose and benzodiazepine or alcohol withdrawal.
Flashbacks, visual illusions and visual hallucinations can occur with
hallucinogens such as MDMA and LSD. (30, pp 984–91)
195. Answer: C. Methadone has a long half-life. Withdrawals reach their peak
after 4–6 days, and if untreated, may take 10–12 days to resolve.
Untreated heroin withdrawals reach their peak in 32–72 hours after
last use and usually subside significantly in 5 days. The opiate partial
agonist buprenorphine causes less respiratory depression and milder
withdrawal symptoms than methadone. Opioid withdrawal is not an
emergency but opioid toxicity is. (6, pp 320–6)
95
A. Alpha 1
B. Alpha 2
C. Alpha 2A
D. Alpha 2B
E. Alpha 2C
199. Regarding cognitive errors in diagnosis, being misled by prototypes is best described by which of
the following?
A. Attribution error
B. Availability error
C. Confirmation bias
D. Diagnosis momentum
E. Representative error
200. Which of the following is least true with regard to the epidemiology of Alzheimer’s disease?
96
200. Answer: C. Alzheimer’s disease is the most common of the dementias occurring
in 60% of dementia cases. At all ages, women outnumber men by 2
or 3: 1 except in the early onset familial forms of the disease, which
have equal prevalence in women and men (autosomal dominant).
Prevalence rises exponentially with age and it affects 11% of those
aged 80–89 years. People with Down’s syndrome have an increased
risk of the disease. (32, p 545)
97
References
1. Puri BK, Tyrer PJ. Sciences Basic to Psychiatry. Edinburgh; New York: Churchill Livingstone; 2000.
2. Preskorn SH. Clinically relevant pharmacology of selective serotonin reuptake inhibitors. An overview with
emphasis on pharmacokinetics and effects on oxidative drug metabolism. Clin Pharmacokinet. 1997;
32(Suppl 1):1–21.
3. Goodnick PJ. Pharmacokinetic optimisation of therapy with newer antidepressants. Clin Pharmacokinet.
1994; 27(4):307–30.
4. Katzung BG. Basic & Clinical Pharmacology. 8th ed. New York: Lange Medical Books/McGraw Hill; 2006.
5. Stein G, Wilkinson G. Seminars in General Adult Psychiatry. London: Royal College of Psychiatrists; 1998.
6. Taylor D, Paton C, Kerwin R. Prescribing Guidelines. 9th ed. London: Informa Healthcare; 2007.
7. Cohen LM, Tessier EG, Germain MJ, Levy NB. Update on psychotropic medication use in renal disease.
Psychosomatics. 2004; 45(1):34–48.
8. Bilia AR, Gallori S, Vincieri FF. St. John's wort and depression: efficacy, safety and tolerability – an update.
Life Sci. 2002; 70(26):3077–96.
9. Liebermann JA, Murray RM. Comprehensive Care of Schizophrenia: a textbook of clinical management.
London: Martin Dunitz; 2001.
10. Schneider D, Ravin PD. Tardive Dystonia, Medscape 2009. Retrieved 30 May 2015.
11. Woolcott JC, Richardson KJ, Wiens MO et al. Meta-analysis of the impact of 9 medication classes on falls
in elderly persons. Arch Intern Med. 2009; 169(21):1952–60.
12. Cassano P, Fava M. Tolerability issues during long-term treatment with antidepressants. Ann Clin Psychiatry.
2004; 16(1):15–25.
13. Lawlor B. Revision Psychiatry. Dublin: MedMedia Ltd; 2001.
14. Puri B, Hall A. Revision Notes in Psychiatry. London: Arnold; New York: Oxford Unviersity Press; 1998.
15. Merck Research Laboratories. The Merck Manual of Diagnosis and Therapy. 18th ed. Beers M, Porter R,
editors. Whitehouse Station, NJ: Merck Research Laboratories; 2006.
16. Swartz MS, Stroup TS, McEvoy JP et al. What CATIE found: results from the schizophrenia trial. Psychiatr
Serv. 2008; 59(5):500–6.
17. Rupke SJ, Blecke D, Renfrow M. Cognitive therapy for depression. Am Fam Physician. 2006; 73(1):83–6.
18. Ahuja N, Cole A. Hyperthermia syndromes in psychiatry. Adv Psychiatr Treat. 2009; 15(3):181–91.
19. American Psychiatric Association. Diagnostic and Statistical Manual of Mental Disorders: DSM-IV-TR.
Washington, DC: American Psychiatric Association; 2000.
20. Jones I, Smith S. Puerperal psychosis: identifying and caring for women at risk. Adv Psychiatr Treat. 2009;
15(6):411–8.
21. Stein G, Wilkinson G. Seminars in General Adult Psychiatry: London: RCPsych Publications; 2007.
22. Jefferies K, Agrawal N. Early-onset dementia. Adv Psychiatr Treat. 2009; 15(5):380–8.
23. Morris J. Interpersonal psychotherapy – a trainee’s ABC? Psychiatr Bull. 2002; 26(1):26–8.
24. World Health Organization. The ICD-10 Classification of Mental and Behavioural Disorders: clinical
descriptions and diagnostic guidelines. Geneva: World Health Organization; 1992.
25. Thambirajah MS. Psychological Basis of Psychiatry. Edinburgh; New York: Churchill Livingstone; 2005.
pp. 126–7.
26. King D, editor. Seminars in Clinical Psychopharmacology. London: Royal College of Psychiatrists, Gaskell;
2004.
27. Gelder M, Cowen P, Harrison P. Shorter Oxford Textbook of Psychiatry. Oxford; New York: Oxford University
Press; 2006.
28. Mace C. Mindfulness in psychotherapy: an introduction. Adv Psychiatr Treat. 2007; 13(2):147–54.
29. Denman C. Cognitive–analytic therapy. Adv Psychiatr Treat. 2001;7(4):243–52.
30. Stahl S. Stahl’s Essential Psychopharmacology. 3rd ed. Cambridge; New York: Cambridge University
Press; 2008.
31. Wright P, Stern J, Phelan M. Core psychiatry. 2nd ed. Philadelphia, PA: Elsevier Press; 2005.
32. David A, Fleminger S, Kopelman M et al., editors. Lishman’s Organic Psychiatry. 4th ed. Chichester, UK:
Wiley-Blackwell; 2009.
33. Gross R. Psychology: the science of mind and behaviour. London: Hodder Arnold; 2005.
34. The Skeptic’s Dictionary; 2009. http://www.skepdic.com. Retrieved on the 9th of April 2009.
98
201. You are giving a lecture to medical students about genetic factors and their role in influencing
susceptibility to various psychiatric disorders. The current evidence most strongly favours dysbindin
( DTNBP1) as a susceptibility gene for which one of the following psychiatric disorders?
A. Anorexia nervosa
B. Bulimia nervosa
C. Emotionally unstable (borderline) personality disorder
D. Generalised anxiety disorder
E. Schizophrenia
202. A 79-year-old woman diagnosed with a depressive episode attends your clinic and is receiving
antidepressant treatment with citalopram. On review today she complains of muscle cramps,
nausea, dizziness and lethargy. She has recently been prescribed ibuprofen by her GP for back pain.
Which of the following clinical states most closely fits the symptoms described by this patient?
A. Delirium
B. Hyperkalaemia
C. Hyponatraemia
D. Neuroleptic malignant syndrome
E. Serotonin syndrome
A. Dosulepin
B. Doxepin
C. Mianserin
D. Mirtazapine
E. Reboxetine
204. Which of the following drugs is most likely to induce cytochrome P4502C?
A. Carbamazepine
B. Cimetidine
C. Fluoxetine
D. Fluvoxamine
E. Phenytoin
100
201. Answer: E. Much work has been done to identify susceptibility genes in
schizophrenia and bipolar disorder. The current evidence implicates
specific genes in both disorders. Evidence supports neuregulin 1
(NGR1), dysbindin (DTNBP1), DISC1, D-amino acid oxidase
activator (DAOA (G72)), D-amino acid oxidase (DAO) and regulator
of G-protein signalling (RGS4) as schizophrenia susceptibility loci.
For bipolar disorder the strongest evidence supports DAOA (G72)
and brain-derived neurotrophic factor (BDNF). Increasing evidence
suggests an overlap in genetic susceptibility across the traditional
classification system that dichotomised psychotic disorders into
schizophrenia or bipolar disorder, most notably with association
findings at DAOA (G72), DISC1, and NGR1. (1, p 572)
202. Answer: C. This woman presents with symptoms that are associated with
hyponatraemia. She has several risk factors for hyponatraemia
including being female, elderly and receiving co-therapy with a
non-steroidal anti-inflammatory drug (NSAID). While she could have
delirium as a consequence of hyponatraemia, this vignette does not
describe symptoms associated with delirium such as confusion.
Symptoms of serotonin syndrome could include restlessness,
diaphoresis, tremor, shivering, myoclonus, confusion or convulsions
leading to death. Symptoms of hypokalaemia could include muscle
weakness, hypotonia, cardiac arrhythmias, cramps and tetany; they
are often due to diuretics. The symptoms of neuroleptic malignant
syndrome include fever, diaphoresis, rigidity, confusion, fluctuating
consciousness, fluctuating blood pressure, tachycardia, elevated
creatinine kinase, leucocytosis and altered liver function tests.
(2, pp 210–1, 235, 103–5)
203. Answer: E. Dosulepin, doxepin and mirtazapine are associated with a high
incidence of sedation; mianserin is moderately sedating, and
reboxetine is associated with a low incidence of sedation. (2, p 250)
101
205. Which of the following psychotropic drugs is most extensively metabolised by the liver?
A. Amisulpride
B. Aripiprazole
C. Gabapentin
D. Lithium
E. Sulpiride
206. Which of the following antidepressants is least likely to cause postural hypotension?
A. Fluoxetine
B. Lofepramine
C. Reboxetine
D. Trazodone
E. Venlafaxine
207. You are asked to assess a 63-year-old man by the medical team. He is an inpatient in a medical
ward and was diagnosed with an acute myocardial infarction on admission 10 days ago. He is
medically stable and ready for discharge from hospital. You find him to be significantly depressed
and to require treatment with antidepressant medication. Given the recent myocardial infarction,
which of the following antidepressants would you consider most safe in managing this man’s
depressive illness?
A. Citalopram
B. Duloxetine
C. Fluoxetine
D. Nortriptyline
E. Sertraline
208. Which of the following psychotropic agents are reported to most likely decrease Prothrombin Time/
International Normalised Ratio?
A. Bupropion
B. Carbamazepine
C. Disulfiram
D. Fluoxetine
E. Fluvoxamine
209. A 7-year-old boy diagnosed with attention-deficit hyperactivity disorder (ADHD) is attending your
clinic for review. He has been taking methylphenidate for six months with good improvement in his
symptoms. His mother reports the emergence of motor tics in the past two weeks and would like
a change in her son’s medication. Which of the following alternative drug treatments is the most
suitable alternative?
A. Atomoxetine
B. Carbamazepine
C. Clonidine
D. Modafinil
E. Risperidone
102
206. Answer: E. Hypotension is not a problem with venlafaxine, but it can cause
increases in blood pressure at higher doses. SSRI antidepressants
such as fluoxetine have minimal effect on blood pressure.
Reboxetine is associated with marginal increases in both systolic
and diastolic blood pressures but may cause a postural decrease
in blood pressure at higher doses. Hypotension can be a problem
with lofepramine but it is better tolerated than the older tricyclic
antidepressants. Trazodone can cause significant decreases in blood
pressure. (2, pp 222–4)
103
210. You are asked to review a 22-year-old woman at your clinic who is undergoing treatment for
schizophrenia. She is taking olanzapine and insists she is stopping the medication immediately as
she has gained 10 kg in weight since commencement of treatment 18 months ago. You persuade
her to consider a trial of another antipsychotic. Which of the following antipsychotic agents might
you prescribe as the best possible alternative given this woman’s concern regarding weight gain?
A. Aripiprazole
B. Chlorpromazine
C. Quetiapine
D. Risperidone
E. Zotepine
211. Regarding central serotinergic neurotransmission, which of the following serotonin receptors is most
likely to operate presynaptically?
A. 5HT1A
B. 5HT2A
C. 5HT2C
D. 5HT1D
E. 5HTY
213. Which of the following is most likely a feature of pure word-blindness, alexia without agraphia?
214. Which of the following is most likely a feature of pure word-deafness, subcortical auditory
dysphasia?
104
210. Answer: A. Although all antipsychotics may cause weight gain, mean weight gain
is highly variable between agents and not all individuals gain weight
while taking them. Aripiprazole has a low relative risk of weight gain.
Chlorpromazine, quetiapine and risperidone carry a moderate relative
risk of weight gain. Zotepine has a moderate or high relative risk of
weight gain. (2, p 110)
211. Answer: D. In addition to the serotonin transporter, the 5HT1D receptor is a key
presynaptic serotonin receptor and another key presynaptic serotonin
receptor is the alpha 2 noradrenergic heteroreceptor. A, B, C and E
are all postsynaptic serotonin receptors. (3, p 172)
214. Answer: C. The defect is restricted to the understanding of spoken speech. The
patient hears words as sounds but fails to recognise these sounds
as words. As a consequence, the patient is unable to repeat words
spoken to him/her and cannot write to dictation. The patient can
speak fluently and write normally. (5, pp 49–50)
105
215. Regarding the benzodiazepine drug diazepam, which of the following statements is least likely to be
true?
216. Following administration of electroconvulsive therapy (ECT), which of the following acute hormonal
changes would most likely be observed?
218. Which of the following antipsychotic medications is least likely to cause prolactin elevation?
A. Amisulpride
B. Chlorpromazine
C. Clozapine
D. Risperidone
E. Sulpiride
219. A 52-year-old woman with a diagnosis of schizophrenia attends your clinic. She is concerned that
she has developed abnormal orofacial movements in the preceding month. She is not distressed
by them but would like them to stop. She has been prescribed amisulpride 600 mg for many years,
and procyclidine 5 mg daily, the indications for which are unclear. You diagnose tardive dyskinesia
and find no evidence of other extrapyramidal side effects. Which of the following steps would you be
most likely to consider first in an attempt to manage her side effects?
106
215. Answer: D. Bioavailability following oral dosing of the drug is almost complete
and peak plasma levels are usually attained after 30–90 minutes. The
rate but not the extent of absorption is reduced by the presence of
food. Intramuscular administration is best avoided as absorption is
erratic and often slower than following oral administration. (6, p 113)
217. Answer: B. A serum peak level is reached 2–3 hours after ingestion. Lithium is
not bound to serum proteins, is not metabolised and is excreted
unchanged almost entirely by the kidney. There is a good correlation
between blood levels and both clinical response and side effects.
(6, p 194)
107
220. Regarding fragile X syndrome, which of the following statements is most true?
A. A fragile site on the X chromosome occurs in this syndrome and has been localised to Xq27.
B. The syndrome affects approximately 0.001% of males.
C. The syndrome does not occur in females.
D. The syndrome is a result of an autosomal recessive disorder.
E. The syndrome is associated with mitral valve prolapse in approximately 20% of cases.
221. Which of the following antidepressant drugs is most likely to prolong the QTc interval on ECG
recording at therapeutic doses?
A. Amitriptyline
B. Fluoxetine
C. Mirtazapine
D. Moclobemide
E. Venlafaxine
222. Which of the following antipsychotic agents is least likely to prolong the QTc interval on ECG
recording at therapeutic doses?
A. Aripiprazole
B. Chlorpromazine
C. Haloperidol
D. Quetiapine
E. Sertindole
223. A 35-year-old man with a diagnosis of schizophrenia attends your clinic. He is unable to sit still
and constantly crosses and uncrosses his legs. He describes being distressed by subjective
restlessness. He is taking quetiapine for treatment of his schizophrenia, which was increased two
weeks previously. You diagnose akathisia as a side effect of antipsychotic treatment. Which of
the following antipsychotics is an alternative to his current treatment that is least likely to cause
akathisia?
A. Clozapine
B. Fluphenazine
C. Haloperidol
D. Olanzapine
E. Risperidone
224. You review a 36-year-old married man with a diagnosis of depressive disorder at your clinic.
His depressive symptoms have fully resolved following drug treatment with venlafaxine. He now
complains of erectile dysfunction. He has read in the patient information leaflet that this may be
a side effect of his medication and would like to change to another antidepressant. Which of the
following antidepressants is least likely to cause erectile dysfunction?
A. Citalopram
B. Fluoxetine
C. Mirtazapine
D. Paroxetine
E. Reboxetine
108
220. Answer: A. It is the second most common form of intellectual disability after
Down’s syndrome and affects almost 0.1% of males. Females
carrying fragile X sites may demonstrate varying degrees of
exophenotypic expression. Mitral valve prolapse occurs in
approximately 80% of cases. (7, p 197)
221. Answer: A. Tricyclic antidepressants can prolong the QTc interval at therapeutic
doses. When used as single agents, fluoxetine and mirtazapine have
no effect on the QTc interval. Moclobemide can prolong the QTc
interval following overdose and this is also possible with venlafaxine.
(2, p 224)
222. Answer: A. Aripiprazole has no effect on the QTc interval. Chlorpromazine and
quetiapine are associated with a moderate effect on QTc. Haloperidol
and sertindole are drugs for which extensive QTc prolongation has
been noted. (2, pp 117–18)
224. Answer: E. Sexual dysfunction can occur as a side effect of all antidepressants
and as part of the clinical picture in depression itself. Individual
susceptibility is variable and rates are variable between agents. The
approximate prevalence of sexual side effects in treatment with
selective serotonin reuptake inhibitors (SSRIs) is 70%. Paroxetine
is associated with more erectile dysfunction than other SSRI
antidepressants. The approximate prevalence of sexual side effects
in treatment with mirtazapine is 25% and is approximately 5–10%
with reboxetine. Venlafaxine is estimated to have an approximate
prevalence of sexual side effects of 70% with erectile dysfunction
being less common. (2, pp 231–2)
109
A. Aripiprazole
B. Chlorpromazine
C. Clozapine
D. Promazine
E. Zotepine
226. Regarding the pharmacokinetics of psychotropic medication, which of the following is most true?
227. In the management of an inpatient presenting with first episode psychosis, which of the following is
most correct?
A. A baseline electrocardiogram (ECG) need only be obtained if the patient has an abnormal
physical examination or there is a family history of cardiovascular disease.
B. Benzodiazepines may be helpful.
C. Dosages of antipsychotic medication may need to be adjusted according to the patient’s weight
and gender.
D. Drug absorption from intramuscular injection is reduced if the patient is agitated.
E. The addition of a second antipsychotic medication may be helpful if the patient’s symptoms
persist.
228. Blockade of which of the following receptors is most likely responsible for the reduced sexual
dysfunction associated with mirtazapine?
A. α1
B. H1
C. 5-HT2c
D. 5-HT3
E. M1
110
225. Answer: A. In comparison with other antipsychotics, aripiprazole has a very low
relative incidence of sedation. All of the other antipsychotics in this
question are associated with high relative incidences of sedation.
(2, p 141)
226. Answer: B. Phase II metabolism is a synthetic reaction that converts the active
drug to an inactive compound, usually through conjugation with
glucuronic acid, rendering it water soluble so that it can be excreted
by the kidneys. Drug half-life is directly proportional to volume of
distribution. As most psychotropic medications are lipid soluble,
their half-lives increase with age as the proportion of adipose tissue
increases, leading to an accumulation of the drug in the body. A
steady-state concentration is reached when equilibrium occurs
between absorption and excretion. The kidneys preferentially
excrete ionised and water-soluble compounds. Lipid soluble drugs
preferentially cross the blood–brain barrier. (8, pp 32–5)
228. Answer: C. Blockade of the 5-HT2c receptor is the mechanism through which
mirtazapine is associated with a reduction in sexual dysfunction.
Blockade of the 5-HT3 receptor has an anti-nausea effect. Blockade
of adrenergic and muscarinic receptors is generally associated
with increased sexual dysfunction. The anti-histaminergic effects of
mirtazapine are responsible for its sedative properties. (8, p 71)
111
229. With respect to electro-convulsive therapy (ECT), which of the following is most true?
230. In a patient who fulfils the diagnostic criteria for treatment-resistant depression and is currently
receiving a single antidepressant, which of the following augmentation strategies is most likely to be
efficacious?
A. High levels of transference interpretation in sessions may be associated with higher levels of
therapeutic alliance.
B. Drawing the patient’s attention to dysfunctional thinking patterns may be helpful if there is a
poor therapeutic alliance.
C. Therapist experience has little effect in the treatment of complicated cases.
D. The therapeutic alliance makes a modest contribution to outcome.
E. The therapist’s evaluation of the therapeutic alliance in sessions is the best predictor of outcome.
232. Which of the following is least likely a function of the limbic system?
A. Arousal
B. Declarative memory
C. Food intake
D. Procedural memory
E. Sexual behaviour
233. Regarding the embryonic development of the central nervous system, which of the following is most
true?
A. Three distinct germ cell layers become apparent during the third week of embryonic
development.
B. The neural tube is completely formed by the end of the second week of embryonic
development.
C. The prosencephalon, mesencephalon and rhombencephalon are identifiable by the fifth week of
embryonic development.
D. The midbrain is derived from the metencephalon.
E. The cerebral hemispheres develop from the diencephalon.
112
230. Answer: C. Although all of the medications listed are useful augmentation
strategies for treatment-resistant depression, the evidence for lithium
is strongest, with response rates of 50–60%. (11, p 2843)
232. Answer: D. Procedural memory, the ability to perform a previously learned skill
(e.g., driving), is a function of the basal ganglia and cerebellum.
Declarative memory, the ability to recall facts, is a function of the
hippocampus (which is part of the limbic system). The limbic system
is also involved in homeostatic behaviour such as arousal in the fight-
or-flight reaction, sexual behaviour and food intake. (13, pp 280–95)
113
234. Which of the following diseases is most correctly paired with its macroscopic neuropathology?
235. A 55-year-old married woman is referred to your clinic by her general practitioner. Her husband tells
you that over the past year she has progressively become less active about the house. Last week
she arrived home without the shopping and told her husband that she argued with the checkout
operator after she had jumped the queue. She has become verbally aggressive and inappropriate at
social events. She scores 28/30 on the Folstein Mini-Mental State Examination, performing poorly
on testing of verbal fluency and abstraction. Which of the following diagnoses best fits this clinical
picture?
A. Alzheimer’s disease
B. Creutzfeldt-Jakob disease (CJD)
C. Huntington’s disease
D. Lewy-body dementia
E. Pick’s disease
236. Regarding the sequelae of head injury, which of the following is most correct?
A. Twenty per cent of patients will develop cognitive impairment following severe head injury.
B. Thirty per cent of patients will suffer personality change following severe head injury.
C. Ten per cent of patients with penetrating injuries will develop epilepsy.
D. Duration of anterograde amnesia is a poor prognostic indicator.
E. Twenty per cent of patients with impaired consciousness lasting one month will return to work.
237. Which one of the following antipsychotic medications is least likely to prolong the cardiac QT
interval?
A. Amisulpride
B. Aripiprazole
C. Olanzapine
D. Quetiapine
E. Risperidone
238. With regard to the epidemiology of suicide, which of the following is most correct?
A. Seventy per cent of people who commit suicide have made previous attempts.
B. Alcohol has been detected in one in six suicides.
C. Suicide rates for people aged 65 years and older have been increasing over previous decades.
D. Suicide risk in depression increases with increasing levels of treatment.
E. The male suicide rate is 50% higher than female suicide rate.
114
236. Answer: E. Only 20% of people who suffer a serious head injury with
impaired consciousness lasting one month will be able to re-enter
employment. Forty per cent of patients with impaired consciousness
lasting one month will die. Three per cent of patients who suffer
severe head injuries will experience cognitive impairment, especially
if there is a long period of post-traumatic amnesia (greater than
48 hours), there is damage to the left fronto-parietal areas of the
brain, and there is a penetrating injury or complications such as
haemorrhage or infection occur. Up to 18% of patients will develop
personality change, which may occur directly as a result of brain
damage or may be an exaggeration of premorbid personality traits.
Thirty per cent of patients with penetrating head injuries will develop
epilepsy. (17, pp 192–5)
237. Answer: B. Aripiprazole has been shown to have no effect on QTc interval.
Amisulpride, olanzapine and risperidone have all been shown to have
a low effect on QTc interval. Quetiapine has a moderate effect on QTc
interval. (2, p 117)
238. Answer: D. The risk of suicide increases with intensity of treatment and a history
of inpatient admission, reflecting the fact that patients with severe
illness receive higher levels of treatment. Alcohol has been detected
in one in three suicides. The suicide rate for the over 65 years of age
has progressively fallen over the past two decades. The male suicide
rate is 2–4 times the female suicide rate. Up to 40% of suicides have
made a previous attempt(s). (18, pp 1033–44)
115
239. Regarding the genetics of schizophrenia, which of the following statements is most true?
241. An overweight 21-year-old male arts student with a family history of diabetes mellitus has been
admitted as a voluntary patient under your care with a two-month history of hearing voices
commenting on his actions. He tells you that he believes the children on his street are breaking into
his house and rearranging his possessions. He is not aggressive and states that he is happy to
remain on the ward. Which of the following statements is most true?
A. Cognitive behavioural therapy is likely to confer significant improvements to his mental state.
B. Cognitive behavioural therapy reduces the likelihood of relapse.
C. If he refuses to take oral medication, he should be detained under the Mental Health Act and
given medication intramuscularly.
D. He is likely to become physically aggressive at some point during his admission.
E. Olanzapine would be the antipsychotic medication of choice.
116
240. Answer: E. While the term ‘agnosia’ was first introduced by Freud, the condition
had been described much earlier. The agnosias are a group of
disorders characterised by impairments in the recognition of
objects. Agraphia is a type of apraxia rather than an agnosia. The
doppelgänger phenomenon describes an autoscopic hallucination of
the body being projected into external space. It is usually experienced
by the patient as being pathological. Recognition is composed of
a two-step process – the perception of an object, followed by the
association of the object with meaning. While many agnosic patients
experience a combination of deficits in the areas of apperception
and association, some have agnosias that are either apperceptive or
associative, e.g., the apperceptive agnosic cannot copy a drawing
whereas the associative agnosic can copy the drawing but cannot
identify it. Lesions of the occipital, parietal and temporal regions of
the brain are responsible for visual object agnosia. (5, pp 58–60)
241. Answer: A. Cognitive behavioural therapy (CBT) has been shown to significantly
reduce psychopathology compared with standard treatment in the
early stages of psychosis and leads to significantly shorter inpatient
admissions. While CBT in this group of patients does not reduce
relapse rates, patients exhibit fewer positive symptoms in subsequent
presentations. Verbal aggression is common in patients admitted with
a first episode of psychosis, physical violence is not. The association
between olanzapine and glycaemic dysregulation would suggest that
an alternative antipsychotic medication should be used in this man
who is at an increased risk of diabetes mellitus. (12, p 286, 19)
117
243. Regarding the epidemiology of affective disorders, which of the following is most correct?
244. Regarding the treatment of substance use disorders, which of the following is most correct?
A. A short course of benzodiazepines may assist the symptomatic treatment of opiate withdrawal.
B. Disulfiram inhibits the alcohol dehydrogenase enzyme.
C. Methadone in tablet and sugar-free preparations is less likely to be injected.
D. Evidence for the efficacy of naltrexone in the treatment of alcohol dependence is limited.
E. Pregnant women should be withdrawn from opiates immediately.
245. Regarding the treatment of catatonia, which of the following is most true?
118
119
246. A 29-year-old married woman with bipolar affective disorder attends her routine psychiatric
outpatient appointment with her husband. She is in her fourth week of pregnancy according to her
general practitioner. She asks your advice about the safety of continuing lithium. Her last episode of
mania was nine months ago. Which of the following is most correct?
247. Regarding the treatment of obsessive compulsive disorder, which of the following is most correct?
A. Cognitive and behavioural strategies are more effective than serotonergic antidepressants.
B. Group therapy is less effective than individual therapy.
C. Ten to fifteen per cent of patients believe their obsessional thoughts are rational.
D. More frequent therapy sessions are associated with improved outcomes.
E. Treatment of any kind is more successful if compulsions are present.
248. Regarding caregiver burden in dementia, which of the following is most correct?
249. With respect to the electroencephalogram (EEG), triphasic waves are least likely to be present in
which of the following?
A. Alzheimer’s disease
B. Anoxic brain damage
C. Creutzfeldt–Jakob disease
D. Hepatic encephalopathy
E. Huntington’s disease
120
247. Answer: E. Treatment is more effective if compulsions are present. Cognitive and
behavioural therapies have similar effectiveness to SSRIs. Likewise,
group and individual therapies have similar success rates. Up to a
third of patients in one study believed that their obsessional thoughts
were rational. Patients with more bizarre obsessional thoughts are
less likely to resist carrying out compulsive acts. The frequency of
psychotherapy sessions has not been shown to influence outcome
although duration of each session and the therapy overall is
associated with better outcomes. (12, pp 198–215)
248. Answer: B. Compared with non-caregivers and caregivers who do not report
strain associated with caregiving, caregivers reporting strain have an
increased mortality rate of approximately 60% higher. This may be due
to the physical effects of chronic stress and self-neglect associated
with caring for a patient with dementia. Females are more likely to
report depressive symptoms. Non-white caregivers use a variety
of coping mechanisms such as religion and have strong beliefs of
family responsibility, hence the reduced level of distress experienced
in the care of these patients. Levels of psychological distress among
caregivers as a group have been shown to be relatively stable over
time. While admitting a relative with dementia to a care facility may
reduce distress in some cases; feelings of guilt and sadness have
been reported in those who had relatives placed in nursing homes.
At least 30% of caregivers report symptoms of depression while
caring for a family member with dementia. (11, p 3841)
121
250. A 3-year-old boy is referred to a child psychiatrist with deficits in attention, motor control and
perception. The psychiatrist suspects the presence of DAMP (deficits in attention, motor control and
perception). Which of the following is the most relevant in developmental history-taking?
251. A patient with schizophrenia presents to his GP with frequent urination, excessive thirst, hunger and
weight loss. He has been taking olanzapine 20 mg for the last year. His GP suspects that he might
have developed diabetes. What is the most likely odds ratio for developing diabetes on olanzapine
compared to controls not taking antipsychotic medications?
A. 1–2
B. 2–3
C. 4–6
D. 10
E. None of the above
252. A 24-year-old Caucasian man with a history of paranoid schizophrenia was found by his parents in a
lethargic state. He has a history of suicide attempts and admitted to ingesting 2000 mg quetiapine.
Which of the following is the least likely to be observed in quetiapine overdose?
A. Heart block
B. Chest pain
C. Hypocalcaemia
D. Myocarditis
E. QT prolongation
253. A 19-year-old girl with bulimia nervosa on fluoxetine medication asks her psychiatrist for
psychotherapy. Of the following psychological therapies available for a person with an eating
disorder, which would be the most likely to be applicable?
A. Art therapy
B. Cognitive behavioural therapy
C. Family dialectical behavioural therapy
D. Music therapy
E. Nidotherapy
254. A 44-year-old man recovering from a road traffic accident complained he found it difficult to find his
way around both familiar and new places. Injury to which part of the brain is most likely to cause this
sort of a symptom?
A. Cerebellar injury
B. Frontal lobe injury
C. Optic chiasm injury
D. Right parietal lobe injury
E. Temporal lobe injury
122
250. Answer: B. The concept of DAMP (deficits in attention, motor control and
perception) has been in clinical use in Scandinavia for about 20 years.
DAMP is diagnosed on the basis of concomitant attention deficit/
hyperactivity disorder and developmental coordination disorder
in children who do not have severe learning disability or cerebral
palsy. In clinically severe forms, it affects about 1.5% of the general
population of school-age children. Boys are overrepresented;
however this may be because girls are underdiagnosed. Maternal
alcohol abuse in pregnancy appears to be associated with a
much increased risk of DAMP in the offspring. Equally, smoking in
pregnancy probably has a separate effect on the odds for developing
DAMP or ADHD in the child. (21)
251. Answer: D. The UK General Practice Research Database compared 451 new
incident cases of diabetes with 2696 controls not on antipsychotic
medication. The odds ratio was 5.8 for the risk of developing
diabetes for patients taking olanzapine compared with controls. (22)
253. Answer: B. Quite a few forms of psychological treatment are available for bulimia.
The most commonly used is cognitive behavioural therapy. Dialectical
behavioural therapy and family therapy have also been widely used.
For anorexia nervosa, apart from psychodynamic therapy and family
therapy, the Maudsley Clinic have their own Maudsley approach. (24)
254. Answer: D. The parietal lobe is located just behind the frontal lobe. Damage
to the right parietal lobe can cause visuospatial deficits, e.g., the
patient may have difficulty finding his or her way around both familiar
and new places. Damage to the left parietal lobe area can result in
disruption in the patient’s ability to understand written or spoken
language. (25)
123
255. The Rancho Los Amigos Scale (RLAS) is useful in assessing the patient in the first weeks or months
following a brain injury as it does not require cooperation from the patient. Which of the following
measures are RLAS levels based on?
A. Blood pressure
B. Electrocardiogram findings
C. Pupillary response
D. Response to external stimulus
E. Skin sensitivity
257. Which of the following is least likely to be a side effect of treatment with methylphenidate
medication?
A. Exacerbation of tics
B. Erythema multiforme
C. Insomnia
D. Palpitations
E. Weight gain
258. Which one of the following genes is most likely to be implicated in attention-deficit hyperactivity
disorder (ADHD)?
124
255. Answer: D. The Rancho Los Amigos Scale (RLAS) is most helpful in assessing
the patient in the first weeks or months following a brain injury
because it does not require cooperation from the patient. The RLAS
is based on observation of the patient’s response to external stimuli.
The RLAS provides a descriptive guideline of the various stages a
brain injury patient will experience as they progress to recovery.
(26, pp 511–12)
258. Answer: A. Meta-analysis of genetic data in IMpACT has so far focused mainly
on established ADHD genes from studies in children. The gene
encoding the dopamine transporter DAT1, a regulator of signalling
through the neurotransmitter dopamine in the brain, has been
studied multiple times. Though results have been inconsistent for
single genetic variants, a combination of genetic variations at two
positions of the gene seems to increase ADHD risk in children.
However, in a recent study of 1440 patients and 1769 controls
in IMpACT, a different combination of variants at the same two
positions was found to increase the risk for the persistent adult form
of ADHD. This shows that age is an important factor to be taken
into account in genetic association studies in ADHD and might
explain some of the discrepancies in earlier studies. Transposons
are sequences of DNA that can transpose into new positions in the
genome of a single cell. (29)
125
259. Which of the following would be the most effective measure to check out the prevalence of
schizophrenia in a given population?
A. Audit
B. Cross-sectional survey
C. Cohort study
D. Case–control study
E. Randomised control study
260. With regard to statistical measures, which of the following best describes ‘number of children’?
A. Binary measure
B. Continuous measure
C. Ratio measure
D. Dichotomous variable
E. Quantitative variable
261. Which of the following is most likely to be a neuronal secretory product of specialised hypothalamic
neuroendocrine transducer cells that convert a neural impulse into a hormone?
A. Fatty acid
B. Neurohormone
C. Peptide
D. Steroid
E. Thyroid hormone
262. There are mainly three levels of epidemiological investigations, namely analytical, descriptive and
experimental. Which one of the following best describes an analytical study?
263. With regards to epidemiology, which of the following statements is most true?
A. Lifetime prevalence is almost always based on subject recall, which can be inaccurate.
B. Lifetime prevalence rate does allow for the fact that incidence rates might have changed.
C. Lifetime prevalence is a standard and not a measure.
D. Lifetime prevalence includes those over the represented full age range.
E. Lifetime prevalence is used to summarise the number of cases of a disorder that exist at any
time during a specified time period.
126
262. Answer: A. Analytical studies are those that explore the basis of variations in
illness rates among different groups in order to identify risk factors
that may contribute to the development of the disorder. Once the
basic rates of illness are established, it is possible to identify groups
in the population with high rates of illness. (31, pp 378–9)
127
264. A polymorphism can be detected using polymerase chain reaction (PCR). Which of the following is
least true about PCR markers?
265. Which one of the following is least likely to be included in the ‘metapsychological concepts’ that
form the foundation of the psychoanalytical theory of mental phenomenon?
A. Dynamic
B. Economic
C. Equilibrium
D. Genetic
E. Topographical
266. Which one of the following is least likely to be a side effect of paroxetine treatment?
A. Agitation
B. Constipation
C. Chronic pain
D. Shortening of left ventricular ejection time
E. Sexual dysfunction
267. Which of the following statements with regards to tricyclic antidepressants is least true?
268. Which one of the following is least likely to be an example of a G-protein receptor?
A. Alpha1
B. Alpha2
C. D1
D. D2
E. GABAA
269. Which of the following terms best describes the process where a section of a DNA molecule unzips
and one of the DNA strands acts as a template for the synthesis of RNA?
A. Adhesion
B. Binary connection
C. Transcription
D. Triangulation
E. Translation
128
264. Answer: E. PCR is a method to selectively amplify small regions of the genome
up to 10 000-fold. This eliminates the need to perform a Southern
blot analysis. It is a technique to amplify DNA resulting in numerous
copies of a particular DNA sequence. The method relies on thermal
cycling, consisting of cycles of repeated heating and cooling of the
reaction for DNA melting and enzymatic replication of the DNA.
Primers (short DNA fragments) containing sequences complementary
to the target region along with a DNA polymerase (after which the
method is named) are key components to enable selective and
repeated amplification. As PCR progresses, the DNA generated is
itself used as a template for replication, setting in motion a chain
reaction in which the DNA template is exponentially amplified. PCR
can be extensively modified to perform a wide array of genetic
manipulations. (31, pp 158–9)
265. Answer: C. The five constructs that are integral to the psychoanalytic conception
of the mind are dynamic, topographical, economic, structural and
genetic. Most of the current psychoanalytic theories and practice are
based on these five concepts. (31, pp 1770–1)
266. Answer: C. Selective serotonin reuptake inhibitors (SSRIs) like paroxetine appear
to be an important peripheral mediator in the pain transmission of
efferent pathways. They have been used in the management of pain
syndromes such as peripheral neuropathy, chronic headaches and
fibromyalgia. The common side effects of paroxetine are dry mouth,
some weight gain, headaches, hyponatreamia and increased risk of
gastrointestinal bleeds. (31, pp 2066–77)
268. Answer: E. All the receptors important to psychiatry, with the exception of the ion
channel glutamate and GABAA receptors, are G-protein receptors.
(31, p 583)
269. Answer: C. Transcription is the synthesis of RNA under the direction of DNA.
RNA synthesis, or transcription, is the process of transcribing DNA
nucleotide sequence information into RNA sequence information.
Both nucleic acid sequences use complementary language, and the
information is simply transcribed, or copied, from one molecule to the
other. DNA sequence is enzymatically copied by RNA polymerase to
produce a complementary nucleotide RNA strand, called messenger
RNA (mRNA), because it carries a genetic message from the DNA to
the protein-synthesising machinery of the cell. (31, pp 37–8)
129
270. Which of the following best describes a brief transitional state between wakefulness and sleep,
characterised by mixed frequency EEG and slow wave movements?
A. Sleep latency
B. Stage 1 sleep
C. Stage 2 sleep
D. Stage 3 sleep
E. Stage 4 sleep
271. In relation to autosomal dominant inheritance, which one of these statements is least likely to be
true?
272. A 42-year-old man with bipolar disorder, treated with lithium carbonate 1000 mg for the past year,
presents to Casualty with lethargy, weakness, fever and tremulousness. He is diagnosed with an
encephalopathic syndrome. This disorder was attributed to another of his prescribed medications.
Which of the following medications is most likely to cause this syndrome with lithium use?
A. Non-steroidal anti-inflammatory
B. Ferrous sulphate
C. Neuroleptic medication
D. Penicillin
E. Vitamin supplement
273. Which of the following is least likely to be true when it comes to describing someone with a dissocial
personality disorder?
274. A 25-year-old man who is a chronic heavy cigarette smoker with a diagnosis of schizophrenia
was recently started on an antipsychotic medication for his illness. Which one of the following
antipsychotic medications is most likely to be affected, given this man’s level of smoking?
A. Risperidone
B. Clozapine
C. Quetiapine
D. Ziprasidone
E. Sertindole
130
270. Answer: C. Stage 2 sleep is the state between wakefulness and sleep. In
humans, each sleep cycle lasts from 90 to 110 minutes on average,
and each stage may have a distinct physiological function. Stage
2 is characterised by sleep spindles ranging from 12 to 16 Hz and
K-complexes. During this stage, muscular activity as measured
by EMG decreases, and conscious awareness of the external
environment disappears. This stage occupies 45–55% of total sleep
in adults. (31, pp 80–1)
271. Answer: A. Autosomal dominant diseases are uncommon. In the case of these
diseases, the dominant gene causes a defect. Often such diseases
cause death before a person can bear offspring. For that reason
they are eliminated by evolutionary pressures. Autosomal dominant
diseases that do not appear until after sexual maturity have a chance
of being spread to offspring. This is the case in Huntington’s disease.
(30, pp 39–40)
274. Answer: B. Clozapine and olanzapine are two antipsychotic medications that
are substrates for cytochrome P4501A2, and cigarette smoking
induces this enzyme system. Clozapine, quetiapine, ziprasidone
and sertindole are substrates for cytochrome P4503A4 enzyme
system. This system is not affected by cigarette smoking.
Risperidone, clozapine and olanzapine are substrates for cytochrome
P4502D6 enzyme system that is not affected by cigarette smoking.
Risperidone’s metabolite is also an active antipsychotic (9-OH-
risperidone). (3, pp 437–40)
131
275. A 50-year-old woman with a diagnosis of bipolar disorder and epilepsy has been well maintained
on sodium valproate. She recently had a relapse of both illnesses after a second medication was
introduced. Which of these medication is mostly likely responsible for her relapse?
A. Carbamazepine
B. Lamotrigine
C. Clonazepam
D. Levetiracetam
E. Diazepam
276. A 30-year-old man with a diagnosis of treatment-resistant schizophrenia has been on high-dose
clozapine medication. To reduce his risk of seizures, an anticonvulsant medication was introduced.
He developed agranulocytosis requiring the intervention of a haematologist. Which of these
anticonvulsant medications is most likely responsible for his agranulocytosis in combination with
clozapine?
A. Sodium valproate
B. Lamotrigine
C. Carbamazepine
D. Clonazepam
E. Levetiracetam
277. A 30-year-old man with a diagnosis of schizoaffective disorder being treated with clozapine
medication was found to have mild neutropenia on routine blood investigation. Which of these
psychotropic medications is most likely to have a beneficial effect in correcting this man’s
neutropenia?
A. Olanzapine
B. Risperidone
C. Lamotrigine
D. Sodium valproate
E. Lithium carbonate
278. A 50-year-old man was recently diagnosed with bipolar affective disorder. He was stabilised on
lithium carbonate. He later developed hypertension requiring antihypertensive medication. Which of
these antihypertensive medications has the least potential for interaction with lithium?
A. Ramipril
B. Frusemide
C. Bendroflumethiazide
D. Losartan
E. Atenolol
132
277. Answer: E. Lithium carbonate has been found to be useful in increasing white
blood cell count in patients who have developed neutropenia with
clozapine. Olanzapine, risperidone, lamotrigine and sodium valproate
may play a role in this patient’s treatment; however there is no
evidence to suggest that they increase white cell count. (36)
133
279. A 60-year-old man with a long history of alcohol dependence syndrome was recently diagnosed with
Wernicke’s encephalopathy. Which of the following clinical features is the most common in patients
with this condition?
A. Liver disease
B. Serious malnutrition
C. Peripheral neuropathy
D. Gait abnormalities (ataxia)
E. Ocular abnormalities (nystagmus, sixth nerve palsy, conjugate gaze palsy)
280. A 35-year-old man with a diagnosis of schizophrenia presented with loss of libido and non-
compliance with his antipsychotic medication, risperidone. This man’s psychiatrist attributed his loss
of libido to risperidone. Which of these options best explains the aetiology of loss of libido due to
risperidone in this patient?
281. A 24-year-old woman who is two weeks postpartum develops a sudden onset of a psychotic
disorder with symptoms of elation, euphoria, rambling speech, flight of ideas, lability of mood,
confusion and overactivity. From recent genetic evidence, which one of the following genes is most
likely related to this psychiatric disorder?
282. In what proportion of deliveries does puerperal psychosis most likely occur?
A. 1 in 10
B. 1 in 100
C. 1 in 200
D. 1 in 1000
E. 1 in 10 000
283. A monozygotic twin sibling of a patient with schizophrenia asks you, his brother’s psychiatrist, what
is his risk of developing schizophrenia. Which one of the following best estimates his lifetime risk of
developing schizophrenia?
A. 46%
B. 48%
C. 13%
D. 17%
E. 9%
134
279. Answer: E. All the options A–E are common in patients with Wernicke’s
encephalopathy; however, the most common are ocular
abnormalities. They are present in about 96% of patients on initial
examination. Ataxia was observed in about 87% of patients who
were testable. Peripheral neuropathy was present in about 82%
of cases and was usually confined to the lower limbs. Serious
malnutrition was evident in about 84% while 66% of patients showed
evidence of liver disease. (37, p 578)
280. Answer: A. The human sexual response has three stages. The first stage is libido
(desire for sex) mediated by mesolimbic dopaminergic D2 receptors
resulting in hyperprolactinaemia (if D2 receptors are blocked) that
affects libido. The second stage is arousal mediated by nitrous oxide
(NO) via cGMP and by acetylcholine receptors. The third stage is
orgasm accompanied by ejaculation. Spinal serotonergic fibres inhibit
orgasm via agonist action on 5HT2A while noradrenergic stimulation
facilitates orgasm. Note that risperidone has antagonistic action on
5HT2A, 5HT7, D2, alpha1, and alpha2 receptors. (3, pp 540–46)
281. Answer: D. The answer is post-partum psychosis. Eighty per cent of these
patients present with prominent affective symptoms with mania
or depression with psychotic symptoms. Neuregulin1, dysbindin,
D amino acids activator (G72), regulator of G protein signalling
(RGS-4) and COMT, a susceptibility gene on chromosome 22q,
have been linked to both schizophrenia and bipolar affective disorder;
however, ongoing molecular genetic studies of puerperal psychosis
have shown interesting findings at the serotonin transporter gene
(Coyle 2000) and linkage evidence pointing to the long arm of
Chromosome 16 (Jones 2007). (38) (39, p 254)
135
284. A patient presented with recurrent viral and fungal infection and tetany. His physical and laboratory
investigations revealed cardiac defects, abnormal facies, thymic hypoplasia, cleft palate,
hypocalcaemia and microdeletion of chromosome 22q. From which one of the following psychiatric
disorders is this patient most likely suffering?
A. Depressive disorder
B. Bipolar affective disorder
C. Obsessive compulsive disorder
D. Anxiety disorder
E. Schizophrenia
285. A 50-year-old female monozygotic twin informed her psychiatric team that her co-twin has a
diagnosis of depressive disorder. Which of the following is least correct regarding this woman’s
genetic aetiology?
286. A 21-year-old woman who has a diagnosis of systemic lupus erythematosus (SLE) presented with
delusions and hallucinations. The liaison psychiatrist made a diagnosis of neuropsychiatric SLE
(NPSLE). Which of the following antipsychotic medications is best recommended for the treatment
of NPSLE?
A. Chlorpromazine
B. Haloperidol
C. Risperidone
D. Olanzapine
E. Sulpiride
287. An infantry soldier was diagnosed with post-traumatic stress disorder (PTSD) following his retirement
12 years after he was involved in a life-threatening combat. He coped with his symptoms until his
retirement. He attributed his ability to cope to ‘group effect’ in active military service. Which one of
these is not 1 of the 11 curative factors, as described by Irvin D. Yalom, that may have helped this
man to cope with his symptoms?
A. Instillation of hope
B. Universality
C. Altruism
D. Cohesiveness
E. Pairing
136
287. Answer: E. Irvin D. Yalom (1968) described 11 curative factors that occur
in a group. These are instillation of hope, universality, group
cohesiveness, altruism, corrective recapitulation of primary family
group, development of socialising techniques, imitative behaviour,
catharsis (expression of pent-up feelings), existential factors,
imparting information (education) and interpersonal learning.
Pairing, dependency and fight/flight are the three basic assumptions
according to W.R. Bion (1961). Bion focused on the unconscious
defences of a group as a whole rather than the problems of individual
group members. (44, pp 603–4)
137
288. An 18-year-old girl has presented numerous times to her local emergency department for treatment
of deliberate self-harm symptoms. Which one of the following psychological interventions has been
shown to have the strongest evidence in treating self-harming behaviour in adolescents?
289. Which one of the following psychological therapies is best avoided in patients with PTSD as it lacks
sufficient evidence to support its use?
290. Interpersonal therapy (IPT) has been shown to be effective in the treatment of major depressive
disorder and bulimia nervosa. IPT is closest associated with which one of the following?
291. A 50-year-old man who has a long-standing history of alcohol dependence syndrome asks his GP to
prescribe disulfiram medication to help him address his alcohol dependence. Which of the following
terms best describes this treatment?
A. Reciprocal inhibition
B. Aversion therapy
C. Compliance therapy
D. Response prevention
E. Biofeedback
138
288. Answer: A. Dialectical behaviour therapy (DBT) is the only empirically supported
treatment for adults with multiple mental health problems at risk
of suicide. In a two-year randomised controlled trial, DBT reduced
suicidal behaviour, inpatient days and anger ratings compared with
treatment as usual. Family therapy, psychodynamic psychotherapy,
interpersonal therapy and multisystem therapy do not have a
sufficient evidence base in self-harming behaviour. (45)
290. Answer: A. IPT is based on the work of Harry Stack Sullivan (1953). IPT has
two main aims: to reduce depressive symptoms and to address
social and interpersonal problems associated with the onset
of the symptoms. It is a structured, individual and time-limited
(8–12 sessions) psychotherapy. CBT is the work of Aaron Beck.
DBT is the work of Marsha Linehan. CAT is the work of Anthony Ryle.
RET is the work of Albert Ellis. (47, pp 853–60)
291. Answer: B. Aversion therapy: in this case the unpleasant negative stimulus, the
effect of the disulfiram as it acts to prevent breakdown of ethanol
in the body, is used to suppress unwanted drinking behaviour.
Reciprocal inhibition: systematic desensitisation is coupled with
a response that is incompatible with the anxiety, e.g., relaxation.
Biofeedback: an individual uses information on an aspect of bodily
function, e.g., blood pressure, to alter the function of that bodily
function, usually in an indirect way such as relaxing. Response
prevention: patient stops himself from performing a particular ritual
despite having a strong urge to perform the ritual. Compliance
therapy: methods used to improve patients compliance to treatment;
psycho-education forms an important aspect of this therapy.
(44, pp 522–23; 593–4)
139
292. In Erickson’s stages of psychosexual development, which of the following statements is most
correct?
A. There is a conflict of trust vs. mistrust between the ages of 0 and 18 months.
B. Autonomy is the main task between the ages of 0 and 18 months.
C. Crisis of integrity vs. despair occurs between 40 and 65 years.
D. Separation-individuation is part of Erickson’s stages of psychosexual development.
E. Identity vs. role confusion occurs between the ages of 6 and 12 years.
293. Which one of the following is not a psychosocial vulnerability factor associated with depression
according to George Brown’s classic study of working class women?
294. A 25-year-old woman with a diagnosis of borderline personality disorder was recently admitted into
a therapeutic community. Which of the following characteristics is most true regarding the treatment
provided in the therapeutic community?
295. According to Jean Piaget, if an equal amount of water is poured from a wide glass into a taller/
thinner glass, a 4-year-old will pick the thinner and taller as containing more water. Which of the
following best describes this stage of cognitive development?
A. Sensorimotor stage
B. Pre-operational stage
C. Concrete operational stage
D. Formal operational stage
E. None of the above
140
293. Answer: D. Brown and Harris described four psychological vulnerability factors
associated with the development of depression as described in the
question. Poverty is not one of these factors; however poverty has
been described as a risk factor for depression. (49, 50)
141
296. In relation to the epidemiology of bipolar affective disorder, which one of the following is least
correct?
A. The prevalence of bipolar disorder as estimated by the epidemiological catchments area (ECA)
study is 0.011.
B. The lifetime risk of bipolar disorder lies between 0.3% and 1.5%.
C. The prevalence in men and women is the same.
D. Female sufferers have proportionately fewer manic episodes.
E. The age of onset is later in bipolar disorder than in major depressive disorder.
297. In relation to the epidemiology of unipolar depression, which of the following is least correct?
A. The lifetime rate varies considerably in different studies; however, the true figure probably lies
between 10% and 20%.
B. Before puberty, prevalence rates are similar in both sexes, but rise after puberty to about twice
as great in women as in men.
C. The mean age of onset is about 27 years.
D. The 12-month prevalence of major depression in the community lies between 10% and 20%.
E. In the UK, the most common psychiatric diagnosis in people who have committed suicide is
mood disorder.
298. In the report of the National Confidential Inquiries into Suicide and Homicide by People with Mental
Illness (NCI 1999), which of the figures listed below best describes the percentage of people who
were in contact with services within a week of their suicide?
A. 9%
B. 19%
C. 29%
D. 39%
E. 49%
299. Which of the following antidepressants has the longest approximate half-life?
A. Citalopram
B. Escitalopram
C. Fluoxetine
D. Paroxetine
E. Sertraline
142
298. Answer: E. According to the National Confidential Inquiries into Suicide and
Homicide by People with Mental Illness, 49% of the patients who
died by suicide had been in contact with services in the previous
week, while 19% had contact in the previous 24 hours. About 25%
of all suicides had contact with mental health services in the year
before their death. Sixteen per cent of this group were psychiatric
inpatients at the time of death. According to the July 2010 publication
of the NCI, the most common methods of suicide were hanging/
strangulation, self-poisoning (overdose) and jumping/multiple injuries.
299. Answer: C. All selective serotonin reuptake inhibitors (SSRIs) have long half-lives
allowing once daily dosing. Approximate elimination half-lives of
citalopram and escitalopram are 33 and 30 hours, respectively. The
values for paroxetine and sertraline are 24 and 26 hours, respectively.
Fluoxetine has an approximate half-life of 48-72 hours; however
norfluoxetine, the active metabolite of fluoxetine, has an elimination
half-life of 4-16 days. (2, pp 180–1)
300. Answer: C. Sodium valproate is highly protein bound, but aspirin is more highly
protein bound and can displace sodium valproate from albumin, thus
leading to sodium valproate toxicity. Other less protein bound drugs
such as warfarin can be displaced by sodium valproate. Many side
effects of valproate are related to the peak plasma level. The ‘chrono’
form produces lower peak levels and may be better tolerated.
(2, pp 144–5)
143
References
1. Craddock N, O’Donovan MC, Owen MJ. The genetics of schizophrenia and bipolar disorder: dissecting
psychosis.
J Med Genet. 2005; 42(3):193–204.
2. Taylor D, Paton C, Kerwin R. The South London and Maudsley NHS Foundation Trust Oxleas NHS
Foundation Trust: prescribing guidelines. 9th ed. London: Informa Healthcare; 2007.
3. Stahl S. Essential Psychopharmacology, Neuroscientific Basis and Practical Applications. 2nd ed.
Cambridge: Cambridge University Press; 2000.
4. Hodges JR. Cognitive Assessment for Clinicians. Oxford; New York: Oxford University Press; 1994. xii,
242 pp.
5. Lishman WA. Organic Psychiatry: the psychological consequences of cerebral disorder. 3rd ed. Oxford;
Malden, MA: Blackwell; 1988.
6. King D, editor. Seminars in Clinical Psychopharmacology. London: Royal College of Psychiatrists, Gaskell;
1999.
7. Puri B, Tyrer P. Sciences Basic to Psychiatry. Edinburgh; New York: Churchill Livingstone; 2000.
8. Anderson IM, Reid IC. Fundamentals of Clinical Psychopharmacology. London; New York: CRC Press; 2004.
9. National Collaborating Centre for Mental Health (UK). Schizophrenia: core interventions in the treatment
and management of schizophrenia in primary and secondary care (update). Leicester: British Psychological
Society; 2009 March.
10. National Collaborating Centre for Nursing and Supportive Care (UK). Violence: the short-term management
of disturbed/violent behaviour in in-patient psychiatric settings and emergency departments. London: Royal
College of Nursing (UK); 2005 February.
11. Sadock BJ, Sadock VA, Kaplan HI. Kaplan & Sadock's Comprehensive Textbook of Psychiatry. 8th ed.
Philadelphia, PA: Lippincott Williams & Wilkins; 2005.
12. Roth A, Fonagy P. What Works for Whom?: a critical review of psychotherapy research. New York: Guilford
Press; 2004.
13. Afifi AK, Bergman RA. Functional Neuroanatomy: text and atlas. New York: Lange Medical Books/
McGraw-Hill; 2005.
14. Puri BK, Hall AD. Revision Notes in Psychiatry. London: Arnold; 2004.
15. Crossman AR, Neary D. Neuroanatomy: an illustrated colour text. 2nd ed. Edinburgh; New York: Churchill
Livingstone; 2000.
16. Hodges JR. Cognitive Assessment for Clinicians. Oxford; New York: Oxford University Press; 2005.
17. Buckley P, Bird J, Harrison G. Examination Notes in Psychiatry: a postgraduate text. Oxford; Boston, MA:
Butterworth-Heinemann; 1998.
18. Gelder MG, López Ibor JJ, Andreasen NC. New Oxford Textbook of Psychiatry. Oxford: Oxford University
Press; 2004.
19. Foley SR, Kelly BD, Clarke M et al. Incidence and clinical correlates of aggression and violence at
presentation in patients with first episode psychosis. Schizophr Res. 2005; 72(2-3):161–8.
20. Rösner S, Hackl-Herrwerth A, Leucht S et al. Opioid antagonists for alcohol dependence. Cochrane
Database Syst Rev. 2010; 8(12):CD001867.
21. Gillberg C. Deficits in attention, motor control, and perception: a brief review. Arch Dis Child. 2003;
88(10):904–10.
22. Koro CE, Fedder DO, Gilbert JL et al. Assessment of independent effect of olanzapine and risperidone on
risk of diabetes among patients with schizophrenia: population based nested case–control study. BMJ.
2002; 325(7358):243.
23. Strachan P, Benoff B. Quetiapine overdose induced acute respiratory distress syndrome. Cest J. 2005;
128(4_MeetingAbstracts):419S.
24. Agras WS, Walsh T, Fairburn CG et al. A multicenter comparison of cognitive-behavioral therapy and
interpersonal psychotherapy for bulimia nervosa. Arch Gen Psychiatry. 2000; 57(5):459–66.
25. Berryhill ME, Olson IR. The right parietal lobe is critical for visual working memory. Neuropsychologia. 2008;
46(7):1767–74.
26. Dobkin BH. The Clinical Science of Neurologic Rehabilitation. New York: Oxford University Press; 2003.
27. Benes F. Neural circuitry models of schizophrenia: is it dopamine, GABA, glutamate, or something else?
Biol Psychiatry. 2009; 65(12):1003–5.
28. Rutter M, Taylor EA. Child and Adolescent Psychiatry. Oxford; Malden, MA: Blackwell; 2002.
144
29. Cormand B, Franke B. The Role of Genetic Factors in Adult ADHD. European College of
Neuropsychopharmacology: Istanbul, Turkey; 2009. pp. 1–4.
30. Wright P, Stern J, Phelan M. Core Psychiatry. 2nd ed. Philadelphia, PA: Elsevier; 2005.
31. Kaplan HI, Sadock BJ. Comprehensive Textbook of Psychiatry. 6th ed. Baltimore, MD: Williams & Wilkins;
1995.
32. Taylor D, Paton C, Kerwin R. Prescribing Guidelines. 9th ed. London: Informa Healthcare; 2007.
33. Smith D, Keane P, Donovan J et al. Lithium encephalopathy. J R Soc Med. 2003; 96(12):590–1.
34. World Health Organization. The ICD-10 Classification of Mental and Behavioural Disorders: clinical
descriptions and diagnostic guidelines. Geneva: World Health Organization; 1992.
35. Kalra PA. Essential Revision Notes for MRCP. Revised ed. Cheshire: PasTest; 2002.
36. Blier P, Slater S, Measham T et al. Lithium and clozapine-induced neutropenia/agranulocytosis. Int Clin
Psychopharmacol. 1998; 13(3):137–40.
37. Lishman WA. Organic Psychiatry: the psychological consequences of cerebral disorder. 3rd ed. Oxford;
Malden, MA: Blackwell; 1978.
38. Jones I, Smith S. Puerperal psychosis: identifying and caring for women at risk. Adv Psychiatr Treat. 2009;
15(6):413–4.
39. Wright P, Stern J, Phelan M. Core Psychiatry. 2nd ed. Philadelphia, PA: Elsevier; 2005.
40. Schneider AS, Szanto PA. Pathology. Philadelphia, PA: Lippincott Williams & Wilkins; 2002.
41. Murphy KC, Owen MJ. Velo-cardio-facial syndrome: a model for understanding the genetics and
pathogenesis of schizophrenia. Br J Psychiatry. 2001; 179(5):397–402.
42. McGuffin P, Rijsdijk F, Andrew M et al. The heritability of bipolar affective disorder and the genetic
relationship to unipolar depression. Arch Gen Psychiatry. 2003; 60(5):497–502.
43. Mak A, Ho RCM, Lau CS. Clinical implications of neuropsychiatric systemic lupus erythematosus. Adv
Psychiatr Treat. 2009; 15(6):451–8.
44. Gelder M, Cowen P, Harrison P. Shorter Oxford Textbook of Psychiatry. Oxford; New York: Oxford University
Press; 2006.
45. Wood A. Self-harm in adolescents. Adv Psychiatr Treat. 2009; 15(6):434–41.
46. Rose SC, Bisson J, Churchill R et al. Psychological debriefing for preventing post traumatic stress disorder
(PTSD). Cochrane Database Syst Rev. 2002;(2):CD000560.
47. Freeman CPL, Zealley AK. Companion to Psychiatric Studies. 6th ed. Johnstone EC, editor. Edinburgh:
Churchill Livingstone; 1998.
48. Cherry K. Erik Erikson’s Stages of Psychosocial Development, the eight stages of psychosocial development
[accessed 14/03/2014]. Available from: http://psychology.about.com/od/psychosocialtheories/a/
psychosocial.htm
49. Bruce ML, Takeuchi DT, Leaf PJ. Poverty and psychiatric status. Longitudinal evidence from the New Haven
Epidemiologic Catchment Area study. Arch Gen Psychiatry. 1991; 48(5):470–4.
50. Patten SB. Are the Brown and Harris ‘vulnerability factors’ risk factors for depression? J Psychiatry
Neurosci. 1991; 16(5):267.
51. Eysenck MW. Simply Psychology. 2nd ed. Hove; New York: Psychology Press; Taylor & Francis; 2002.
52. National Confidential Inquiry into Suicide and Homicide by People with Mental Illness. Manchester: The
University of Manchester, 2010.
53. Korneva EA, Shanin SN, Rybakina EG. The role of interleukin-1 in stress-induced changes in immune
system function. Neurosci Behav Physiol. 2001; 31(4):431–7.
145
301. According to ICD-10, which of the following best describes the reliance on non-living objects as a
stimulus for sexual arousal and sexual gratification?
A. Fetishism
B. Fetishistic transvestism
C. Frotteurism
D. Transsexualism
E. Voyeurism
302. Methadone is a licensed medication for heroin dependence treatment. Which of the following
statements is most likely regarding methadone?
303. Which of the following features best differentiates pseudo-dementia from dementia?
304. Regarding procedural memory, which of the following statements is least likely?
148
303. Answer: C. Labile and shallow affect, faulty attention and concentration,
concealment of disability and a lack of complaints relating to
cognitive loss are all features of dementia. (3, p 339)
149
305. A 24-year-old female recently diagnosed with a moderate depressive episode comes to your
outpatient clinic worried about weight gain. What class of antidepressants is most likely to cause her
weight gain?
306. Which of the following antidepressant medications is most likely to cause significant postural
hypotension?
A. Fluoxetine
B. Fluvoxamine
C. Lofepramine
D. Trazodone
E. Venlafaxine
307. Hyponatraemia is a potentially serious adverse effect of antidepressant medications. Which one of
the following is least likely to be a risk factor for developing hyponatraemia?
A. Renal failure
B. Female sex
C. Increased body weight
D. Old age
E. Warm weather
308. One of the following drugs may decrease carbamazepine plasma concentration, and carbamazepine
may increase or decrease its plasma concentration. Which of the following medications is the best fit
to possess these properties?
A. Alprazolam
B. Clomipramine
C. Clozapine
D. Haloperidol
E. Phenytoin
309. A 21-year-old man with a family history of substance misuse attends his local addiction service
asking the duty psychiatrist about his probability of becoming ‘dependant on drugs’. If he has tried
would he have at least once, which one he the highest probability of becoming dependent on?
A. Alcohol
B. Anxiolytics
C. Cannabis
D. Heroin
E. Tobacco
150
151
310. Which one of the following is least advisable on prescribing depot antipsychotic medication?
312. Regarding Freud dream theory, which one of the following terms best describes the latent content of
the dream?
313. Regarding cognitive behaviour therapy, which of the following statements is least likely?
314. In classical psychoanalysis the patient is typically asked to lie on a couch and to free-associate,
saying whatever comes to mind without any type of censorship. The analyst sits behind the patient,
remaining a relatively shadowy and neutral figure. Which of the following statements best describes
the reasoning for this?
152
312. Answer: C. Manifest content is the actual content of the dream. Latent content
is the masked meaning of the dream. Symbolism is visual expression
of hidden ideas. Condensation is where a single image represents
several unconscious wishes. Secondary revision is the intellectual
understanding of the dream by the dreamer. (4, p 312)
314. Answer: B. In classical psychoanalysis the analyst sits behind the patient,
remaining a relatively shadowy and neutral figure to encourage the
development of transference, because if the analyst becomes too
human or real, then transference cannot develop easily. (7, p 465)
153
315. Regarding autosomal dominant transmission, which of the following is least likely?
316. Which one of the following psychiatric illnesses has the strongest genetic effect?
317. A 28-year-old pregnant woman is referred to your psychiatric outpatient clinic worried about her
alcohol use during pregnancy. Which of the following regarding fetal alcohol syndrome is least likely?
318. Regarding causes of Down’s syndrome, which of the following is most likely?
A. Approximately 95% of cases result from trisomy 21 following non-disjunction during meiosis.
B. Approximately 95% of cases result from trisomy 21 following non-disjunction during mitosis.
C. Approximately 1% results from translocation involving chromosome 14.
D. Approximately 1% results from translocation involving chromosome 21.
E. Approximately 4% of cases are mosaics.
319. Which of the following statements best describes the epidemiology of bulimia nervosa?
320. Which of the following is most likely to be associated with stress-induced changes in the immune
system of rodents?
154
317. Answer: D. Fetal alcohol syndrome affects about one-third of all infants born
to alcoholic women. The syndrome is characterised by growth
retardation of prenatal origin (height, weight), minor anomalies,
including microphthalmia, short palpebral fissure, midface hypoplasia,
a smooth or short philtrum and a thin upper lip. Central nervous
system manifestations include microcephaly, delayed development,
hyperactivity, attention deficits, intellectual deficits and seizures.
(3, p 20)
319. Answer: A. Bulimic symptoms are relatively common in the general population.
Ninety per cent of sufferers are female. Bulimia nervosa tends to have
a later age of onset, typically 18 year of age. In the UK, the incidence
of cases presenting to primary care increased threefold between
1988 and 1993. The incidence seems to be highest in large cities,
intermediate in urbanised areas and lowest in rural areas. (9, p 231)
320. Answer: D. In the passive avoidance and learning task in mice, stress-induced
changes cause decreased interferon production, delayed allograph
rejection, decreased antibody response to challenge and increased
death rate. Stress of different durations and intensities induced
the formation of lymphocyte-activating factors by peritoneal
macrophages and increased IL-1 α levels in mouse blood but led
to different changes in the responses of target thymocytes to the
committing actions of IL-1B that correlated with changes in the level
of the humoral immune response. (10, pp 122–33, 11)
155
322. Which one of the following neuroimaging techniques is most likely to give information about regional
cerebral blood flow, ligand binding and metabolic changes in the brain?
A. CT scan
B. MRI scan
C. PET scan
D. SPECT scan
E. X-ray
323. A 24-year-old male involved in a road traffic accident two years ago presents to your psychiatric
outpatient clinic with a history of changes in his personality, disturbed mood, poor judgement,
impaired motivation, social awareness and childishness. Which part of his brain is most likely
affected?
A. Amygdala
B. Frontal lobe
C. Occipital lobe
D. Parietal lobe
E. Temporal lobe
324. Regarding resting membrane ion permeability, which of the following is most likely to be correct?
325. Which of the following is unlikely to be correct regarding macroscopic changes in Alzheimer’s
disease?
156
321. Answer: D. The Hierarchical Model of Suicide is divided into primary, secondary
and tertiary risk factors. Primary factors are seen as powerful
predictors of suicide and do not require the presence of secondary
or tertiary factors to remain active, for example, mental disorders and
health care. Secondary factors are concerned with adverse aspects
of everyday life, e.g., isolation and loss. They are powerful in the
presence of primary factors but are weak alone. Tertiary factors are
mainly demographic, e.g., male gender, marital status. They have a
very low predictive power in the absence of primary or secondary
factors. (4, p 663)
322. Answer: C. Both PET and SPECT can give information about ligand binding and
regional cerebral blood flow, but PET can also give information about
metabolic changes in the brain. (8, pp 208–9)
324. Answer: A. The comparative permeability to different ions of the resting neuronal
membrane is as follows: potassium ions are relatively permeable,
sodium ions are relatively impermeable, chloride ions are freely
permeable, organic anions are relatively impermeable. (8, p 211)
325. Answer: B. The macroscopic changes in Alzheimer’s disease include global brain
atrophy, ventricular enlargement and sulcal widening. Histological
changes include neuronal loss, shrinking of dendritic branching,
reactive astrocytosis, neurofibrillary tangles and senile plaques.
(8, p 193)
157
326. John has a diagnosis of bipolar affective disorder and he is on haloperidol depot injection every
month. Two years later he develops movements in his mouth diagnosed as tardive dyskinesia.
Which of the following option is least suitable for him?
327. A patient newly diagnosed with schizophrenia has adherence issues with antipsychotic medication.
Which of the following approaches would best support adherence to medication?
A. Avoid informing the patient about the side effects of the medication.
B. Using oral medication rather than depot injection.
C. Using paliperidone instead of chlorpromazine.
D. Using higher doses at the start of treatment in order that patient’s body can become quickly
accustomed to the medication.
E. Providing psycho-education to patient but not to his family.
328. A 61-year-old lorry driver presents with a four-month history for depression. Further assessment
revealed history of ataxia, fatigue, flulike symptoms and rapid cognitive impairment. His pupils are
normal and there is no evidence of rash. What is the most likely diagnosis?
A. Alzheimer’s disease
B. Creutzfeldt–Jakob disease, new variant type
C. Creutzfeldt–Jakob disease, sporadic type
D. Lyme disease
E. Neurosyphilis
329. Andrew has schizophrenia and is currently on an antipsychotic medication. His psychiatrist wishes
to keep the drug plasma concentration at optimum levels. Which of the following is least true in
this case?
330. A 78-year-old man presents to the psychiatric clinic with memory impairment and behavioural
difficulties. You diagnose him with Alzheimer’s disease. His daughter, who works as a nurse, asks
you about the pharmacological options available for treating this condition. Which of the following
options has the best evidence for the treatment of Alzheimer’s disease?
158
327. Answer: C. Using medication that has an easy dosing regimen will enhance
adherence by patients with their medication. Paliperidone has
a longer half-life and is usually administered once a day while
chlorpromazine needs to be dispensed at a higher frequency. Other
factors that can improve adherence are the following: (1) compliance
therapy; (2) psycho-education to the family; (3) medication
supervision; (4) providing clear advice and (5) referral to day-hospitals,
day-centres or community psychiatric nurses. (13, pp 26–7)
330. Answer: E. There are three types of agents used in the treatment of Alzheimer’s
dementia: (1) the preventive approaches, e.g., antioxidant, anti-
inflammatory and hormonal treatments; (2) disease-modifying agents,
e.g., memantine, and there is some evidence for monoamine oxidase
inhibitors and (3) symptomatic treatment, e.g., acetyl cholinesterase
inhibitors such as donepezil, galantamine and rivastigmine.
(13, pp 353–58)
159
331. A young man presents to Accident and Emergency following a road traffic accident with impaired
comprehension, naming, reading and writing difficulties. He also shows semantic irrelevancies in his
speech. Which of the following best describes his condition?
332. A 25-year-old man was involved in a road traffic accident, sustaining a head injury with frontal lobe
damage. Which of the following is least likely to occur in frontal lobe damage?
A. Inability to plan
B. Difficulty with attention
C. Working memory deficits
D. Difficulties with facial recognition
E. Motor deficits
333. A woman presents to your clinic with her son, who has been diagnosed with an autistic spectrum
disorder. She is concerned about attachment difficulties. Which of the followings is least correct
about attachment?
A. Selective attachment develops within the first six months of an infant’s life.
B. Attachment behaviour is associated more with affection than with seeking food.
C. Attachment behaviour displayed by an infant is highly correlated with subsequent development
and behaviour.
D. Secure attachment behaviour is related to child temperament.
E. Insecure attachment behaviour includes the inability to express affection in a comfortable way
with the attachment figure.
334. A married couple were involved in a road traffic accident three months ago. They were referred
to you by their general practitioner for psycho-education related to post-traumatic stress disorder
(PTSD). From your knowledge of the epidemiology of PTSD, which of the followings is most true?
160
332. Answer: D. Frontal lobe damage can lead to motor and cognitive symptoms that
may not occur in every case. Cognitive symptoms include inability
to plan, difficulties in attention and working memory disturbances.
Facial recognition is a function of the parietal lobe and the lack of it is
prosopagnosia. (16, p 98)
333. Answer: A. The infant develops selective attention after six months of age.
(16, pp 128–30)
334. Answer: C. The risk of developing PTSD after traumatic events is 8–13% for men
and 20–30% for women. Lifetime prevalence is estimated at 7.8%
with almost double the chance in females. (17, p 368)
161
335. A young man presented with symptoms of psychosis and was commenced on atypical
antipsychotic medication. His mother is concerned about the association between weight gain and
atypical antipsychotics. Which of the following is least likely a proposed mechanism of weight gain
caused by atypical antipsychotics?
336. A 78-year-old lady was referred by her general practitioner with symptoms of insomnia for the past
two months. Which of the following is most true about the epidemiology of insomnia?
337. A 33-year-old man was referred to the psychiatric services by his GP with memory loss, a movement
disorder and tics. Family history revealed that his father had a similar problem and was diagnosed
with Huntington’s disease. Which of the following is most true about this condition?
338. A 16-year-old boy is brought to the psychiatric outpatients by his mother because of behavioural
difficulties. His mother informs you that his behaviour is repetitive and that he is shy and has a limited
social circle. On examination, you note the boy has large ears, large testicles and there is a murmur
audible on auscultation in the mitral valve area. What is the most likely diagnosis?
A. ADHD
B. Autistic spectrum disorder
C. Lesch–Nyhan syndrome
D. Fragile X syndrome
E. Hunter syndrome
339. A lesion in which of the following areas of the brain would most likely lead to loss of appetite?
A. Amygdala
B. Caudate nucleus
C. Nucleus accumbens
D. Lateral nucleus of hypothalamus
E. Medial nucleus of hypothalamus
162
335. Answer: E. Another proposed mechanism is increased thirst with increased fluid
intake and fluid retention. Factors that increase the risk of weight gain
are female sex, previous pattern of overeating, narcissistic traits and
family or personal history of obesity. (17, p 852)
339. Answer: D. The amygdala, nucleus accumbens and caudate nucleus are part
of the basal ganglia. These parts are associated with motor activity
and emotional regulation. The hypothalamus is the centre for the
autonomic motor system controlling appetite, rage, temperature,
blood pressure and sexual function. Lesions of the upper lateral
nucleus result in destruction of the hunger centre causing loss of
appetite. Ventral-medial area lesions result in destruction of the
satiety centre causing increased appetite. (18, pp 76–8)
163
340. A 42-year-old patient presented to Accident and Emergency with psychotic symptoms. His medical
records show that he has Huntington’s disease. His brain scan is most likely to show which of the
following?
341. A 22-year-old man presented to your psychiatric outpatient clinic with hallucinations and delusions.
He admits to recently using ketamine. Which of the following is the most likely proposed mechanism
of his psychotic presentation?
342. A 22-year-old woman presented with symptoms suggestive of emotionally unstable personality
disorder. You want to assess her personality with a Minnesota Multiphasic Personality Inventory
(MMPI). Which of the following is not a scale of the MMPI?
A. Hypochondriasis
B. Masculinity-femininity
C. Bipolar affective scale
D. Schizophrenia
E. Paranoia
343. A 25-year-old man is admitted to hospital with deliberate self-harm. He is illiterate. Which of the
following tests is best suited to assess his personality?
344. A 32-year-old woman is diagnosed with a mild depressive episode. She is considered for cognitive
behaviour therapy. Which of the following would be least likely to occur in a cognitive behaviour
therapy session?
A. Graded assignment
B. Psycho-education
C. Distraction
D. Active interpretation
E. Thought rehearsal
164
344. Answer: D. CBT was developed by Aaron Beck in the 1960s. The techniques
include cognitive and behaviour components. Behavioural
components are activity scheduling, graded assignment, exposure
and response prevention, distraction and relaxation. Cognitive
techniques are psycho-education, identifying and challenging
negative automatic thoughts, and keeping a diary of thoughts.
(18, p 786)
165
345. Which of the following techniques would be most likely to occur in group therapy sessions?
A. Mirroring
B. Examination of parapraxes
C. Flooding
D. Traps
E. Snags
346. In an 80-year-old man with Alzheimer’s disease, which of the following histopathological changes is
least likely to be seen in the temporal lobe?
A. Granulovacuolar degeneration
B. Hirano bodies
C. Pick’s bodies
D. Neurofibrillary tangles
E. Senile plaques
347. In a 45-year-old man with Huntington’s disease, which one of the following macroscopic
neuropathological changes is least likely to be seen?
348. Which of the followings is least likely to be a feature of rapid eye movement (REM) sleep?
349. A 25-year-old male construction worker presented to Accident and Emergency several days after
a left-sided head injury at work with associated speech difficulties including poor articulation and
sparse speech. Which of the following brain areas is most likely affected in this case?
166
348. Answer: D. During REM sleep there is a maximal loss of muscle tone. REM sleep
is associated with increased sympathetic nervous system activation
that will subsequently increase the rate of all physiological functions
including heart rate, blood pressure, respiratory rate, cerebral blood
flow and protein synthesis. (19, p 143)
349. Answer: C. The frontal operculum consists of the Broca’a area. Lesions in
this area can lead to expressive (motor aphasia) Broca’s nonfluent
aphasia. Superior temporal gyrus consists of Wernicke’s area
and lesions in this region can lead to receptive (sensory aphasia)
Wernicke’s fluent aphasia. Lesions in the posterior inferolateral
region can lead to prosopagnosia and impaired object recognition.
Left-sided lesions affecting the medial temporal region can lead to
anterograde amnesia affecting verbal information. Left-sided lesions
in the inferior parietal lobule can lead to conduction aphasia and
tactile agnosia. (19, pp 109–10)
167
350. A 70-year-old woman diagnosed with Gerstmann’s syndrome is least likely to present with which of
the following features?
A. Agraphia
B. Dyscalculia
C. Finger agnosia
D. Right–left disorientation
E. Anosognosia
351. Structural neuroimaging has the least utility in which of the following clinical indications?
352. The mutation that changes the DNA codon but does not affect the produced amino acid is best
described as which of the following?
A. Somatic mutation
B. Germinal mutation
C. Silent mutation
D. Nonsense mutation
E. Chromosomal mutation
353. In a 40-year-old man with Huntington’s disease, the pattern of inheritance of this disease is best
described as which of the following?
354. Which of the following lifetime risks of developing schizophrenia in relatives of a patient with
schizophrenia is least likely?
A. General population: 1
B. Nephew/niece of a patient: 4
C. Dizygotic twin sibling of a patient: 35
D. Monozygotic twin sibling of a patient: 48
E. Parent of a patient: 6
168
351. Answer: E. The structural CT and MRI scans are widely used in neuropsychiatric
research. Their clinical uses include the detection of intracranial
expanding lesions, shifts of intracranial structures, cerebral infarction,
cerebral oedema, atrophy of brain structures and demyelination
changes. On the other hand, applications relating to cerebral blood
flow are best visualised using functional neuroimaging technologies
including PET, SPECT and fMRI scanning. (19, pp 137–8)
352. Answer: C. The silent mutation is a point mutation that changes the DNA codon
but does not affect the amino acid inserted into the protein, while
the nonsense mutation occurs where the sequence is changed to
that of a stop codon, causing production of an incomplete protein.
The mutation during mitotic division in cells that do not give rise
to gametes is called a somatic mutation, while mutations in cell
precursors of gametes are called germinal mutations. Chromosomal
mutations involve a change in the number of chromosomes or
rearrangement of sequences during DNA replication. (20, p 50)
353. Answer: B. In autosomal dominant inheritance the disease gene is dominant; the
phenotype is evident in both the heterozygotes and homozygotes.
The affected/unaffected ratio for offspring is 1:1 and the disorder
does not escape generations. Huntington’s disease is a good
example of this pattern of inheritance. Options A, B and C are
considered the three major patterns of Mendelian inheritance.
(9, p 50)
354. Answer: C. The risk of schizophrenia is greatest when there are close relatives
affected, several relatives affected, and when a monozygotic sibling
is affected. The lifetime risk of schizophrenia in a dizygotic twin sibling
of a patient is approximately 17 not 35. (9, pp 274–75)
169
355. Which of the following questionnaire response types closest reflects where the subject always tends
either to agree or disagree with questions?
A. Social acceptability
B. Response set
C. Bias towards centre
D. Halo effect
E. Hawthorne effect
356. Which of the following circumstances is least likely to increase the rate of absorption of drugs
administered intramuscularly?
A. Lipid-soluble drugs
B. Drugs with low molecular mass
C. Physical exercise
D. Cardiac failure
E. Emotional excitement
357. Which of the following is more suggestive of unipolar depression than bipolar depression?
358. Which of the following is the most common reaction involved in the hepatic phase I metabolism of
psychotropic drugs?
A. Hydrolysis
B. Oxidation
C. Conjugation
D. Reduction
E. None of the above
359. In a 29-year-old male with schizophrenia on typical antipsychotic medication, which of the following
side effects is most likely secondary to antidopaminergic action on the tuberoinfundibular pathway?
A. Parkinsonism
B. Pyrexia
C. Dry mouth
D. Low sperm count
E. Urinary retention
170
355. Answer: B. Social acceptability is when the subject chooses the acceptable
answer rather than the true one. Bias towards the centre occurs
when the subject tends to choose the middle response and shun
extremes. The Halo effect occurs when the answers are chosen to
fit with previously chosen answers and the Hawthorne effect occurs
when the subject improves an aspect of their behaviour simply in
response to the fact that they are in an experiment. (21, p 5)
171
A. Acute akathesia with antipsychotic medication has been associated with polymorphism of
DRD3 and DRD2.
B. Polymorphism of DRD3 and DRD2 has been associated with antipsychotic-induced
hyperprolactinaemia.
C. Resistance to antipsychotic medication is well documented in patients with CY2D6.
D. Weight gain with antipsychotic medication has been associated with polymorphism of DRD3
and DRD2.
E. Heat shock protein 30-1 in clozapine-induced agranulocytosis is well documented.
361. In a 23-year-old, single unemployed woman with borderline personality disorder who has a
significant history of childhood emotional difficulties, which of the following phases of human
development has been mostly affected in her early life to develop this condition?
A. Rapprochement sub-phase
B. Object constancy
C. Practicing sub-phase
D. Differentiation sub-phase
E. None of the above
A. Treatment of mania
B. Prophylaxis for bipolar disorder
C. Prophylaxis for unipolar disorder
D. Maintenance therapy for bipolar disorder
E. Augmentation in treatment-resistant depression
363. A 23-year-old woman who has obsessive-compulsive disorder (OCD) attended her psychiatric
outpatient appointment with you. She revealed to you that she had a throat infection in the past with
complications. Which is of the following is most true in OCD?
364. In assessing intelligence, which of the followings is least likely to be part of the verbal component of
the Wechsler Adult Intelligence Scale (WAIS-III)?
A. Arithmetic
B. Information
C. Letter number sequencing
D. Digit span
E. Digit symbol
172
363. Answer: E. Sleep EEG abnormality in OCD patients is comparable with those
that have depression. Streptococcal infection has been associated
with OCD. PET studies have demonstrated increased activity in the
frontal lobe, basal ganglia and the cingulum. (25, p 14)
364. Answer: E. Wechsler Adult Intelligence Scale (WAIS-III) has two scales – verbal
and performance with seven tests each. The verbal scale includes
arithmetic, comprehension, digit span, information, letter number
sequencing, similarities and vocabulary. The performance scale
includes block design, digit symbol, matrix reasoning, object
assembly, picture arrangement, picture completion and symbol
search. (14, p 97)
173
365. A 19-year old man was recently diagnosed with paranoid schizophrenia. The risk of this man
attempting suicide is best described by which of the following percentages?
A. 1%
B. 5%
C. 15%
D. 50%
E. 80%
366. Which of the following assessment tools would be least suitable for studying psychiatric morbidity in
a large population?
367. A 20-year-old ballet dancer presents to you with symptoms suggestive of a depressive episode. She
is agreeable to start antidepressant medication. Which of the following antidepressants is most likely
to cause weight loss in this woman?
A. Sertraline
B. Fluoxetine
C. Citalopram
D. Amitriptyline
E. Clomipramine
368. Which one of the followings is least likely be part of the limbic lobe?
A. Septal nucleus
B. Cingulate gyrus
C. Parahippocampal gyrus
D. Subcallosal gyrus
E. Caudate nucleus
370. Which of the following is less likely to be associated with increased risk of suicide?
A. Unemployment
B. Physical illness
C. Alcoholism
D. Religious affiliation
E. Prison inmates
174
367. Answer: B. While fluoxetine is associated with weight loss, paroxetine and
fluvoxamine are known to cause slight weight loss. Sertraline has
been noticed to cause limited weight gain while citalopram has no
effect on weight. Tricyclic antidepressants are all associated with
some weight gain. (17, p 853)
368. Answer: E. The caudate nucleus is part of the corpus striatum, which is one of
the components of the basal ganglia. The limbic lobe is composed
of cortical areas including the cingulate gyrus, parahippocampal
gyrus, subcallosal gyrus, and amygdaloidal and septal nuclei.
(19, pp 111–12)
370. Answer: D. Religious affiliation is a protective factor against suicide. On the other
hand, alcoholism, psychiatric morbidity, personality disorder, physical
illness and being a prison inmate are all associated with increased
rates of suicide. (21, pp 150–2)
175
371. A 50-year-old patient with schizophrenia living in a supported hostel is brought to the Accident
and Emergency Department by a nurse from the hostel. He had been well until recently. He is
complaining of headache and blurred vision, and is reported to have vomited twice that morning. On
mental state examination, he describes persecutory paranoid beliefs but is fully orientated. Nursing
staff note that he is more thirsty in the past few weeks. His blood glucose is normal. What other
biochemical marker is most likely to be abnormal?
A. Bicarbonate
B. Chloride
C. Potassium
D. Sodium
E. Urea
372. A patient with schizophrenia who has good symptom control for several years on thioridazine
develops abnormal movements, including lip-smacking and tongue protrusion. Changing to which
medication is most likely to result in an improvement in these symptoms?
A. Aripiprazole
B. Clozapine
C. Olanzapine
D. Quetiapine
E. Risperidone
373. A 30-year-old man whom you commenced on amitriptyline four months ago has reported
experiencing problems with erectile dysfunction. You agree to switch to an alternative
antidepressant. Which of the following would be the most appropriate medication to minimise this
adverse effect?
A. Duloxetine
B. Paroxetine
C. Phenelzine
D. Trazodone
E. Venlafaxine
374. A 24-year-old woman with a history of depressive disorder has developed a post-natal depressive
episode. She is keen to breastfeed and asks your opinion on an appropriate medication. Which
would be your treatment of choice?
A. Doxepin
B. Escitalopram
C. Fluoxetine
D. Nefazodone
E. Paroxetine
176
371. Answer: D. The most likely diagnosis given the history and exclusion of diabetes
mellitus is water intoxication, secondary to illness-related polydipsic
behaviour. This has been measured at prevalence rates of 5% in
some groups of chronic patients. The diagnosis is based on a history
of polydipsia and hyponatraemia at <120 mmol/L. (15, p 582)
372. Answer: B. The patient displays evidence of tardive dyskinesia, more common
with typical antipsychotic treatment. The most appropriate treatment
would be to switch to an alternative antipsychotic medication. While
all of the above have demonstrated some effect, the evidence most
strongly supports the use of clozapine. (2, p 99)
373. Answer: D. Most antidepressants have been shown to have a risk of adverse
effects on sexual function, be it erectile problems or libido,
independent of the underlying depressive illness. With respect to
erectile problems alone, all of the above hold varying risk of such,
except for trazodone, which can actually be used to promote
erections and which also carries a risk of priapism. (2, pp 231–2)
374. Answer: E. All of these antidepressants are excreted in breast milk, and there
are reports of mild-to-moderate effects on the infant in some case
reports. For newer medications, the evidence base is more limited.
Paroxetine is recommended as the antidepressant of choice where
pharmacological treatment is indicated. (2, pp 377–80)
177
375. A 25-year-old woman you have been treating for depression for eight months appears resistant to
all trials of pharmacological and psychotherapeutic treatment so far. You have never been able to
develop more than a superficial rapport. Her initial attitude of having great faith in your abilities to
treat her has now changed to challenging this, not believing you can help her and blaming you for a
lack of improvement. She has also indulged in binges of substance abuse, which she later regrets
and seeks help for. She has few social supports, having alienated friends with confrontational and
erratic behaviour, although she has not been violent towards others. What would be the most likely
personality disorder in keeping with the above description?
376. A 35-year-old man on lithium for one year for the treatment of bipolar disorder is brought to Accident
and Emergency by his family due to concerns related to a deterioration in his mental state. Following
examination you suspect lithium toxicity. Which of the following would be of most concern?
A. Diarrhoea
B. Coarse tremor
C. Impaired concentration
D. Marked apathy
E. Polydipsia
377. A 45-year-old man with diagnosed alcohol dependence syndrome has had recurrent admissions for
detoxification. He has a history of poor self-control due to strong cravings and relapses quickly. He
agrees to pharmacological treatment. Which of the following would be the most appropriate?
A. Acamprosate
B. Disulfiram
C. Low-dose chlordiazepoxide
D. Naltrexone
E. Topiramate
378. A 30-year-old woman presents with a six-month history of low mood, associated with low self-
esteem, social withdrawal and frequent guilt feelings about her current functional state. Her
premorbid function was good, she is in a stable and supportive marriage, there are no relevant major
events in her history, and there is no evidence of personality disorder. What form of psychotherapy
would be most appropriate for her?
178
376. Answer: D. All of these symptoms are associated with various degrees of toxicity,
but marked apathy is the one most associated with severe toxicity.
(15, p 36)
377. Answer: A. Given the instability of his substance use, disulfiram would present
too much of a risk to use, or is likely to be discontinued in favour
of alcohol use. Acamprosate is the most appropriate and safest
licensed medication to use to control cravings. Naltrexone and
topiramate have both shown evidence for helping to reduce cravings.
Benzodiazepines are not recommended for long-term use. (2, p 315)
378. Answer: B. Her symptomatology and ideation appear only related to the
current depressive episode, and there is no evidence of significant
interpersonal difficulties or premorbid traumatic events impacting on
her mental state. Supportive therapy may help, but given the strong
association between her current cognitive state and her mood, CBT
would be the most appropriate model in this case. (14, p 1394)
179
379. Which of the following agents is most likely to increase clozapine metabolism via induction of
CYP1A2?
A. Carbamazepine
B. Cigarette smoking
C. Ciprofloxacin
D. Fluvoxamine
E. Phenytoin
380. A patient has a history of QTc prolongation on sertindole. What would be the most appropriate
antipsychotic medication to reduce risk of recurrence of this?
A. Aripiprazole
B. Clozapine
C. Haloperidol
D. Olanzapine
E. Quetiapine
381. An 80-year-old woman with a history of chronic obstructive pulmonary disease and right heart
failure, the latter treated with diuretics, presents with depressive disorder. There is no history of
stroke. You intend to prescribe a selective serotonin reuptake inhibitor. Which adverse event would
you be most concerned about monitoring?
A. Bleeding disorder
B. Diabetes mellitus
C. Hyponatraemia
D. Sedation
E. Urinary retention
382. A patient has not responded to an adequate trial of his current antidepressant treatment. You have
recently learned that he was previously treated successfully with tranylcypromine and wishes to use
this again. He complied well with necessary requirements of using this previously and you agree to
this switch. Based on his current medication, which of the following would be the most appropriate
switching regime?
383. A patient with a diagnosis of Pick’s disease presents to Accident and Emergency, referred by his
general practitioner with suspected Klüver–Bucy syndrome. Which feature is most consistent with
this?
A. Childish behaviour
B. Elated mood
C. Hypermetamorphosis
D. Partial seizures
E. Visual hallucinations
180
380. Answer: A. All of the options except for aripiprazole have been seen to confer
various degrees of risk, from haloperidol with the highest, then
quetiapine, olanzapine and clozapine. (2, p 117)
381. Answer: C. Given her age and her diuretic treatment, there is a high risk of
hyponatraemia. While there is some degree of risk of coagulation
problems or urinary retention, these would be less likely in this
patient. Some degree of reduced risk of diabetes is seen with SSRIs.
(2, pp 210–23)
181
384. A patient presents to you referred by their general practitioner with depression. You discover that
they have been prescribed diazepam for the past four months. Which of the following features would
make you most suspicious of dependence?
385. A patient informs you that the group psychotherapy he is attending for emotionally unstable
personality disorder is going well, although he had initially had problems when starting several weeks
ago. Which of the following best represents an early stage in the group development?
A. Deconstruction
B. Differentiation
C. Imitation
D. Independence
E. Individualisation
387. You are asked to see a confused elderly patient on the general medical ward who came in last night.
In your assessment, what feature would make you more likely to diagnose a delirium rather than a
dementia?
A. Disorientation
B. Fluctuating confusion during admission
C. Hallucinatory phenomena
D. Impoverished thinking
E. The patient is alert
388. Which of the following characteristics would most likely suggest an individual would engage well with
a cognitive behavioural model of therapy?
182
384. Answer: B. While all of these items would be consistent with dependence,
increased tolerance is the only feature that is diagnostic, in
association with other features. It is not clear from the description
what the motivation for reinstatement is. This may reflect abuse rather
than dependence. (1, pp 75–6)
385. Answer: C. Options B and C are recognised stages of development in the group
process, with imitation forming the earliest stage as individuals start
to coalesce and recognise shared features, and differentiation during
the final phase as individuals begin to separate from the group
identity. Deconstruction may occur later during the internalising
phase, as thoughts are reflected on, then reconstructed. (14, p 1453)
387. Answer: B. All of these features can be present in either condition; however,
given the fluctuation in confusion within such a short space of time,
this presentation is more consistent with delirium than a dementia.
(15, p 512)
388. Answer: B. Options A, C and E are not necessary features to have for
engagement in cognitive behavioural therapy, as it is mainly
focused on the ‘here and now’ symptomatology and rarely requires
exploration into deeper and past issues as in psychodynamic
therapies. While acceptance of a formal diagnosis of depression is
preferred, and is often implied with good engagement, some patients
may have a different understanding of the cause for their symptoms.
They may still accept and use the CBT model as a means to
recovery. (15, pp 129–34)
183
389. Which of the following is most true with respect to epidemiological measures?
A. Incidence refers to the number of cases present in a population at or within a specific point or
period in time.
B. Reliability of a test is the degree to which it measures what it purports to measure.
C. The higher the sensitivity of a test, the lower the false-negative rate.
D. Numerator error is an error in measuring the baseline population from which the cases are
taken.
E. Validity is potentially affected if more than one rater administers a test.
390. A prospective mother is concerned about the risk of teratogenicity if she gets pregnant while on
lithium. Which of the following best describes the risk of Ebstein’s anomaly associated with lithium
treatment in pregnancy?
A. 1:10
B. 1:100
C. 1:1000
D. 1:2000
E. 1:500
392. A 45-year-old man in the medical ward on linezolid antibiotics developed confusion, muscle rigidity
and hyperthermia shortly after he was placed on citalopram for low mood. What is the most likely
diagnosis?
A. Serotonin syndrome
B. Drug overdose
C. Alcohol withdrawal state
D. Metabolic disturbance
E. Infective process
393. A 25-year-old female factory worker was treated with risperidone following a three-week history
of third-person auditory hallucinations and persecutory delusions. Several days later, she became
febrile with fluctuating blood pressure, impaired consciousness and muscle rigidity. What is the most
likely diagnosis?
184
389. Answer: C. Option A refers to prevalence, not incidence, which is the number of
new cases developing within a specified period. B defines validity,
not reliability, which measures the consistency of results obtained on
repeating a test (e.g., either at different times or by different raters),
thus in option E it is reliability that may be affected. Option D defines
denominator error, as the numerator refers to the number of cases,
not the total population being examined. (15, pp 698–700)
391. Answer: D. Fragile X syndrome is the second most common cause of learning
disability after Down’s syndrome, and it is the most common inherited
cause of learning disability. Features include learning disability, macro-
orchidism, large ears, hypertelorism and blue eyes. (26, p 256)
185
394. Which of the following laboratory investigations will help most in arriving at the diagnosis in a patient
with suspected neuroleptic malignant syndrome?
395. A 30-year-old petrol attendant with a three-year history of schizophrenia recalled seeing a white cat
smiling at him on date 06/06/06, which led to his psychotic symptoms. He strongly believes that
date is synonymous with the devil. How is this psychopathology is best described?
A. A persecutory delusion
B. A delusional belief
C. A delusional memory
D. A delusional mood
E. A normal belief
396. A 63-year-old man presents with bradykinesia, limb rigidity, unsteady gait, visual hallucinations and
fluctuating cognitive impairment on clinical examination. What is the most likely diagnosis?
397. Which of the following finding would you most likely expect in a 23-year-old woman who has been
diagnosed with anorexia nervosa?
A. Increased leptin
B. Decreased cortisol
C. Increased T3
D. Increased growth hormone
E. Increased libido
398. In patients with post-traumatic stress disorder, which of the following statements is most true?
A. Hyperactivity of medial prefrontal or anterior cingulate networks in regulating the amygdala have
been implicated.
B. Saccadic eye movement control is a useful treatment.
C. Eye movement desensitisation and reprocessing is a proven treatment (EMDR).
D. Borderline personality disorder is not a risk factor.
E. Being male is a risk factor.
186
394. Answer: A. Serum creatinine kinase is always raised, leucocytosis with a left shift
and disordered liver function tests are present. (2, pp 11, 103–6)
397. Answer: D. Increased growth hormone and cortisol, reduced T3 and libido are
common findings in patients with anorexia nervosa. Decreased leptin
is associated with anorexia nervosa. (26, p 90)
398. Answer: C. Evidence-based studies have shown the efficacy of eye movement
desensitisation and reprocessing (EMDR). Borderline personality
disorder and being female are associated risk factors. Underactivity
of medial prefrontal or anterior cingulate networks in regulating the
amygdala has been implicated. (31)
187
399. In patients with Parkinson’s disease, which of the following is most true?
A. Hypermetabolism in caudate, inferior orbitofrontal and medial frontal regions are observed in
positron emission tomography.
B. Dopamine agonists have been implicated in pathological gambling.
C. Auditory hallucinations are major psychotic symptoms.
D. The frequency of psychiatric symptoms is 20%.
E. Hypomania is a major neuropsychiatric disorder.
400. A 40-year-old man with a history of alcohol liver disease presents in an alcohol withdrawal state.
Which of the following is your preferred choice of benzodiazepine treatment for his withdrawal
symptoms?
A. Diazepam
B. Oxazepam
C. Chlordiazepoxide
D. Flurazepam
E. All of the above
188
189
References
1. World Health Organization. The ICD-10 Classification of Mental and Behavioural Disorders: clinical
descriptions and diagnostic guidelines. Geneva: World Health Organization; 1992.
2. Taylor D, Paton C, Kerwin R. The South London and Maudsley NHS Foundation Trust Oxleas NHS
Foundation Trust: prescribing guidelines. 9th ed. London: Informa Healthcare; 2007.
3. Sadock B, Kaplan H, Sadock V. Kaplan & Sadock's Synopsis of Psychiatry: behavioral sciences. 10 ed.
Philadelphia, PA: Wolter Kluwer/Lippincott Williams & Wilkins; 2007.
4. Johnstone E, Owens D, Lawrie S, Sharpe M, Freeman C. Companion to Psychiatric Studies. 7th ed.
Edinburgh; New York: Churchill Livingstone; 2008.
5. Stahl S. Stahl's Essential Psychopharmacology. 3rd ed. Cambridge: Cambridge University Press; 2008.
6. Sadock BJ, Sadock VA, Kaplan HI. Kaplan & Sadock's Comprehensive Textbook of Psychiatry. 8th ed.
Philadelphia, PA: Lippincott Williams & Wilkins; 2005.
7. Andreasen N, Black D. Introductory Textbook of Psychiatry. Washington, DC: American Psychiatric
Publishing; 2006.
8. Puri BK, Hall AD. Revision Notes in Psychiatry. London: Arnold; 2004.
9. Wright P, Stern J, Phelan M. Core Psychiatry. London: WB Saunders; 2000.
10. Wright P, Stern J, Phelan M. Core Psychiatry. 2nd ed. Philadelphia, PA: Elsevier; 2005.
11. Korneva EA, Shanin SN, Rybakina EG. The role of interleukin-1 in stress-induced changes in immune
system function. Neurosci Behav Physiol. 2001; 31(4):431–7.
12. Freeman CPL, Zealley AK. Companion to Psychiatric Studies. 6th ed. Johnstone EC, editor. Edinburgh:
Churchill Livingstone; 1998.
13. Stein G, Wilkinson G. Seminars in General Adult Psychiatry. London: Royal College of Psychiatrists; 1998.
14. Gelder MG, López Ibor JJ, Andreasen NC. New Oxford Textbook of Psychiatry. Oxford: Oxford University
Press; 2000.
15. Stein G, Wilkinson G. Seminars in General Adult Psychiatry: London: RCPsych Publications; 2007.
16. Eysenck M. Simply Psychology. Hove, East Sussex, UK: Psychology Press; 1996.
17. Semple D, Smyth R, Burns J, Darjee R, Mc Intosh A. Oxford Handbook of Psychiatry. 1st ed. Oxford
University Press; 2005.
18. Kaplan H, Sadock B, Sadock V. Synopsis of Psychiatry: behavioral sciences. Philadelphia, PA: Lippincott
Williams & Wilkins; 2003.
19. Puri B, Hall A. Revision Notes in Psychiatry. London: Arnold; New York: Oxford University Press; 1998.
20. Lawlor B. Revision Psychiatry. Dublin: MedMedia; 2001.
21. Buckley P, Bird J, Harrison G. Examination Notes in Psychiatry: a postgraduate text. Oxford Butterworth-
Heinemann; 1995.
22. Singh T, Williams K. Atypical depression. Psychiatry (Edgmont). 2006; 3(4):33–9.
23. Dayalu P, Chou KL. Antipsychotic-induced extrapyramidal symptoms and their management. Expert Opin
Pharmacother. 2008; 9(9):1451–62.
24. Sadock B, Sadock V. Kaplan & Sadock's Comprehensive Textbook of Psychiatry. 7th ed. Philadelphia, PA:
Lippincott Williams & Wilkins; 2000.
25. Maj M, Sartorius N, Okasha A, Zohar J. Obsessive-Compulsive Disorder. Chichester, UK; Hoboken, NJ:
Wiley; 2002.
26. Buckley P, Prewette D, Byrd J et al. Examination Notes in Psychiatry. 4th ed. Boca Raton, FL: CRC Press;
2003.
27. Huang V, Gortney J. Risk of serotonin syndrome with concomitant administration of linezolid and serotonin
agonists. Pharmacotherapy. 2006; 26(12):1784–93.
28. Morales-Molina J, Mateu-de Antonio J, Marín-Casino M, Grau S. Linezolid-associated serotonin syndrome:
what we can learn from cases reported so far. J Antimicrob Chemother. 2005; 56(6):1176–8.
29. Sims A. Symptoms in the Mind: an introduction to descriptive psychopathology. 3rd ed. London:
WB Saunders; 2003.
30. Byrne J. Dementia with Lewy bodies. Adv Psychiatr Treat. 1998; 4(6):360–3.
31. Shapiro F. Eye movement desensitization and reprocessing (EMDR): evaluation of controlled PTSD
research. J. Behav Ther Exp Psychiatry. 1996; 27(3):209–18.
32. Steeves TD, Miyasaki J, Zurowski M et al. Increased striatal dopamine release in parkinsonian patients
with pathological gambling: a [11C] raclopride PET study. Brain. 2009; 132(Pt 5):1376–85.
190
191
192
193
194
195
I M
Immune system and stress, 154–5 Malingering
Implicit memory, 118–9 amnestic disorder and, 44–5
Impotence, 6–7 with schizoaffective disorder, 2
Injury, head, 114–5 Manic depression, 20
Insecure-avoidant attachment, 50–1 Manifest content of dreams, 152–3
Insomnia, 162–3 Memory
Intelligence assessment, 172–3 delusional, 36–7, 186–7
Interpersonal therapy (IPT), 69–70, 138–9 implicit, 118–9
Interviewing, motivational, 74–5 loss, 28, 44
Intoxication, water, 174–5 procedural, 112–3, 148–9
Intramuscular administration of drugs, 170–1 Messenger RNA, 129
Intuition, delusional, 36–7 Metabolism, drug, 170–1
IPT. See Interpersonal therapy (IPT) Metapsychological concepts, 128–9
IQ tests, 76–7 Methadone, 94–5, 148–9
safety during pregnancy, 148–9
K Methylphenidate, 124–5
Ketamine, 164–5 Metonymy, 33
Kinaesthetic hallucinations, 13 Mindfulness-based psychotherapy, 82–3
Klüver–Bucy syndrome, 180–1 Minnesota Multiphasic Personality Inventory
Koro, 40–1 (MMPI), 13, 164–5
Korsakoff’s syndrome, 14–5 Mirroring, 166–7
Mirtazapine, 13, 64–5, 76–7
L sedation with, 100–1
Lamotrigine, 86–7 sexual side effects with, 110–1
Latah, 41 Monoamine oxidase inhibitor antidepressants,
Latent content of dreams, 152–3 54, 57
Learning theory, 76–7 Motivational interviewing, 74–5
Lesch-Nyhan syndrome, 162–3 Movement disorders, 12–3
Lewy body dementia, 66–7, 80–1, 186–7 Multiple sclerosis, 17
Libido, loss of, 12–3, 38–9 Myclonic movements, 13
schizophrenia and, 134–5 Myocardial infarction, 102–3
Likert scale, 8–9
Limbic system, 33, 112–3, 174–5 N
Linezolid antibiotics, 184–5 Narcissistic personality disorder, 21–2
Lithium, 9, 54–5, 106–7 Narcolepsy, 17
bipolar affective disorder and, 10–1 National Comorbidity Survey, 70–1
breastfeeding and, 120–1 National Confidential Inquiries into Suicide and
haemodialysis and, 57 Homicide by People with Mental Illness,
half-life, 55 142–3
hypertension and, 132–3 Neurofibromatosis, 60–1
least effective scenarios for, 172–3 Neurohormones, 126–7
neutropenia and, 132–3 Neuroimaging, 15–7, 168–9
pregnancy and, 184–5 Neuroleptic malignant syndrome (NMS)
toxicity, 178–9 diagnosis, 21–2, 184–5
for treatment-resistant depression, 112–3 mortality rate, 90–1
tremors with, 46–7 Neurotransmitters, 70–1
Liver metabolism, 102–3 Neutropenia, 132–3
Lofepramine, 150–1 Niemann-Pick disease, 60–1
Logloclonia, 36–7 Nightmares, 32
Lorazepam, 54–5 Night terrors, 17
for catatonia, 118–9 Noradrenergic receptors, 96–7
196
197
Q residual, 50–1
QTc interval, 108–9 speech disorders, 36–7
antipsychotics and, 114–5, 122–3, susceptibility genes, 100–1
180–1 thought disorders, 44–5
Questionnaire bias, 8–9 treatment-resistant, 132–3
Quetiapine, 64–67, 108–9 undifferentiated, 50–1
Schizotypal personality disorder, 22
R Seasonal affective disorder, 14–5, 60–1
Ramsay-Hunt syndrome, 17 Secondary revision, 152–3
Rancho Los Amigos Scale (RLAS), 124–5 Secure attachment, 50–1
Rapid eye movement (REM) sleep, 166–7 Sedation
Reciprocal inhibition, 138–9 with antidepressants, 100–1
Reflex hallucinations, 6–7 with antipsychotics, 110–1
Representative error, 96–7 Seizures, 18–9
Repression, 94–5 epileptic, 42–3
Residual schizophrenia, 50–1 tricyclic antidepressants and, 128–9
Response prevention, 138–9 withdrawal, 5
Resting membrane ion permeability, 156–7 Selective serotonin reuptake inhibitors (SSRIs), 90–1
Restlessness, 2–3 half-lives, 143
Revision, secondary, 152–3 side effects, 128–9
Rhythms, EEG, 72–3 Self-harm, 4, 17, 38. See also Suicide
Ribonucleic acid (RNA), 128–9 dialectical behaviour therapy and, 138–9
Risperidone, 64–7, 184–5 personality assessment and, 164–5
schizoaffective disorder and, 2–3 Self-rated assessment tools, 174–5
sexual side effects, 134–5 Sensorimotor stage, 140–1
Sensory aphasia, 160–1
S Separation individuation, 172–3
Schizoaffective disorder, 2–3 Serotonin
breastfeeding and, 2 degradation, 36
clang association, 33 effect on sleep and temperature, 124–5
malingering versus factitious disorder, 2 receptors, 104–5
neutropenia with, 132–3 syndrome, 22, 64–5, 184–5
urinary retention with, 8 Sertraline, 54–5, 57, 64–5
Schizophrenia, 12, 20 weight loss with, 174–5
biochemical markers, 174–5 Serum prolactin, 90–1, 106–7
Capgras syndrome, 25 Sexuality
catatonic, 50–1 disorders, female, 34–5
CATIE trial and treatment of, 62–3 drug side effects and, 7, 12–3, 38–9, 108–11,
delusional perception in, 21 174–5
diagnosis, 74–5 Erickson’s stages of development of, 140–1
disorganisation syndrome, 38–9 ICD-10 classifications of disorders of, 148–9
drug noncompliance, 134–5, 158–9 loss of libido, 12–3, 38–9, 134–5
familial risk of, 21–2 schizophrenia and, 134–5
genetics and, 80–1, 100–1, 116–7, Simultaneous conditioning, 9
134–5 Sleep
involuntary admission for, 48 cycle and EEG, 44–5
medications, 28, 64–5 drunkenness, 17
movement abnormalities with, 23 paralysis, 32–3
paranoid, 50–1, 116–7, 122–3, 174–5 rapid eye movement (REM), 166–7
post-mortem brains of patients serotonin effect on, 124–5
with, 78–9 transitional state between wakefulness and,
prevalence, 126–7 130–1
198
199